Quants

You might also like

Download as pdf or txt
Download as pdf or txt
You are on page 1of 436

Math 

Calculator  Flag 
0:01

Choose the option that best answers the question.


√​2
x2
If f(x) = + 4 and f(2k) = 36, then which of
the following is one possible value of k?
2

4

2√​2

√​14

EndPractice
End Practice and
and See
See Results
Results Question 1 of 737 Submit Answer
Submit Answer Next Question

Math 
Calculator  Flag 
0:01

Choose the option that best answers the question.

1
The numbers {a, b, c } are three positive
a×b×c
integers.  If
​  equals an integer and 2
14
b×c
​  equals an integer, what is the
4 4
smallest possible integer
value of a ?
7

14
EndPractice
End Practice and
and See
See Results
Results Question 2 of 737 Submit Answer
Submit Answer Next Question

Math 
Calculator  Flag 
0:01

Enter the answer as a fraction. Fractions do not need to be in simplest form

The first term in a certain sequence is 1,


and for all integers n ≥ 2, the nth term in
the sequence is the sum of 1 plus the
reciprocal of the previous term in the
sequence.  What is the value of the sixth
term in this sequence?

Give your answer as a fraction.

EndPractice
End Practice and
and See
See Results
Results Question 3 of 737 Submit Answer
Submit Answer Next Question

Math 
Calculator  Flag 
0:01

Choose the correct statement.

The quantity in Column A is greater


Column A Column B
The quantity in Column B is greater
The number of The number of
distinct prime distinct prime The two quantities are equal
factors of 206 factors of 3210
The relationship cannot be
determined from the information
given
EndPractice
End Practice and
and See
See Results
Results Question 4 of 737 Submit Answer
Submit Answer Next Question

Math 
Calculator  Flag 
0:01

Choose the option that best answers the question.

A developer proposes converting a


In the downtown of a certain city, there are gigantic old warehouse complex into
8,000 apartments for rent. Here is their apartments. The proposed new building
breakdown by total area: would add 250 economy apartments
(area = 625 sq. ft.), 200 regular
apartments (area = 925 sq. ft.), and 50
luxury apartments (area = 1800 sq. ft.).
If these apartments are added, then
apartments with an area of 750 – 1000
sq. ft. will constitute what percent of the
total number of apartments downtown.

16%

24%

35%

50%

72%

EndPractice
End Practice and
and See
See Results
Results Question 5 of 737 Submit Answer
Submit Answer Next Question

Math 
Calculator  Flag 
0:02
Choose the option that best answers the question.

The median area falls into what group?


In the downtown of a certain city, there are
8,000 apartments for rent. Here is their 500 – 750 sq. ft.
breakdown by total area:
750 – 1000 sq. ft.

1000 – 1250 sq. ft.

1250 – 1500 sq. ft.

1500 – 1750 sq. ft.

EndPractice
End Practice and
and See
See Results
Results Question 6 of 737 Submit Answer
Submit Answer Next Question

Math 
Calculator  Flag 
0:01

Choose the option that best answers the question.

What is the total number of apartments


In the downtown of a certain city, there are with area between 500 and 1000 sq.
8,000 apartments for rent. Here is their ft.?
breakdown by total area:
1440

1840

2080

3280

3920
EndPractice
End Practice and
and See
See Results
Results Question 7 of 737 Submit Answer
Submit Answer Next Question

Math 
Calculator  Flag 
0:01

Choose the option that best answers the question.

​ ​ ​ ​
Which of the following is the correct 2√​13 < 3√​6 < 5√​2 < 4√​3
​ ​ ​ ​
ordering of 2√​13, 4√​3, 5√​2 and  3√​6? ​ ​ ​ ​
3√​6 < 5√​2 < 4√​3 < 2√​13
​ ​ ​ ​
4√​3 < 5√​2 < 2√​13 < 3√​6
​ ​ ​ ​
5√​2 < 4√​3 < 3√​6 < 2√​13
​ ​ ​ ​
2√​13 < 3√​6 < 4√​3 < 5√​2

EndPractice
End Practice and
and See
See Results
Results Question 8 of 737 Submit Answer
Submit Answer Next Question

Math 
Calculator  Flag 
0:01

Choose the correct statement.

The quantity in Column A is greater


An orchard contains only cherry trees,
apple trees and peach trees. The ratio of
The quantity in Column B is greater
apple trees to peach trees is 2:3, and the
ratio of cherry trees to peach trees is 2:1. The two quantities are equal
There are 33 trees altogether.
The relationship cannot be
Column A Column B determined from the information
given
Number of peach trees 8

EndPractice
End Practice and
and See
See Results
Results Question 9 of 737 Answer
Next Question
Submit Answer
Submit

Math 
Calculator  Flag 
0:01

Choose the option that best answers the question.

m12/5
For all positive numbers x, Δx is defined as
the cube root of x, and ∇x is defined as the
m6
square root of x. If ∇(Δk) = m2 , then k =
m12

m36

m64

EndPractice
End Practice and
and See
See Results
Results Question 10 of 737 Answer
Next Question
Submit Answer
Submit

Math 
Calculator  Flag 
0:03

Choose the correct statement.

The quantity in Column A is greater


Clyde drove 30 miles in 20 minutes, and
then drove an additional 10 miles in 10
The quantity in Column B is greater
minutes.
The two quantities are equal
Column A Column B
The relationship cannot be
Clyde’s average speed for 75 miles
determined from the information
the entire trip. per hour
given

EndPractice
End Practice and
and See
See Results
Results Question 11 of 737 Submit Answer
Submit Answer Next Question

Math 
Calculator  Flag 
0:03

Choose the option that best answers the question.

48
Three friends are buying a gift for a friend.
Declan contributes 4 dollars more than
1 54
​  the cost of the gift, Ed contributes 1
4
1 60
dollar less than ​  the cost of the gift, and
3
Frank contributes the remaining 22 dollars. 66
What is the cost of the gift?
72

EndPractice
End Practice and
and See
See Results
Results Question 12 of 737 Submit Answer
Submit Answer Next Question

Math 
Calculator  Flag 
0:05

Consider each of the choices separately and select all that apply.

1
In the standard x,y-plane, Line L passes
through the point (1, 3). If Line L passes
1/2
below the point (4, 5), then which of the
following could be the slope of Line L?  2/3
Indicate all possible slopes.
3/5

4/5

4/7

5/4

5/6

EndPractice
End Practice and
and See
See Results
Results Question 13 of 737 Answer
Next Question
Submit Answer
Submit

Math 
Calculator  Flag 
0:05

Enter the answer in the blank.

In the
diagram, BC = CD = BD, angle FDE
= 40°, angle EFD
= 75°, and line BG is
parallel to line DF. What is
the measure
of angle A
in degrees?
EndPractice
End Practice and
and See
See Results
Results Question 14 of 737 Submit Answer
Submit Answer Next Question

Math 
Calculator  Flag 
0:01

Choose the correct statement.

The quantity in Column A is greater

The quantity in Column B is greater

The two quantities are equal

The relationship cannot be


Column A Column determined from the information
B given

Number of different triangles 42


possible using the given points
as vertices.
EndPractice
End Practice and
and See
See Results
Results Question 15 of 737 Submit Answer
Submit Answer Next Question

Math 
Calculator  Flag 
0:01

Choose the option that best answers the question.

62
Ten students wrote a test, and the
distribution of scores is shown on the
65
frequency table. If the average (arithmetic
mean) score is 62, what is the value of x ? 71

76

83

EndPractice
End Practice and
and See
See Results
Results Question 16 of 737 Submit Answer
Submit Answer Next Question

Math 
Calculator  Flag 
0:02

Choose the correct statement.

The quantity in Column A is greater


w , x and y are consecutive even integers.

wxy = 0 The quantity in Column B is greater

w<x<y The two quantities are equal


Column A Column B The relationship cannot be
determined from the information
x 0
given

EndPractice
End Practice and
and See
See Results
Results Question 17 of 737 Answer
Next Question
Submit Answer
Submit

Math 
Calculator  Flag 
0:01

Choose the option that best answers the question.

In 2006, the ratio of the number of


widgets sold by Company C, Company
E (not shown) and Company D was 5
to 8 to 2, respectively. How many
widgets did Company E sell in 2006?

300,000

600,000

2,400,000

4,800,000

6,000,000

EndPractice
End Practice and
and See
See Results
Results Question 18 of 737 Answer
Next Question
Submit Answer
Submit

Math 
Calculator  Flag 
0:02
Choose the option that best answers the question.

In 2005, Company C sold what percent


of the widgets sold by the four
companies listed?

24

25

30

37.5

42.9

EndPractice
End Practice and
and See
See Results
Results Question 19 of 737 Submit Answer
Submit Answer Next Question

Math 
Calculator  Flag 
0:01

Choose the option that best answers the question.

31.25
Increasing the original price of a certain
item by 25 percent and then increasing the
37.50
new price by 25 percent is equivalent to
increasing the original price by what 50.00
percent?
52.50

56.25

EndPractice
End Practice and
and See
See Results
Results Question 20 of 737 Submit Answer
Submit Answer Next Question
Math 
Calculator  Flag 
0:01

Choose the correct statement.

The quantity in Column A is greater


Column A Column
B The quantity in Column B is greater

The average (arithmetic mean) 34 The two quantities are equal


of 32, 34, 36
The relationship cannot be
determined from the information
given

EndPractice
End Practice and
and See
See Results
Results Question 21 of 737 Submit Answer
Submit Answer Next Question

Math 
Calculator  Flag 
0:02

Choose the option that best answers the question.

If ak – b = c – dk, then k = b+c−a−d

b+c+d

a
c b
​ +​
a d
b−c

a−d
b+c

a+d

EndPractice
End Practice and
and See
See Results
Results Question 22 of 737 Answer
Next Question
Submit Answer
Submit

Math 
Calculator  Flag 
0:02

Choose the option that best answers the question.

1.25x
A
circle has an area of x. If the diameter is
increased by 50%, what is the area
of the
1.5x
resulting circle in terms of x?
2x

2.25x

3x

EndPractice
End Practice and
and See
See Results
Results Question 23 of 737 Answer
Next Question
Submit Answer
Submit

Math 
Calculator  Flag 
0:01

Enter the answer in the blank.

The roots of an algebraic


expression are
the x-values that make the expression
equal to zero. What are the sum of the
roots of the
expression (x –
2)(x2 + 7x
+
12)?
EndPractice
End Practice and
and See
See Results
Results Question 24 of 737 Answer
Next Question
Submit Answer
Submit

Math 
Calculator  Flag 
0:01

Choose the correct statement.

Half of w is x The quantity in Column A is greater

Half of y is w The quantity in Column B is greater

w + x + y = 28 The two quantities are equal

Column A Column B The relationship cannot be


determined from the information
w 7
given

EndPractice
End Practice and
and See
See Results
Results Question 25 of 737 Answer
Next Question
Submit Answer
Submit

Math 
Calculator  Flag 
0:01

Choose the option that best answers the question.

38
At a certain company, 30 percent of the
male employees and 50 percent of the
52
female employees have an MBA. If 40
percent of the employees are female, what 54
percent of the employees do not have an
MBA? 62
85

EndPractice
End Practice and
and See
See Results
Results Question 26 of 737 Answer
Next Question
Submit Answer
Submit

Math 
Calculator  Flag 
0:01

Choose the correct statement.

The quantity in Column A is greater


For positive numbers j and k, 4j2 = 17k2.  

The quantity in Column B is greater


Column A Column B

The two quantities are equal


j 2k

The relationship cannot be


determined from the information
given

EndPractice
End Practice and
and See
See Results
Results Question 27 of 737 Answer
Next Question
Submit Answer
Submit

Math 
Calculator  Flag 
0:01

Choose the option that best answers the question.

Profit = Revenue – Costs. If costs


The following tables show the revenues & remain constant from 2007 to 2008,
costs, in thousands of dollars, for a small and if revenues increase by 10% in that
company in the year 2007. same period, by what percent will
profits increase from 2007 to 2008?
11.6%

25.8%

34.7%

71.2%

116.3%

EndPractice
End Practice and
and See
See Results
Results Question 28 of 737 Submit Answer
Submit Answer Next Question

Math 
Calculator  Flag 
0:01

Choose the option that best answers the question.

The CEO has promised that any


The following tables show the revenues & increase in revenues from investments
costs, in thousands of dollars, for a small in 2008 will go toward increasing the
company in the year 2007. R&D budget. Assume that revenues
from investments increase by 40% from
2007 to 2008, and that these additional
funds are the only change to the R & D
budget. By what percent does the R &
D budget increase?

2.8%
4.0%

11.5%

28.3%

56.6%

EndPractice
End Practice and
and See
See Results
Results Question 29 of 737 Submit Answer
Submit Answer Next Question

Math 
Calculator  Flag 
0:01

Choose the option that best answers the question.

The costs associated with insurance


The following tables show the revenues & and the physical plant are what
costs, in thousands of dollars, for a small percentage of total costs?
company in the year 2007.
6.8%

16.4%

27.2%

83.6%

90.0%
EndPractice
End Practice and
and See
See Results
Results Question 30 of 737 Submit Answer
Submit Answer Next Question

Math 
Calculator  Flag 
0:02

Choose the option that best answers the question.


​ 18√​2
√​8​ 1 + 81 + 81 + 81 + 81 + 81 + 81 + 81 =

36√​2

72

162√​2

648

EndPractice
End Practice and
and See
See Results
Results Question 31 of 737 Submit Answer
Submit Answer Next Question

Math 
Calculator  Flag 
0:01
Choose the correct statement.

The quantity in Column A is greater


n is a positive integer

The quantity in Column B is greater


Column A Column B

The two quantities are equal


(0.99)n 0.01

The relationship cannot be


determined from the information
given

EndPractice
End Practice and
and See
See Results
Results Question 32 of 737 Submit Answer
Submit Answer Next Question

Math 
Calculator  Flag 
0:01

Choose the option that best answers the question.

0.53
At Joe's candy store, the total cost of 1
gumball and 1 lollipop is $0.74. The total
0.59
cost of 1 chocolate bar and 1 lollipop is
$0.92. The total cost of 1 gumball and 1 0.63
chocolate bar is $1.24. What is the cost in
dollars of 1 chocolate bar? 0.67

0.71

EndPractice
End Practice and
and See
See Results
Results Question 33 of 737 Submit Answer
Submit Answer Next Question

Math 
Calculator  Flag 
0:01
Consider each of the choices separately and select all that apply.

-6
Which of the following is a value of x that
makes the expression 
-3
(x2​ + 5x + 6)(x2​ − 7x + 6)

(x2​ − 4x − 21) -2

equal to zero? 1
Indicate all possible values of x.
4

EndPractice
End Practice and
and See
See Results
Results Question 34 of 737 Submit Answer
Submit Answer Next Question

Math 
Calculator  Flag 
0:01

Enter the answer in the blank.


If √​8x2​ + 17 = 3x − 2 , what is the value of
2x?

EndPractice
End Practice and
and See
See Results
Results Question 35 of 737 Submit Answer
Submit Answer Next Question

Math 
Calculator  Flag 
0:01
Choose the correct statement.

The quantity in Column A is greater


Column A Column
B The quantity in Column B is greater

The remainder when positive 11 The two quantities are equal


integer x is divided by 11
The relationship cannot be
determined from the information
given

EndPractice
End Practice and
and See
See Results
Results Question 36 of 737 Submit Answer
Submit Answer Next Question

Math 
Calculator  Flag 
0:01

Choose the option that best answers the question.

75
In a certain town in Connecticut, the ratio of
NY Yankees fans to NY Mets fans is 3:2,
80
and the ratio of NY Mets fans to Boston
Red Sox fans is 4:5. If there are 300 90
baseball fans in the town, each of whom is
a fan of exactly one of those three teams, 120
how many NY Mets fans are there in this
town? 133

EndPractice
End Practice and
and See
See Results
Results Question 37 of 737 Submit Answer
Submit Answer Next Question
Math 
Calculator  Flag 
0:01

Choose the correct statement.

w , x , and y are positive integers. The quantity in Column A is greater

w + x + y = 90 The quantity in Column B is greater

Column A Column B The two quantities are equal

Average (arithmetic Median of w , The relationship cannot be


mean) of w , x , and y x , and y determined from the information
given

EndPractice
End Practice and
and See
See Results
Results Question 38 of 737 Submit Answer
Submit Answer Next Question

Math 
Calculator  Flag 
0:02

Choose the option that best answers the question.

–21
s1, s2, s3, s4, s5, ….

In the sequence above, each term after the 0.75


first term is equal to the preceding term
divided by a positive number p, such that p 1.5
> 1.  If s3 = 24 and s5 = 6, which of the
2
following is the value of s8?

EndPractice
End Practice and
and See
See Results
Results Question 39 of 737 Submit Answer
Submit Answer Next Question
Math 
Calculator  Flag 
0:01

Choose the correct statement.

The quantity in Column A is greater

The quantity in Column B is greater

The two quantities are equal

The relationship cannot be


Column A Column B
determined from the information
a+b+c+d e+f+g+h given

EndPractice
End Practice and
and See
See Results
Results Question 40 of 737 Submit Answer
Submit Answer Next Question
Math 
Calculator  Flag 
0:01

Choose the option that best answers the question.

100x
Don has x marbles. If y marbles are white, ​
x−y
what percent of Don’s marbles are NOT
white? 100(x − y)

x
x−y

100x
x

100(x − y)

100y

x

EndPractice
End Practice and
and See
See Results
Results Question 41 of 737 Submit Answer
Submit Answer Next Question

Math 
Calculator  Flag 
0:02

Consider each of the choices separately and select all that apply.

a + (b + c) = (a + b) + c
If a, b, and c are real numbers, which of the
following must be true?
a − (b − c) = (a − b) – c

a*(b*c) = (a*b)* c

a/(b/c) = (a/b)/c
EndPractice
End Practice and
and See
See Results
Results Question 42 of 737 Submit Answer
Submit Answer Next Question 

Math 
Calculator  Flag 
0:01

Enter the answer in the blank.

(1.37 x 1024) x (2.6 x 10–22) =

EndPractice
End Practice and
and See
See Results
Results Question 43 of 737 Submit Answer
Submit Answer Next Question

Math 
Calculator  Flag 
0:01

Choose the correct statement.

w + x + y = 21 The quantity in Column A is greater

The quantity in Column B is greater


Column A Column B

Average of x and y 7 The two quantities are equal

The relationship cannot be


determined from the information
given

EndPractice
End Practice and
and See
See Results
Results Question 44 of 737 Submit Answer
Submit Answer Next Question
Math 
Calculator  Flag 
0:01

Choose the option that best answers the question.

College C received approximately how


much in state appropriations per
student?

$9,000

$12,000

$19,000

$27,000

$39,000

EndPractice
End Practice and
and See
See Results
Results Question 45 of 737 Submit Answer
Submit Answer Next Question

Math 
Calculator  Flag 
0:07

Choose the option that best answers the question.

Suppose in the next year, 2007,


College D's expenses and enrollment
remain about the same, but in addition
to their current revenues, they receive
an additional $50,000,000 grant. This
would allow them to reduce average
tuition by how much?
$1388.89

$3571.43

$5555.56

$9500.00

$25888.89

EndPractice
End Practice and
and See
See Results
Results Question 46 of 737 Answer
Next Question
Submit Answer
Submit

Math 
Calculator  Flag 
0:06

Choose the option that best answers the question.

What is the total dollar amount that


College B received in grants?

$16,000,000

$48,000,000

$96,000,000

$160,000,000

$192,000,000
EndPractice
End Practice and
and See
See Results
Results Question 47 of 737 Answer
Next Question
Submit Answer
Submit

Math 
Calculator  Flag 
0:02

Choose the option that best answers the question.

10, 11, 12
Which of the following sets of numbers has
the greatest standard deviation?
–3, –4, –5

–2, 0, 2

5.1, 5.2, 5.3

20, 22, 22.5

EndPractice
End Practice and
and See
See Results
Results Question 48 of 737 Answer
Next Question
Submit Answer
Submit

Math 
Calculator  Flag 
0:10

Choose the correct statement.


The quantity in Column A is greater

The quantity in Column B is greater

The two quantities are equal

The relationship cannot be


The four small rectangles have the same determined from the information
dimensions given

Column A Column B

DC/BC 4/3

EndPractice
End Practice and
and See
See Results
Results Question 49 of 737 Submit Answer
Submit Answer Next Question

Math 
Calculator  Flag 
0:02

Choose the option that best answers the question.

$22
Nina has exactly enough money to
purchase 6 widgets. If the cost of each
$24
widget were reduced by $1.25, then Nina
would have exactly enough money to $30
purchase 8 widgets. How much money
does Nina have? $36

$40

EndPractice
End Practice and
and See
See Results
Results Question 50 of 737 Submit Answer
Submit Answer Next Question
Math 
Calculator  Flag 
0:02

Choose the correct statement.

Ashley's score was 20% higher than Bert's The quantity in Column A is greater
score. Bert's score was 20% lower than
The quantity in Column B is greater
Charles' score.
The two quantities are equal
Column A Column B
The relationship cannot be
Ashley's score Charles' score
determined from the information
given

EndPractice
End Practice and
and See
See Results
Results Question 51 of 737 Submit Answer
Submit Answer Next Question

Math 
Calculator  Flag 
0:01

Choose the option that best answers the question.

y is 25% of x
If xy ≠ 0, and 75% of x equals 125% of y,
which of the following is true?
y is 40% of x

y is 60% of x

y is 140% of x

y is 1662/3% of x

EndPractice
End Practice and
and See
See Results
Results Question 52 of 737 Submit Answer
Submit Answer Next Question
Math 
Calculator  Flag 
0:01

Consider each of the choices separately and select all that apply.

3
In a certain sequence of numbers, each
term after the first is found by adding 1 to
8
the previous term and then doubling this
sum.  If the third term is 18, which of the 10
following numbers are in the sequence?
16
Indicated all such numbers.

34

38

EndPractice
End Practice and
and See
See Results
Results Question 53 of 737 Answer
Next Question
Submit Answer
Submit

Math 
Calculator  Flag 
0:01

Enter the answer in the blank.

Set X={a, b, c}, where a < b < c. If the


average (arithmetic mean) of a and b is 3x
– 13, and the average of b and c is 3x + 11,
what is the range of set X?

EndPractice
End Practice and
and See
See Results
Results Question 54 of 737 Answer
Next Question
Submit Answer
Submit
Math 
Calculator  Flag 
0:01

Choose the correct statement.

The average (arithmetic mean) of 3, 3, 5, 6 The quantity in Column A is greater


and x is 2.
The quantity in Column B is greater

Column A Column B
The two quantities are equal
x −8
The relationship cannot be
determined from the information
given

EndPractice
End Practice and
and See
See Results
Results Question 55 of 737 Answer
Next Question
Submit Answer
Submit

Math 
Calculator  Flag 
0:01

Choose the option that best answers the question.

8/
a 3
​ +1
b
​ c
​ 4
b
9
In the expression above, a, b and c are /2
different numbers and each is one of the
numbers 2, 3 or 5. What is the greatest 5
possible value of the expression?
6

EndPractice
End Practice and
and See
See Results
Results Question 56 of 737 Answer
Next Question
Submit Answer
Submit
Math 
Calculator  Flag 
0:01

Choose the correct statement.

The quantity in Column A is greater

The quantity in Column B is greater

The two quantities are equal

The relationship cannot be


Column A Column B determined from the information
given
AB + AD DC + BC

EndPractice
End Practice and
and See
See Results
Results Question 57 of 737 Submit Answer
Submit Answer Next Question

Math 
Calculator  Flag 
0:01

Choose the option that best answers the question.

If gas costs $4/gallon, and one is going


to drive a compact car on a 200 mile
trip, what is the difference in fuel costs
required for this trip between the most
fuel efficient and least fuel efficient
compact car?

$64
$80
Note: mpg = miles per gallon
$160

$200

$800

EndPractice
End Practice and
and See
See Results
Results Question 58 of 737 Answer
Next Question
Submit Answer
Submit

Math 
Calculator  Flag 
0:02

Choose the option that best answers the question.

The range from the least fuel efficient


Station Wagon to the most fuel efficient
Station Wagon is what?

20 mpg

28 mpg

36 mpg

42 mpg
Note: mpg = miles per gallon

50 mpg
EndPractice
End Practice and
and See
See Results
Results Question 59 of 737 Submit Answer
Submit Answer Next Question

Math 
Calculator  Flag 
0:02

Enter the answer in the blank.

In how many different categories is


it possible to select a vehicle with a fuel
efficiency of 28 mpg?

Note: mpg = miles per gallon

EndPractice
End Practice and
and See
See Results
Results Question 60 of 737 Submit Answer
Submit Answer Next Question
Math 
Calculator  Flag 
0:02

Choose the option that best answers the question.

36
If the average (arithmetic mean) of 24
consecutive odd integers is 48, what is the
47
median of the 24 numbers?
48

49

72

EndPractice
End Practice and
and See
See Results
Results Question 61 of 737 Submit Answer
Submit Answer Next Question

Math 
Calculator  Flag 
0:01

Choose the correct statement.

The quantity in Column A is greater


Column A Column B
The quantity in Column B is greater
Product of the even Product of the odd
integers from –12 integers from –5 to The two quantities are equal
to 6 inclusive. 13 inclusive.
The relationship cannot be
determined from the information
given
EndPractice
Practice and
and See
See Results Question 62 of 737 Answer
Next Question
Submit Answer
End Results Submit

Math 
Calculator  Flag 
0:01

Choose the option that best answers the question.

6
For all numbers x and y, the operation ϕ is
defined by 
12
x ϕ y = (x + y)(x – y) + (y – x)(y + x) + xy. 
​ ​ 18
What is the value of  √​12 ϕ  √​3 ?
24

36

EndPractice
End Practice and
and See
See Results
Results Question 63 of 737 Answer
Next Question
Submit Answer
Submit

Math 
Calculator  Flag 
0:01

Consider each of the choices separately and select all that apply.

0.1
A weighted coin has a probability p of
showing heads. If successive flips are
0.2
independent, and the probability of getting
at least one head in two flips is greater than 0.3
0.5, then what could p be? 
0.4
Indicate all possible values.

0.6
0.7

EndPractice
End Practice and
and See
See Results
Results Question 64 of 737 Submit Answer
Submit Answer Next Question

Math 
Calculator  Flag 
0:01

Enter the answer in the blank.

x is a positive integer less than 100. When


x is divided by 5, the remainder is 4, and
when x is divided by 23, the remainder is 7.
What is the value of x?

EndPractice
End Practice and
and See
See Results
Results Question 65 of 737 Submit Answer
Submit Answer Next Question

Math 
Calculator  Flag 
0:01

Choose the correct statement.

The quantity in Column A is greater


The probability that event A will occur is 0.5

The probability that event B will occur is 0.4 The quantity in Column B is greater

The probability that event A or B will occur The two quantities are equal
is 0.8
The relationship cannot be
Column A Column determined from the information
B given
Probability that A and B both 0.1
occur

EndPractice
End Practice and
and See
See Results
Results Question 66 of 737 Answer
Next Question
Submit Answer
Submit

Math 
Calculator  Flag 
0:01

Choose the option that best answers the question.

-4
Which of the following is a root of the
equation 2x2 - 20x = 48?
2

12

EndPractice
End Practice and
and See
See Results
Results Question 67 of 737 Answer
Next Question
Submit Answer
Submit

Math 
Calculator  Flag 
0:01

Choose the correct statement.

The quantity in Column A is greater

The quantity in Column B is greater


The two quantities are equal

The relationship cannot be


Column A Column B determined from the information
given
x y

EndPractice
End Practice and
and See
See Results
Results Question 68 of 737 Submit Answer
Submit Answer Next Question

Math 
Calculator  Flag 
0:02

Choose the option that best answers the question.

x + 11 -11
If x + 2 =  ​ , then x2 + 4x - 5 =
x+3
-5

EndPractice
End Practice and
and See
See Results
Results Question 69 of 737 Submit Answer
Submit Answer Next Question

Math 
Calculator  Flag 
0:04
Choose the correct statement.

The quantity in Column A is greater


x, y, and z are three numbers, and 2x = y

The quantity in Column B is greater


Column Column B
A
The two quantities are equal
  x + z​ the average (arithmetic
3 The relationship cannot be
mean)
determined from the information
of the three numbers
given

EndPractice
End Practice and
and See
See Results
Results Question 70 of 737 Submit Answer
Submit Answer Next Question

Math 
Calculator  Flag 
0:01

Choose the option that best answers the question.

840
If K is the least positive integer that is
divisible by every integer from 1 to 8
2,520
inclusive, then K = 
6,720

20,160

40,320

EndPractice
End Practice and
and See
See Results
Results Question 71 of 737 Submit Answer
Submit Answer Next Question
Math 
Calculator  Flag 
0:01

Consider each of the choices separately and select all that apply.

0.5
The sides of a triangle are 1, x, and x2.
What are possible values of x?
1
Indicate all possible values.
1.5

2.5

3.5

EndPractice
End Practice and
and See
See Results
Results Question 72 of 737 Submit Answer
Submit Answer Next Question

Math 
Calculator  Flag 
0:03

Enter the answer in the blank.

If p and q are the solutions to x2 + 13x =


48, then what is the value of (p − q)2?  

EndPractice
End Practice and
and See
See Results
Results Question 73 of 737 Submit Answer
Submit Answer Next Question
Math 
Calculator  Flag 
0:01

Choose the correct statement.

The quantity in Column A is greater

The quantity in Column B is greater

The two quantities are equal

The relationship cannot be


determined from the information
AB = 12, AC = 10, AD = 18
given
Note: the region above is circular

Column A Column B

x y

EndPractice
End Practice and
and See
See Results
Results Question 74 of 737 Answer
Next Question
Submit Answer
Submit

Math 
Calculator  Flag 
0:01

Choose the option that best answers the question.

What is the average (arithmetic mean)


number of televisions per household?

Cannot be determined

2.1

2.3
2.5

2.7

EndPractice
End Practice and
and See
See Results
Results Question 75 of 737 Answer
Next Question
Submit Answer
Submit

Math 
Calculator  Flag 
0:02

Choose the option that best answers the question.

If k is the number of households with


exactly 2 televisions, then the number
of households with exactly 4 televisions
is

k−4

2k

k
​ −4
100
5k

6
500k

6
EndPractice
End Practice and
and See
See Results
Results Question 76 of 737 Submit Answer
Submit Answer Next Question

Math 
Calculator  Flag 
0:02

Choose the option that best answers the question.

What is the median number of


televisions per household?

Cannot be determined

2.5

EndPractice
End Practice and
and See
See Results
Results Question 77 of 737 Submit Answer
Submit Answer Next Question

Math 
Calculator  Flag 
0:01

Choose the option that best answers the question.

7 + 4k
Seven years ago Bob was k times as old
as Ann. If Ann is now 11 years old, what is
11/k + 7
Bob’s present age in terms of k?
11 – 7/k

11 – k/7

4k – 7

EndPractice
End Practice and
and See
See Results
Results Question 78 of 737 Submit Answer
Submit Answer Next Question

Math 
Calculator  Flag 
0:03

Choose the correct statement.

The quantity in Column A is greater


In this diagram, the circle is inscribed in the
square.
The quantity in Column B is greater

The two quantities are equal

The relationship cannot be


determined from the information
given

Column A Column B

5r
Length of diagonal AC /2

EndPractice
End Practice and
and See
See Results
Results Question 79 of 737 Submit Answer
Submit Answer Next Question


Calculator  Flag 
0:01
Math

Choose the option that best answers the question.

$1.20
At Demeter Deals, the cost of 3 apples and
4 bananas is $4.23, and the cost of 5
$1.30
apples and 7 bananas is $7.16.  At these
prices, what would be the cost of 1 apple $1.40
and 1 banana?
$1.50

$1.60

EndPractice
End Practice and
and See
See Results
Results Question 80 of 737 Submit Answer
Submit Answer Next Question
Math 
Calculator  Flag 
0:08

Choose the correct statement.

The quantity in Column A is greater


Column A Column B
The quantity in Column B is greater
The product of The product of
integers from −87 integers from −58 The two quantities are equal
to −36 inclusive to −34 inclusive
The relationship cannot be
determined from the information
given

EndPractice
End Practice and
and See
See Results
Results Question 81 of 737 Submit Answer
Submit Answer Next Question

Math 
Calculator  Flag 
0:02

Choose the option that best answers the question.

0.00001
If 100,000 microns = 1 decimeter, and
1,000,000,000 angstroms = 1 decimeter,
0.0001
how many angstroms equal 1 micron?
0.001

10,000

100,000
EndPractice
Practice and
and See
See Results Question 82 of 737 Answer
Next Question
Submit Answer
End Results Submit

Math 
Calculator  Flag 
0:01

Consider each of the choices separately and select all that apply.

N N
The maximum number of meals an event ​ +​
3 12
organizer can order is N.  On a particular
N N 7N
Friday, he needs to have ​ meals at one ​ +​
4 5 30
N
event and ​ meals available at another N 4N
6 ​ +​
event.  Which of the following expressions 6 15
represent the total number of meals he N 7N
would have to have ready for both of these ​ +​
8 24
events?
N 13N
​ +​
Indicate all such expressions. 18 36

EndPractice
End Practice and
and See
See Results
Results Question 83 of 737 Answer
Next Question
Submit Answer
Submit

Math 
Calculator  Flag 
0:01

Enter the answer in the blank.


​​p2​ √​p​ 5​
For a positive number p, ​ −2 = pn​ .  What
​p ​
is the value of n?
EndPractice
End Practice and
and See
See Results
Results Question 84 of 737 Answer
Next Question
Submit Answer
Submit

Math 
Calculator  Flag 
0:01

Choose the correct statement.

x and y are prime numbers and x + y = 18 The quantity in Column A is greater

The quantity in Column B is greater


Column A Column B

xy 70 The two quantities are equal

The relationship cannot be


determined from the information
given

EndPractice
End Practice and
and See
See Results
Results Question 85 of 737 Answer
Next Question
Submit Answer
Submit

Math 
Calculator  Flag 
0:01

Choose the option that best answers the question.

1
If x is a positive integer and x+2 is divisible
by 10, what is the remainder when x2+4x+9
3
is divided by 10?
5

9
EndPractice
End Practice and
and See
See Results
Results Question 86 of 737 Answer
Next Question
Submit Answer
Submit

Math 
Calculator  Flag 
0:08

Choose the correct statement.

The quantity in Column A is greater

The quantity in Column B is greater

The two quantities are equal

Column A Column B The relationship cannot be


determined from the information
Area of quadrilateral 56 given

EndPractice
End Practice and
and See
See Results
Results Question 87 of 737 Answer
Next Question
Submit Answer
Submit

Math 
Calculator  Flag 
0:01

Choose the option that best answers the question.

From 2010 to 2015, revenue at QN


The following graph shows the total Corporation increased by
revenues and total costs of QN approximately what percent?
Corporation, a publicly traded company,
over the span 2007-2015.  Recall that, for 18%
any year, profit = (revenue) – (cost). 
47%

63%

84%

126%

EndPractice
End Practice and
and See
See Results
Results Question 88 of 737 Submit Answer
Submit Answer Next Question

Math 
Calculator  Flag 
0:01

Choose the option that best answers the question.

In 2013, the dollar amount of the profit


The following graph shows the total at QN Corporation was approximately
revenues and total costs of QN which of the following? 
Corporation, a publicly traded company,
over the span 2007-2015.  Recall that, for $80 million
any year, profit = (revenue) – (cost). 
$160 million

$190 million

$240 million

$350 million

EndPractice
End Practice and
and See
See Results
Results Question 89 of 737 Submit Answer
Submit Answer Next Question
Math 
Calculator  Flag 
0:01

Choose the option that best answers the question.

Given that profit in 2009 was $30M,


The following graph shows the total then from 2009 to 2010, profit at QN
revenues and total costs of QN Corporation increased by
Corporation, a publicly traded company, approximately what percent?
over the span 2007-2015.  Recall that, for
any year, profit = (revenue) – (cost).  81%

165%

300%

433%

533%

EndPractice
End Practice and
and See
See Results
Results Question 90 of 737 Answer
Next Question
Submit Answer
Submit

Math 
Calculator  Flag 
0:01

Choose the option that best answers the question.

1
If xy ≠ 0 and 2x + 3y is equal to 175% of ​
x 16
8x, then  ​ =
y 3

20
3

16
1

4
3

8

EndPractice
End Practice and
and See
See Results
Results Question 91 of 737 Answer
Next Question
Submit Answer
Submit

Math 
Calculator  Flag 
0:01

Choose the correct statement.

A very large fruit bowl contains 160 pieces The quantity in Column A is greater
of fruit, all apples and oranges. The ratio of
The quantity in Column B is greater
apples to oranges in this bowl is 3 to 5.
The two quantities are equal
Column A Column
B The relationship cannot be
determined from the information
the number of apples in this 66
given
fruit bowl

EndPractice
End Practice and
and See
See Results
Results Question 92 of 737 Answer
Next Question
Submit Answer
Submit

Math 
Calculator  Flag 
0:02

Choose the option that best answers the question.


10
1/3
of x% of 50 is equal to 5% of 200. What
is the value of x?
30

60

80

120

EndPractice
End Practice and
and See
See Results
Results Question 93 of 737 Submit Answer
Submit Answer Next Question

Math 
Calculator  Flag 
0:01

Consider each of the choices separately and select all that apply.

3.4
Which of the following, when rounded to
the nearest integer, are rounded to 3?
3.445

3.494

3.499

3.501

3.61

EndPractice
End Practice and
and See
See Results
Results Question 94 of 737 Submit Answer
Submit Answer Next Question

Math 
Calculator  Flag 
0:01
Enter the answer as a fraction. Fractions do not need to be in simplest form

In the diagram above, the shaded


region is
what fraction of the area of the square? 

EndPractice
End Practice and
and See
See Results
Results Question 95 of 737 Submit Answer
Submit Answer Next Question

Math 
Calculator  Flag 
0:01

Choose the correct statement.

24 The quantity in Column A is greater


The fraction ​ , when written in lowest
36  
a
terms, is ​  , where a and b are positive The quantity in Column B is greater
b
numbers.
The two quantities are equal
Column A Column B
The relationship cannot be
a+b 6 determined from the information
given

EndPractice
End Practice and
and See
See Results
Results Question 96 of 737 Submit Answer
Submit Answer Next Question
Math 
Calculator  Flag 
0:02

Choose the option that best answers the question.

R
In Ophiuchus Corporation, 60% of the total ​
2
revenue R is devoted to the advertising
budget. Five-sixths of this advertising R

budget was spent on television advertising. 3
Which of the following represents the dollar 2R

amount spent on television advertising? 3
2R

5
4R

5

EndPractice
End Practice and
and See
See Results
Results Question 97 of 737 Submit Answer
Submit Answer Next Question

Math 
Calculator  Flag 
0:01

Choose the correct statement.

The quantity in Column A is greater


3 < |y| < 7

The quantity in Column B is greater


Column A Column B

The two quantities are equal


y2 + 5 50

The relationship cannot be


determined from the information
given
EndPractice
End Practice and
and See
See Results
Results Question 98 of 737 Submit Answer
Submit Answer Next Question

Math 
Calculator  Flag 
0:01

Choose the option that best answers the question.

–12
If 2x – 3y = 6, then 6y – 4x =

–6

12

Cannot be determined

EndPractice
End Practice and
and See
See Results
Results Question 99 of 737 Submit Answer
Submit Answer Next Question

Math 
Calculator  Flag 
0:01

Choose the correct statement.

The quantity in Column A is greater


Column A Column B
The quantity in Column B is greater
Number of primes Number of primes
between 50 and 60 between 80 and 90 The two quantities are equal

The relationship cannot be


determined from the information
given

EndPractice
End Practice and
and See
See Results
Results Question 100 of 737 Submit Answer
Submit Answer Next Question
Math 
Calculator  Flag 
0:02

Choose the option that best answers the question.

5
In the xy coordinate system, line k passes y = − ​ x + 2m
2
through points (–5m, 0) and (0, 2m). Which
of the following is a possible equation of 2
y = ​ x − 5m
5
line k?
5
y = ​ x + 2m
2
2
y = ​ x + 2m
5
2
y = − ​ x − 5m
5

EndPractice
End Practice and
and See
See Results
Results Question 101 of 737 Submit Answer
Submit Answer Next Question

Math 
Calculator  Flag 
0:02

Consider each of the choices separately and select all that apply.

57
If J is a positive odd integer less than 10,
and K is an even integer such that 50 ≤ K ≤
80
100, then which of the following could be
the value of J2 + K?    85
Indicated all such numbers.
99

100
159 
162

195

EndPractice
End Practice and
and See
See Results
Results Question 102 of 737 Answer
Next Question
Submit Answer
Submit

Math 
Calculator  Flag 
0:04

Enter the answer in the blank.

A right
triangle has legs of 6 and x,
and a
hypotenuse of r. If 5r = 5x + 9, what is the
value of r + x?

EndPractice
End Practice and
and See
See Results
Results Question 103 of 737 Answer
Next Question
Submit Answer
Submit

Math 
Calculator  Flag 
0:03

Choose the correct statement.


The quantity in Column A is greater
Column A Column B
The quantity in Column B is greater
0.05 percent of 4000 1/200 of 4000
The two quantities are equal

The relationship cannot be


determined from the information
given

EndPractice
End Practice and
and See
See Results
Results Question 104 of 737 Answer
Next Question
Submit Answer
Submit

Math 
Calculator  Flag 
0:01

Choose the option that best answers the question.

Which of the following gives the range


In a survey, 82 high school students were of the median number of hours of TV
randomly selected and asked how many watched last week?
hours of television they had watched in the
previous week. The histogram below 1-5
displays their answers.
6-10

11-15

16-20

41-45

EndPractice
End Practice and
and See
See Results
Results Question 105 of 737 Answer
Next Question
Submit Answer
Submit
Math 
Calculator  Flag 
0:02

Choose the option that best answers the question.

Which of the following could be the


In a survey, 82 high school students were third quartile value for number of hours
randomly selected and asked how many of TV watched last week?
hours of television they had watched in the
previous week. The histogram below 11
displays their answers.
17

21

23

26

EndPractice
End Practice and
and See
See Results
Results Question 106 of 737 Submit Answer
Submit Answer Next Question

Math 
Calculator  Flag 
0:07

Choose the option that best answers the question.

Suppose all students surveyed


In a survey, 82 high school students were answered in integer number of hours
randomly selected and asked how many only. Suppose, of 82 surveyed, only
hours of television they had watched in the one respondent answered "16 hours."
previous week. The histogram below Within this group, the approximate
displays their answers. percentile of this person would be:

32nd percentile
51st percentile

67th percentile

75th percentile

80th percentile

EndPractice
End Practice and
and See
See Results
Results Question 107 of 737 Answer
Next Question
Submit Answer
Submit

Math 
Calculator  Flag 
0:01

Choose the option that best answers the question.


k is a positive number. If k is twice its 2√​2
reciprocal, and j is twice k, then jk =
4

4√​2

8

8√​2

EndPractice
End Practice and
and See
See Results
Results Question 108 of 737 Answer
Next Question
Submit Answer
Submit

Math 
Calculator  Flag 
0:03

Choose the correct statement.


In Blattodea Hotel, the ratio of single-bed The quantity in Column A is greater
rooms to double-bed rooms is 4 to 11.
The quantity in Column B is greater
In Fremont Hotel, the ratio of single-bed
rooms to double-bed rooms is 3 to 8. The two quantities are equal

Column A Column B The relationship cannot be


determined from the information
The number of The number of given
single-bed rooms in single-bed rooms
Blattodea Hotel in Fremont Hotel

EndPractice
End Practice and
and See
See Results
Results Question 109 of 737 Submit Answer
Submit Answer Next Question

Math 
Calculator  Flag 
0:01

Choose the option that best answers the question.

19
If the population of Townville went from
2105 to 1705, then the percent decrease in
21
population is closest to
22

23

24

EndPractice
End Practice and
and See
See Results
Results Question 110 of 737 Submit Answer
Submit Answer Next Question
Math 
Calculator  Flag 
0:01

Choose the correct statement.

The quantity in Column A is greater

The quantity in Column B is greater

The two quantities are equal

Column A Column B The relationship cannot be


determined from the information
180 + a b+c given

EndPractice
End Practice and
and See
See Results
Results Question 111 of 737 Submit Answer
Submit Answer Next Question

Math 
Calculator  Flag 
0:01

Choose the option that best answers the question.

−3
Line k is in the rectangular coordinate
system. If line k is defined by the equation
−2
3y = 2x + 6, and line k intersects the x-axis
at point (a,b), then what is the value of a? 0

End Practice and See Results Question 112 of 737 Submit Answer Next Question
End Practice and See Results Submit Answer

Math 
Calculator  Flag 
0:01

Consider each of the choices separately and select all that apply.

211
Which of the following are divisors of 
1.2 × 1010
​ ?
75

510

18

39

36

EndPractice
End Practice and
and See
See Results
Results Question 113 of 737 Answer
Next Question
Submit Answer
Submit

Math 
Calculator  Flag 
0:01

Enter the answer in the blank.

In a certain sequence of all


positive terms,

{a1, a2, a3, …}


each term equals the
previous term times a constant factor. If
(a1)(a5) =
900, what is the value of a3? 
EndPractice
End Practice and
and See
See Results
Results Question 114 of 737 Submit Answer
Submit Answer Next Question

Math 
Calculator  Flag 
0:09

Choose the correct statement.

At 9:00 am in a large florist shop, there was The quantity in Column A is greater
an equal number of roses and carnations.
The quantity in Column B is greater
 That morning, there was only one
purchase, the purchase of 18 roses. After The two quantities are equal
this purchase, the ratio of roses to
carnations was 3 to 4.   The relationship cannot be
determined from the information
Column A Column given
B

The number of roses in the 72


florist shop after this purchase.

EndPractice
End Practice and
and See
See Results
Results Question 115 of 737 Submit Answer
Submit Answer Next Question

Math 
Calculator  Flag 
0:01

Choose the option that best answers the question.

3x4y5
What is the Greatest Common Factor
(GCF) of 18x8y20 and 24x12y15?
6x4y5
3x8y15

6x8y15

72x12y20

EndPractice
End Practice and
and See
See Results
Results Question 116 of 737 Submit Answer
Submit Answer Next Question

Math 
Calculator  Flag 
0:01

Choose the correct statement.

Main course: Chicken, Beef, Tofu The quantity in Column A is greater

Side dish: Rice, Salad, Soup, Pasta The quantity in Column B is greater

Dessert: Pie, Cake The two quantities are equal


A meal at a certain restaurant consists of 1
The relationship cannot be
main course, 2 different side dishes and 1
determined from the information
dessert.
given
Column A Column
B

Number of different meals 36


possible

EndPractice
End Practice and
and See
See Results
Results Question 117 of 737 Submit Answer
Submit Answer Next Question
Math 
Calculator  Flag 
0:02

Choose the option that best answers the question.

Company C was responsible for


approximately what percent of total
sales in the sector in 2007?

24%

40%

55%

The graph above shows the total sales, in 70%


millions of dollars, for three companies, A &
B & C, in a particular sector for the years 93%
1998 through 2007 inclusive. Assume
these companies are the only three
companies active in this particular
sector.  Companies A & B existed since the
1980s, although only data from 1998 is
shown. Company C's first year in existence
was 2000.

EndPractice
End Practice and
and See
See Results
Results Question 118 of 737 Submit Answer
Submit Answer Next Question

Math 
Calculator  Flag 
0:01

Choose the option that best answers the question.

5/
If the sum of two numbers is 6, and the 24

sum of their reciprocals is 15/8, what is the 5/


16
product of the two numbers?
16/
5

25/
4

45/
4

EndPractice
End Practice and
and See
See Results
Results Question 119 of 737 Answer
Next Question
Submit Answer
Submit

Math 
Calculator  Flag 
0:02

Choose the correct statement.

K = sum of the integers from 1 to 500 The quantity in Column A is greater


inclusive that are divisible by 5.
The quantity in Column B is greater

Column A Column B
The two quantities are equal
K 25,000
The relationship cannot be
determined from the information
given

EndPractice
End Practice and
and See
See Results
Results Question 120 of 737 Answer
Next Question
Submit Answer
Submit
Math 
Calculator  Flag 
0:01

Choose the option that best answers the question.

1
When positive integer k is divided by 5, the
remainder is 2. When k is divided by 6, the
2
remainder is 5. If k is less than 40, what is
the remainder when k is divided by 7? 3

EndPractice
End Practice and
and See
See Results
Results Question 121 of 737 Submit Answer
Submit Answer Next Question

Math 
Calculator  Flag 
0:01

Consider each of the choices separately and select all that apply.

−10
What are the x-intercepts of the parabola
defined by the equation y = 2x2 − 8x − 90?
−9
Indicate all x-intercepts.
−5

−4

5
9 
10

EndPractice
End Practice and
and See
See Results
Results Question 122 of 737 Submit Answer
Submit Answer Next Question

Math 
Calculator  Flag 
0:01

Enter the answer in the blank.

In the standard x,y plane, a circle


has a
radius 6 and center (7, 3). The
circle
intersects the x-axis at (a, 0) and (b, 0).
What is the value of a + b?

EndPractice
End Practice and
and See
See Results
Results Question 123 of 737 Submit Answer
Submit Answer Next Question

Math 
Calculator  Flag 
0:02

Choose the correct statement.

The quantity in Column A is greater

The quantity in Column B is greater

The two quantities are equal


Column A Column B
The relationship cannot be
x 90 determined from the information
given

EndPractice
End Practice and
and See
See Results
Results Question 124 of 737 Submit Answer
Submit Answer Next Question

Math 
Calculator  Flag 
0:01

Choose the option that best answers the question.

Frozen prepared meals constitute what


The following pie chart shows the percentage of the total sales for the first
breakdown of sales for a particular grocery quarter last year?
store over the first quarter of last year.  The
bar chart shows the detail of breakdown for 2.4%
frozen foods.
8.5%

20%

36%

54%
EndPractice
End Practice and
and See
See Results
Results Question 125 of 737 Answer
Next Question
Submit Answer
Submit

Math 
Calculator  Flag 
0:01

Choose the option that best answers the question.

What is the dollar amount of sales of


The following pie chart shows the canned goods in the first quarter of
breakdown of sales for a particular grocery last year?
store over the first quarter of last year.  The
bar chart shows the detail of breakdown for $6,000
frozen foods.
$9,000

$18,000

$36,000

$90,000
EndPractice
End Practice and
and See
See Results
Results Question 126 of 737 Submit Answer
Submit Answer Next Question

Math 
Calculator  Flag 
0:08

Choose the option that best answers the question.

During the first quarter of last year, this


The following pie chart shows the particular grocery store was finishing its
breakdown of sales for a particular grocery construction of an expanded bakery
store over the first quarter of last year.  The facility, which, when opened at the
bar chart shows the detail of breakdown for beginning of the second quarter, will
frozen foods. offer dozens of new cakes and pies, a
whole new line of pastries, and several
flavors of gourmet coffee.  Assume that
in the second quarter, the bakery sales
triple, and all other sales stay the
same.  Bakery would then account for
what percentage of total sales in the
second quarter?

12%

14.3%

16.1%
18%

25.3%

EndPractice
End Practice and
and See
See Results
Results Question 127 of 737 Submit Answer
Submit Answer Next Question

Math 
Calculator  Flag 
0:08

Choose the option that best answers the question.

1/
If four numbers are randomly selected 256

without replacement from set {1, 2, 3, 4}, 1/


64
what is the probability that the four
numbers are selected in ascending order? 1/
48
1/
24

1/
12

EndPractice
End Practice and
and See
See Results
Results Question 128 of 737 Submit Answer
Submit Answer Next Question

Math 
Calculator  Flag 
0:01

Choose the correct statement.

The area of a circle is equal to the area of a The quantity in Column A is greater
square.
The quantity in Column B is greater

Column A Column B
The two quantities are equal
The circumference The perimeter of
The relationship cannot be
of the circle. the square.
determined from the information
given

EndPractice
End Practice and
and See
See Results
Results Question 129 of 737 Submit Answer
Submit Answer Next Question

Math 
Calculator  Flag 
0:01

Choose the option that best answers the question.

​ ​ 72
If p = 6 + 3 √​2 and q = 6 − 3√​2 , then p2 +
2pq + q2 = ​
72 − 6√​2
120 − 6√​2

144

144 − 6√​2

EndPractice
End Practice and
and See
See Results
Results Question 130 of 737 Answer
Next Question
Submit Answer
Submit

Math 
Calculator  Flag 
0:01

Choose the correct statement.

The quantity in Column A is greater


Column A Column B
The quantity in Column B is greater
Average (arithmetic Average
mean) of integers (arithmetic mean) The two quantities are equal
from −50 to −1 of integers from
inclusive. −50 to 0 inclusive. The relationship cannot be
determined from the information
given

EndPractice
End Practice and
and See
See Results
Results Question 131 of 737 Answer
Next Question
Submit Answer
Submit

Math 
Calculator  Flag 
0:01

Choose the option that best answers the question.

3
Today, Bill is thirteen times as old as Pete.
In nine years, Bill will be four times as old
4
as Pete. How old will Pete be 2 years from
today? 5

EndPractice
End Practice and
and See
See Results
Results Question 132 of 737 Submit Answer
Submit Answer Next Question

Math 
Calculator  Flag 
0:01

Choose the option that best answers the question.

21
A
necklace is made up of three different
colored beads: red, blue, and green. If
the
34
ratio of blue to red beads is 1:3 and red to
green beads is 2:3, what is
the lowest 42
number of beads that could be on the
necklace if the total number of
beads on 51
the necklace is greater than forty?
54

EndPractice
End Practice and
and See
See Results
Results Question 133 of 737 Submit Answer
Submit Answer Next Question

Math 
Calculator  Flag 
0:01

Enter the answer in the blank.


Rectangle ABCD has all four
vertices on
the circle. If AD = 10, DE =
32, and CE =
40, and if the area of the circle is of the
form Kπ ,
then what is the value of K? 

EndPractice
End Practice and
and See
See Results
Results Question 134 of 737 Submit Answer
Submit Answer Next Question

Math 
Calculator  Flag 
0:03

Choose the correct statement.

The quantity in Column A is greater


Column A Column B
The quantity in Column B is greater
43 percent of 63 63 percent of 43
The two quantities are equal

The relationship cannot be


determined from the information
given

EndPractice
End Practice and
and See
See Results
Results Question 135 of 737 Submit Answer
Submit Answer Next Question
Math 
Calculator  Flag 
0:01

Choose the option that best answers the question.

$215
After receiving a 25% discount, Sue paid
$180 for a lawnmower. What is the original
$220
price of the lawnmower before the
discount? $225

$240

$245

EndPractice
End Practice and
and See
See Results
Results Question 136 of 737 Submit Answer
Submit Answer Next Question

Math 
Calculator  Flag 
0:03

Choose the correct statement.

The quantity in Column A is greater

The quantity in Column B is greater

The two quantities are equal

The relationship cannot be


determined from the information
Column A Column B given

x y
EndPractice
End Practice and
and See
See Results
Results Question 137 of 737 Answer
Next Question
Submit Answer
Submit

Math 
Calculator  Flag 
0:01

Choose the option that best answers the question.

If there are 44 leopards at the zoo,


what is the zoo’s total animal
population?

225

275

325

350

375

EndPractice
End Practice and
and See
See Results
Results Question 138 of 737 Answer
Next Question
Submit Answer
Submit

Math 
Calculator  Flag 
0:02

Choose the option that best answers the question.

2
1 ​
​ 5
1 + 1−1​ 1​ 4
3 ​
7
2

3
3

4
5

6

EndPractice
End Practice and
and See
See Results
Results Question 139 of 737 Submit Answer
Submit Answer Next Question

Math 
Calculator  Flag 
0:03

Choose the correct statement.

On a certain table is a collection of 45 pens The quantity in Column A is greater


of various colors of ink and various
The quantity in Column B is greater
designs.  Of these 45 pens, 60% are
ballpoint pens. Also, 60% of the pens on The two quantities are equal
the table have blue ink.  
The relationship cannot be
Column A Column determined from the information
B given

The number of pens on the 8


table that are ballpoint with
blue ink.

EndPractice
End Practice and
and See
See Results
Results Question 140 of 737 Submit Answer
Submit Answer Next Question
Math 
Calculator  Flag 
0:01

Choose the option that best answers the question.

1.4
The length and width of a rectangular yard
are 11 meters and 5 meters respectively. If
1.6
each dimension were reduced by x meters
to make the ratio of length to width 8 to 3, 1.8
what would be the value of x?
2.0

2.2

EndPractice
End Practice and
and See
See Results
Results Question 141 of 737 Submit Answer
Submit Answer Next Question

Math 
Calculator  Flag 
0:01

Consider each of the choices separately and select all that apply.

(1, 6)

(2, 7)

(4, 6)

(5, 4)

(6, 4)

The points (2, 1) and (5, 5) are shown in (7, 2)


the xy-plane above.  Circle C (not shown)
has its center at (2, 1) and passes through (8, 1) 
(5, 5).  Which of the following are
coordinate of points that are outside of
Circle C?  (NOTE: a point on the circle is
not outside the circle.)  

Indicated all such coordinates

EndPractice
End Practice and
and See
See Results
Results Question 142 of 737 Answer
Next Question
Submit Answer
Submit

Math 
Calculator  Flag 
0:02

Enter the answer as a fraction. Fractions do not need to be in simplest form

The triangle in a diagram is


equilateral. The
smaller circle is
tangent to all three sides of
the triangle. The larger circle passes
through all three vertices of the triangle.
What is the ratio of the area of the smaller
circle to the area of the larger circle?

EndPractice
End Practice and
and See
See Results
Results Question 143 of 737 Answer
Next Question
Submit Answer
Submit
Math 
Calculator  Flag 
0:01

Choose the correct statement.

When a coin is flipped, the probability of The quantity in Column A is greater


getting heads is 0.5, and the probability of
The quantity in Column B is greater
getting tails is 0.5

A coin is flipped 5 times The two quantities are equal

Column A Column B The relationship cannot be


determined from the information
Probability of Probability of given
getting exactly 2 getting exactly 3
heads heads

EndPractice
End Practice and
and See
See Results
Results Question 144 of 737 Submit Answer
Submit Answer Next Question

Math  Calculator  Flag 


0:01

Choose the option that best answers the question.

Continents by Area
Continents by
Population

Population density is the population of


a region divided by its geographic
area. Of the seven continents, Asia
has by far the largest population
density. Which continent has the
second largest?

Africa

North America

South America
Europe

Australia

EndPractice
End Practice and
and See
See Results
Results Question 145 of 737 Submit Answer
Submit Answer Next Question

Math 
Calculator  Flag 
0:01

Choose the option that best answers the question.

Continents by Area

Continents by
Population
What is the approximate population of
South America?

399,000,000

833,000,000

1,036,000,000

2,632,000,000

8,432,000,000

EndPractice
End Practice and
and See
See Results
Results Question 146 of 737 Answer
Next Question
Submit Answer
Submit

Math 
Calculator  Flag 
0:01

Choose the option that best answers the question.

Continents by Area
Continents by
Population

What is the approximate area of the


continent of Africa in square
kilometers?

10,200,000

30,450,000

43,950,000

738,916,000

1,421,000,000
EndPractice
End Practice and
and See
See Results
Results Question 147 of 737 Submit Answer
Submit Answer Next Question

Math 
Calculator  Flag 
0:01

Choose the option that best answers the question.

1 5
If 5x+y = 125 and 3x–3y =  ​ , then y =
−​
9 2
1

4
1

2
5

2
5

4

EndPractice
End Practice and
and See
See Results
Results Question 148 of 737 Submit Answer
Submit Answer Next Question

Math 
Calculator  Flag 
0:01

Choose the correct statement.

x and y are prime numbers. The quantity in Column A is greater

x + y = 18 The quantity in Column B is greater

Column A Column B The two quantities are equal

y 14 The relationship cannot be


determined from the information
given

EndPractice
End Practice and
and See
See Results
Results Question 149 of 737 Submit Answer
Submit Answer Next Question

Math 
Calculator  Flag 
0:02

Choose the option that best answers the question.

0.005
​ ​1
5√​2 percent of ​​ ​ = 0.02
​√​200 ​

0.05

0.2

0.5

EndPractice
End Practice and
and See
See Results
Results Question 150 of 737 Submit Answer
Submit Answer Next Question

Math 
Calculator  Flag 
0:01

Choose the correct statement.

Gerry is three times as old as Pat. The quantity in Column A is greater

The quantity in Column B is greater


Column A Column B

Gerry's age 20 years Pat's age in 12 The two quantities are equal
ago years
The relationship cannot be
determined from the information
given

EndPractice
End Practice and
and See
See Results
Results Question 151 of 737 Answer
Next Question
Submit Answer
Submit

Math 
Calculator  Flag 
0:01

Choose the option that best answers the question.

a-4
The average (arithmetic mean) of four
numbers is 4a + 16. When a fifth number is
a+4
added, the average becomes 5a + 20. The
fifth number is 4a + 4

4a + 16

9a + 36

EndPractice
End Practice and
and See
See Results
Results Question 152 of 737 Answer
Next Question
Submit Answer
Submit

Math 
Calculator  Flag 
0:01

Consider each of the choices separately and select all that apply.

3
For which values of x is 

(x2 − 10x + 25)(x2 − 13x + 40) < 10? 4

Indicate all possible values of x. 5

6
7

10

EndPractice
End Practice and
and See
See Results
Results Question 153 of 737 Submit Answer
Submit Answer Next Question

Math 
Calculator  Flag 
0:01

Enter the answer in the blank.

Albert was driving on a straight highway,


from Edenborough to Megiddo, at 60 mph.
Beatrice was driving faster than Albert, on
the same highway, in the same direction.
She started behind Albert. She passed
Albert at 1:20 pm. At 3:20 pm, Beatrice
arrived at Megiddo, and Albert arrived 50
minutes later, at 4:10 pm. Assume both
drivers kept up constant speeds without
stops. What was Beatrice's speed, in mph?

EndPractice
End Practice and
and See
See Results
Results Question 154 of 737 Submit Answer
Submit Answer Next Question

Math 
Calculator  Flag 
0:01

Choose the correct statement.


The quantity in Column A is greater

The quantity in Column B is greater

Column A Column B The two quantities are equal

x y The relationship cannot be


determined from the information
given

EndPractice
End Practice and
and See
See Results
Results Question 155 of 737 Submit Answer
Submit Answer Next Question

Math 
Calculator  Flag 
0:03

Choose the option that best answers the question.

4
46​ − 45​ /3
​ =
3
44/3

44 – 45/3

45 – 44

45

EndPractice
End Practice and
and See
See Results
Results Question 156 of 737 Submit Answer
Submit Answer Next Question

Math 
Calculator  Flag 
0:01

Choose the correct statement.


The quantity in Column A is greater

The quantity in Column B is greater

The two quantities are equal

Column A Column B The relationship cannot be


determined from the information
a+b c+d
given

EndPractice
End Practice and
and See
See Results
Results Question 157 of 737 Submit Answer
Submit Answer Next Question

Math 
Calculator  Flag 
0:01

Choose the option that best answers the question.

Suppose China's production remains


more or less constant from 2004 to
2006. Suppose India is able to sustain
the same percent increase in both of
those years. By what percent would
India's rice production have to increase
from 2004 to 2005 and again from 2005
to 2006 so that it equaled China in rice
production in 2006?
World's Total Rice Production in 2004 =
10.0%
368,080,000 metric tons
12.8%

16.3%

21.5%

27.4%
EndPractice
End Practice and
and See
See Results
Results Question 158 of 737 Answer
Next Question
Submit Answer
Submit

Math 
Calculator  Flag 
0:02

Choose the option that best answers the question.

What was the approximate rice


production of Vietnam in 2004?

8,400,000

22,400,000

43,700,000

80,000,000
World's Total Rice Production in 2004 =
103,060,000
368,080,000 metric tons
EndPractice
End Practice and
and See
See Results
Results Question 159 of 737 Submit Answer
Submit Answer Next Question

Math 
Calculator  Flag 
0:01

Choose the option that best answers the question.

Bangladesh accounted for what percent


of world rice production in 2004?

1.4%

7.2%

11.6%

33%
World's Total Rice Production in 2004 =
47.6%
368,080,000 metric tons

EndPractice
End Practice and
and See
See Results
Results Question 160 of 737 Submit Answer
Submit Answer Next Question
Math 
Calculator  Flag 
0:02

Choose the option that best answers the question.

–1
If 0.25 + x = y and y/x = 0.2, then y =

–1/2

–1/4

–1/8

–1/16

EndPractice
End Practice and
and See
See Results
Results Question 161 of 737 Submit Answer
Submit Answer Next Question

Math 
Calculator  Flag 
0:08

Choose the correct statement.

N equals the number of positive 3-digit The quantity in Column A is greater


numbers that contain odd digits only.
The quantity in Column B is greater

Column A Column B
The two quantities are equal
N 125
The relationship cannot be
determined from the information
given
EndPractice
Practice and
and See
See Results Question 162 of 737 Answer
Next Question
Submit Answer
End Results Submit

Math 
Calculator  Flag 
0:01

Choose the option that best answers the question.

0
If x and y are both positive and 

√​x2​ + y2​ = 3x − y , then x/y =
1/4

1/2

3/4

3/2

EndPractice
End Practice and
and See
See Results
Results Question 163 of 737 Answer
Next Question
Submit Answer
Submit

Math 
Calculator  Flag 
0:01

Choose the option that best answers the question.

625
What is the sum of all the multiples of 5
that are greater than 100 and less than
1250
200? 
1875

2850

3125
EndPractice
End Practice and
and See
See Results
Results Question 164 of 737 Submit Answer
Submit Answer Next Question

Math 
Calculator  Flag 
0:07

Enter the answer in the blank.

Ten years ago, Josh was three times


as old
as Tim. In five years, Josh will
be 10 years
more than twice as old as Tim. How old is
Tim right now? 

EndPractice
End Practice and
and See
See Results
Results Question 165 of 737 Submit Answer
Submit Answer Next Question

Math 
Calculator  Flag 
0:01

Choose the correct statement.

The population of bacteria doubles every The quantity in Column A is greater


30 minutes.
The quantity in Column B is greater
At 3:30 pm on Monday, the population was
240. The two quantities are equal

Column A Column The relationship cannot be


B determined from the information
given
The bacteria population at 2:00 40
pm on Monday
EndPractice
End Practice and
and See
See Results
Results Question 166 of 737 Answer
Next Question
Submit Answer
Submit

Math 
Calculator  Flag 
0:01

Choose the option that best answers the question.

​ 2
If x − 5 = √ ​2x2​ − 18x + 37  then x could equal

EndPractice
End Practice and
and See
See Results
Results Question 167 of 737 Answer
Next Question
Submit Answer
Submit

Math 
Calculator  Flag 
0:02

Choose the correct statement.

Townville has a population of 1213. The quantity in Column A is greater

296 people in Townville speak Spanish. The quantity in Column B is greater

Column A Column The two quantities are equal


B
The relationship cannot be
Percentage of people in 25 determined from the information
Townville who speak Spanish given

EndPractice
End Practice and
and See
See Results
Results Question 168 of 737 Answer
Next Question
Submit Answer
Submit

Math 
Calculator  Flag 
0:01

Choose the option that best answers the question.

$135
On Monday, the regular price of a widget
was discounted by 25 percent. On
$150
Tuesday, the reduced price was discounted
by 50 percent. If the final price was $60, $160
what was the regular price?
$175

$180

EndPractice
End Practice and
and See
See Results
Results Question 169 of 737 Answer
Next Question
Submit Answer
Submit

Math 
Calculator  Flag 
0:01

Choose the correct statement.

The quantity in Column A is greater


Circle A has radius x
Circle B has diameter 2x The quantity in Column B is greater

Column A Column B The two quantities are equal

The circumference The circumference The relationship cannot be


to diameter ratio of to diameter ratio of determined from the information
circle A circle B given

EndPractice
End Practice and
and See
See Results
Results Question 170 of 737 Submit Answer
Submit Answer Next Question

Math 
Calculator  Flag 
0:02

Choose the option that best answers the question.

173.6
To create paint with a certain shade of
gray, one must combine 2.016 liters of
176.4
black paint with every one liter of white
paint. Approximately how many liters of 347.1
white paint must be combined with 350
liters of black paint to create the certain 694.4
shade of gray?
705.6

EndPractice
End Practice and
and See
See Results
Results Question 171 of 737 Submit Answer
Submit Answer Next Question

Math 
Calculator  Flag 
0:04

Choose the option that best answers the question.


1
Line Q passes through (s, 0) and (0, s), for
some number s where s ≠ 0.  What is the
–1
slope of Line Q?
s

–s

the slope cannot be determined

EndPractice
End Practice and
and See
See Results
Results Question 172 of 737 Submit Answer
Submit Answer Next Question

Math 
Calculator  Flag 
0:02

Enter the answer in the blank.

7 years ago, Samir was 3 times as old as


Deepak. In 4 years, Samir will be twice as
old as Deepak. What is Deepak’s present
age?

EndPractice
End Practice and
and See
See Results
Results Question 173 of 737 Submit Answer
Submit Answer Next Question

Math 
Calculator  Flag 
0:02

Choose the correct statement.

The quantity in Column A is greater


The quantity in Column B is greater

The two quantities are equal

The relationship cannot be


determined from the information
Column A Column B given

Length of AB Length of BC

EndPractice
End Practice and
and See
See Results
Results Question 174 of 737 Submit Answer
Submit Answer Next Question

Math 
Calculator  Flag 
0:02

Enter the answer in the blank.

The total number of people at


Melpomene High School has 400 students, Melpomene High School who are
and Thalia High School has 700 students. involved in band and at least one other
The following table shows the percentage group is __________________.
breakdown for various groups in each
school.
EndPractice
End Practice and
and See
See Results
Results Question 175 of 737 Answer
Next Question
Submit Answer
Submit

Math 
Calculator  Flag 
0:01

Choose the option that best answers the question.

How many non-band members at


Melpomene High School has 400 students, Melpomene, regardless of other
and Thalia High School has 700 students. activities, would have to join the band
The following table shows the percentage so that they had the same number of
breakdown for various groups in each band members as does Thalia High
school. School?

12

25

38

46

65

EndPractice
End Practice and
and See
See Results
Results Question 176 of 737 Answer
Next Question
Submit Answer
Submit

Math 
Calculator  Flag 
0:02

Choose the option that best answers the question.

The total number of people in honor


Melpomene High School has 400 students, society at Melpomene High School,
and Thalia High School has 700 students. regardless of other activities, is
The following table shows the percentage approximately what percent higher than
breakdown for various groups in each the total number of people in honor
school. society at Thalia High School,
regardless of other activities?

2%

8%

19%

29%

56%

EndPractice
End Practice and
and See
See Results
Results Question 177 of 737 Submit Answer
Submit Answer Next Question

Math 
Calculator  Flag 
0:02

Choose the option that best answers the question.

11
If the average (arithmetic mean) of 3, 8, w,
x, and y is 14, then the average of w + 2, x
13.2
- 3, and y + 8 is
17

19.4

22

EndPractice
End Practice and
and See
See Results
Results Question 178 of 737 Submit Answer
Submit Answer Next Question
Math 
Calculator  Flag 
0:01

Choose the correct statement.

The quantity in Column A is greater


Column A Column B
The quantity in Column B is greater
Area of a Area of a triangle
rectangle with with base 5 and The two quantities are equal
perimeter 20 height 10.
The relationship cannot be
determined from the information
given

EndPractice
End Practice and
and See
See Results
Results Question 179 of 737 Submit Answer
Submit Answer Next Question

Math 
Calculator  Flag 
0:01

Choose the option that best answers the question.

4 4 4 1/2
If  ​+ ​ = ​   and wx = y, then the average
w x y
(arithmetic mean) of w and x is 1

EndPractice
End Practice and
and See
See Results
Results Question 180 of 737 Submit Answer
Submit Answer Next Question
Math 
Calculator  Flag 
0:01

Choose the correct statement.

(p × q)2​ The quantity in Column A is greater


For numbers p, q, and r, (p × q × r) < 0 and  ​ <0
r
The quantity in Column B is greater
Column A Column B
The two quantities are equal
p*q 0
The relationship cannot be
determined from the information
given

EndPractice
End Practice and
and See
See Results
Results Question 181 of 737 Submit Answer
Submit Answer Next Question

Math 
Calculator  Flag 
0:01

Choose the option that best answers the question.

The fractions to the left are ordered


31
I.  ​ from least to greatest value in which of
50
the following?  
59
II.  ​
100 I, II, III
299
III.  ​
500 I, III, II

II, I, III

II, III, I
III, II, I 

EndPractice
End Practice and
and See
See Results
Results Question 182 of 737 Submit Answer
Submit Answer Next Question

Math 
Calculator  Flag 
0:04

Consider each of the choices separately and select all that apply.

∣ −k ∣ -15
If 6 ∣​∣ ​ + 4∣​∣ > 12 , which of the following
∣ 3 ∣
could be the value of k ? -10

Indicate all values. -5

10

15

20

EndPractice
End Practice and
and See
See Results
Results Question 183 of 737 Submit Answer
Submit Answer Next Question

Math 
Calculator  Flag 
0:01

Enter the answer as a fraction. Fractions do not need to be in simplest form


In the diagram, point D is the center of
the
medium-sized circle that
passes through
C and E,
and it is also the center of the
largest circle that passes through A
and G.
Each of the diameters of the
small circles
with centers B and F
equals the radius of
the medium-sized circle with center D. The
shaded area is what fraction of the largest
circle? 

EndPractice
End Practice and
and See
See Results
Results Question 184 of 737 Submit Answer
Submit Answer Next Question

Math 
Calculator  Flag 
0:01

Choose the correct statement.

The quantity in Column A is greater

The quantity in Column B is greater


The two quantities are equal

The relationship cannot be


Column A Column B determined from the information
given
Length of AB 4

EndPractice
End Practice and
and See
See Results
Results Question 185 of 737 Submit Answer
Submit Answer Next Question

Math 
Calculator  Flag 
0:01

Choose the option that best answers the question.

4
What is the average (arithmetic mean) of 
​ ​ ​ ​
(√​8 + √​2)2​ and (√​8 − √​2)2​ ?
5

10

12

EndPractice
End Practice and
and See
See Results
Results Question 186 of 737 Submit Answer
Submit Answer Next Question

Math 
Calculator  Flag 
0:01
Choose the correct statement.

The quantity in Column A is greater


Column A Column B
The quantity in Column B is greater
90800 8000400
The two quantities are equal

The relationship cannot be


determined from the information
given

EndPractice
End Practice and
and See
See Results
Results Question 187 of 737 Submit Answer
Submit Answer Next Question

Math 
Calculator  Flag 
0:01

Choose the option that best answers the question.

If we were to take the average number


The following chart shows the employee of marketers at five of the companies,
breakdown, number of marketers vs. excluding Fermion, then this average
number of programmers, at six similarly would be approximately how many
sized companies in a particular internet employees larger than if Fermion were
market.  All six have similarly sized included in the average? 
management teams that are not displayed:
all the non-management employees are 6
displayed.  
13

20

26

33
EndPractice
End Practice and
and See
See Results
Results Question 188 of 737 Submit Answer
Submit Answer Next Question

Math 
Calculator  Flag 
0:01

Choose the option that best answers the question.

At Jericho, approximately what percent


The following chart shows the employee of the non-management employees are
breakdown, number of marketers vs. marketers?
number of programmers, at six similarly
sized companies in a particular internet 28%
market.  All six have similarly sized
42%
management teams that are not displayed:
all the non-management employees are
51%
displayed.  
66%

72%
EndPractice
End Practice and
and See
See Results
Results Question 189 of 737 Submit Answer
Submit Answer Next Question

Math 
Calculator  Flag 
0:01

Choose the option that best answers the question.

At Pindar, there are approximately how


The following chart shows the employee many more programmers than
breakdown, number of marketers vs. marketers? 
number of programmers, at six similarly
sized companies in a particular internet 6
market.  All six have similarly sized
22
management teams that are not displayed:
all the non-management employees are
35
displayed.  
57

73

EndPractice
End Practice and
and See
See Results
Results Question 190 of 737 Submit Answer
Submit Answer Next Question

Math 
Calculator  Flag 
0:01
Choose the option that best answers the question.

2x
If 3xm + 2ym − 2yn − 3xn = 0 and m ≠ n , −​
3
then what is the value of y in terms of x?
3x
−​
2
3x2​

2
2x

3
3x

2

EndPractice
End Practice and
and See
See Results
Results Question 191 of 737 Submit Answer
Submit Answer Next Question

Math 
Calculator  Flag 
0:02

Choose the correct statement.

The quantity in Column A is greater


Andy drove from Townville to Villageton at
an average speed of 40 miles per hour. He
The quantity in Column B is greater
then drove the same route back from
Villageton to Townville at an average The two quantities are equal
speed of 60 miles per hour.
The relationship cannot be
Column Column B determined from the information
A given

50 The average speed of Andy's


entire trip in miles per hour.
EndPractice
End Practice and
and See
See Results
Results Question 192 of 737 Submit Answer
Submit Answer Next Question

Math 
Calculator  Flag 
0:02

Choose the option that best answers the question.

In terms of x and y, what is the area of the x2​ − y2​


shaded region? ​
√​3 (x2​ − y2​ )
Note: Figure not drawn to scale ​
√​3 x2​ − y2​

x2​ − y2​

2

​​√​3​ (x2​ − y2​ )

​2

EndPractice
End Practice and
and See
See Results
Results Question 193 of 737 Submit Answer
Submit Answer Next Question

Math 
Calculator  Flag 
0:01

Consider each of the choices separately and select all that apply.

6.9
Three cube-shaped aquariums that are five
inches on each side are filled with water to
8.4
capacity. All of the water from those
three
aquariums is to be transferred into a larger 9.5
cube aquarium so that it must be filled to at
least 50% of its total capacity without
overflowing.

Which of the following could be the


length,
in inches, of a side of the larger aquarium?

Indicate all
possible values.

EndPractice
End Practice and
and See
See Results
Results Question 194 of 737 Submit Answer
Submit Answer Next Question

Math 
Calculator  Flag 
0:01

Enter the answer in the blank.

List
S = {16, 9, 23, X, 13, 16}
In list S, the mean and median and mode
are all equal to one another. What is the
value of X? 

EndPractice
End Practice and
and See
See Results
Results Question 195 of 737 Submit Answer
Submit Answer Next Question

Math 
Calculator  Flag 
0:01

Choose the correct statement.


The quantity in Column A is greater
In the sequence a1, a2, ... , an each term
after the first term is equal to the preceding
The quantity in Column B is greater
term plus a constant c.

a1 + a11 + a21 = 99 The two quantities are equal

                 The relationship cannot be


determined from the information
                   Column A                              
given
 Column B

                     a 3​ + a19
​                    
  66

EndPractice
End Practice and
and See
See Results
Results Question 196 of 737 Answer
Next Question
Submit Answer
Submit

Math 
Calculator  Flag 
0:02

Choose the option that best answers the question.

5 1 1 −6
​ x + ​ y =  ​
6 4 4
1 1 5 −3
​ x −  ​ y =  ​
3 6 2
3
Which of the following is the value of x?
6

12

EndPractice
End Practice and
and See
See Results
Results Question 197 of 737 Answer
Next Question
Submit Answer
Submit


Calculator  Flag 
0:01
Math

Choose the correct statement.

The quantity in Column A is greater

The quantity in Column B is greater

The two quantities are equal

The relationship cannot be


Column A Column B
determined from the information

BD

BC


given
AB DC

EndPractice
End Practice and
and See
See Results
Results Question 198 of 737 Answer
Next Question
Submit Answer
Submit

Math 
Calculator  Flag 
0:01

Choose the option that best answers the question.

0.005%
In 2008, the total revenue from movies was
842.1 million dollars. If Rocky XII generated
0.05%
4.2 million dollars in revenue,
approximately what percent of the total 0.5%
revenue was generated by Rocky XII?
4%

5%

EndPractice
End Practice and
and See
See Results
Results Question 199 of 737 Answer
Next Question
Submit Answer
Submit
Math 
Calculator  Flag 
0:01

Choose the correct statement.

This year, Bonnie’s annual salary was The quantity in Column A is greater
increased by 15% to $68,425
The quantity in Column B is greater

Column A Column
The two quantities are equal
B
The relationship cannot be
Bonnie's annual salary last $60,000
determined from the information
year
given

EndPractice
End Practice and
and See
See Results
Results Question 200 of 737 Submit Answer
Submit Answer Next Question
Math 
Calculator  Flag 
0:01

Choose the option that best answers the question.

6
In the xy-coordinate system, points (2, 9)
and (–1, 0) lie on the straight line k. If the
7
point (n, 21) lies on line k, what is the value
of n? 8

10

EndPractice
End Practice and
and See
See Results
Results Question 201 of 737 Submit Answer
Submit Answer Next Question

Math 
Calculator  Flag 
0:01

Consider each of the choices separately and select all that apply.

1 N −10
If ( − ​ )​ > −8 , which of the following could
2
−7
be the value of N?

−3

10

EndPractice
End Practice and
and See
See Results
Results Question 202 of 737 Submit Answer
Submit Answer Next Question

Math 
Calculator  Flag 
0:01

Enter the answer in the blank.

The diagonal of a polygon is a line segment


from
any vertex to any non-adjacent vertex.
The diagram at right shows
a regular
decagon, a 10-sided
polygon, with
two diagonals drawn. How many possible
diagonals does the regular
decagon have? 

EndPractice
End Practice and
and See
See Results
Results Question 203 of 737 Submit Answer
Submit Answer Next Question

Math 
Calculator  Flag 
0:01

Choose the correct statement.


X is the sum of the first 31 positive odd The quantity in Column A is greater
integers
The quantity in Column B is greater
Y is the sum of the first 30 positive even
integers The two quantities are equal

Column A Column B The relationship cannot be


determined from the information
X − Y 30 given

EndPractice
End Practice and
and See
See Results
Results Question 204 of 737 Submit Answer
Submit Answer Next Question

Math 
Calculator  Flag 
0:01

Choose the option that best answers the question.

What was the percent decrease in the


number of Japanese graduates from
2003 to 2004?

0.2

20

25

EndPractice
End Practice and
and See
See Results
Results Question 205 of 737 Submit Answer
Submit Answer Next Question
Math 
Calculator  Flag 
0:01

Choose the option that best answers the question.

How many more French students


graduated in 2004 than in 2003?

120

202

222

252

EndPractice
End Practice and
and See
See Results
Results Question 206 of 737 Answer
Next Question
Submit Answer
Submit

Math 
Calculator  Flag 
0:03

Choose the option that best answers the question.

2
What is the value of x?

6
EndPractice
End Practice and
and See
See Results
Results Question 207 of 737 Answer
Next Question
Submit Answer
Submit

Math 
Calculator  Flag 
0:07

Choose the correct statement.

Anne pays 150 percent more for a The quantity in Column A is greater
wholesale widget than Bart pays.
The quantity in Column B is greater
Anne’s retail price per widget is 15 percent
greater than the wholesale price she paid. The two quantities are equal

Bart’s retail price per widget is 185 percent The relationship cannot be
greater than the wholesale price he paid. determined from the information
given
Column A Column B

Anne's retail price Bart's retail price


per widget. per widget.

EndPractice
End Practice and
and See
See Results
Results Question 208 of 737 Answer
Next Question
Submit Answer
Submit

Math 
Calculator  Flag 
0:01

Choose the option that best answers the question.

1
On a county-wide baseball team, the best ​
3
players were sent from each high school. 
There are three mutually exclusive 4

13
categories of players on this team:
infielders, outfielders, and pitchers.  If the 5

ratio of infielders to outfielders is 7:4, and 16
the ratio of pitchers to outfielders is 5:3, 20

then if we pick one player at random from 33
the county-wide baseball team, what is the 20
probability that we will pick a pitcher?  ​
53

EndPractice
End Practice and
and See
See Results
Results Question 209 of 737 Submit Answer
Submit Answer Next Question

Math 
Calculator  Flag 
0:01

Choose the correct statement.

The quantity in Column A is greater

The quantity in Column B is greater

The two quantities are equal

The relationship cannot be


determined from the information
Column A Column B
given
AB BC

EndPractice
End Practice and
and See
See Results
Results Question 210 of 737 Submit Answer
Submit Answer Next Question

Math 
Calculator  Flag 
0:01
Choose the option that best answers the question.

1/
A box contains 4 red chips and 2 blue 2

chips. If two chips are selected at random 8/


15
without replacement, what is the probability
that the chips are different colors? 7/
12

2/
3

7/
10

EndPractice
End Practice and
and See
See Results
Results Question 211 of 737 Answer
Next Question
Submit Answer
Submit

Math 
Calculator  Flag 
0:01

Consider each of the choices separately and select all that apply.

x = 3 and y = 7
If x + y ≠ 0, which of the following is a
solution to the inequality
x = −3 and y = 7
x2​ − y2​ − 1 −1
​ > ​ ?
x+y x+y
x = −11 and y = −9
Indicate all solutions.
x = 9 and y = −6

x = −20 and y = −24

x = 12 and y = 9

x = −2 and y = 16

EndPractice
End Practice and
and See
See Results
Results Question 212 of 737 Answer
Next Question
Submit Answer
Submit
Math 
Calculator  Flag 
0:01

Enter the answer in the blank.

List A: {x, x, x, y, y, y, 3x+y, x–y }

If the median of list A is 10 and 0 < x < y,


what is the range of list A?

EndPractice
End Practice and
and See
See Results
Results Question 213 of 737 Submit Answer
Submit Answer Next Question

Math 
Calculator  Flag 
0:01

Choose the correct statement.

When Car S covered a distance of D on a The quantity in Column A is greater


track, it covered 25% more distance that
The quantity in Column B is greater
Car T had covered on the same track.
The two quantities are equal
Column A Column
B The relationship cannot be
determined from the information
the distance covered by Car 0.80D
given
T

EndPractice
End Practice and
and See
See Results
Results Question 214 of 737 Submit Answer
Submit Answer Next Question
Math 
Calculator  Flag 
0:01

Choose the option that best answers the question.

Approximately what percent of nations


The two charts below summarize 2013 in Europe have a national population
information for the 44 nations that occupy less than 500,000 people?
the continent of Europe.  (The diagram
excludes Russia, which occupies both 18%
Europe & Asia.) The top chart includes
22%
"larger" nations (populations > 500,000),
and the bottom chart includes "smaller"
27%
nations (populations < 500,000).  Those
nations in the "NATO member" row, as of 36%
2013, were members of the NATO military
alliance. Those nations in the "Euro" 50%
column, as of 2013, used the Euro as their
primary currency.

                
EndPractice
End Practice and
and See
See Results
Results Question 215 of 737 Submit Answer
Submit Answer Next Question

Math 
Calculator  Flag 
0:01

Choose the option that best answers the question.

Of the nations with national populations


The two charts below summarize 2013 more than half a million people,
information for the 44 nations that occupy approximately what percent of
the continent of Europe.  (The diagram European nations are neither members
excludes Russia, which occupies both of NATO nor primary users of the euro?
Europe & Asia.) The top chart includes
"larger" nations (populations > 500,000), 25%
and the bottom chart includes "smaller"
27%
nations (populations < 500,000).  Those
nations in the "NATO member" row, as of
31%
2013, were members of the NATO military
alliance. Those nations in the "Euro" 47%
column, as of 2013, used the Euro as their
primary currency. 75%
EndPractice
End Practice and
and See
See Results
Results Question 216 of 737 Submit Answer
Submit Answer Next Question

Math 
Calculator  Flag 
0:01

Choose the option that best answers the question.

Of all the nations with populations


The two charts below summarize 2013 greater than 500,000 who are not
information for the 44 nations that occupy members of NATO, approximately what
the continent of Europe.  (The diagram percent of them use the euro as their
excludes Russia, which occupies both primary currency?
Europe & Asia.) The top chart includes
"larger" nations (populations > 500,000), 26%
and the bottom chart includes "smaller"
35%
nations (populations < 500,000).  Those
nations in the "NATO member" row, as of
41%
2013, were members of the NATO military
alliance. Those nations in the "Euro" 48%
column, as of 2013, used the Euro as their
primary currency. 63%

                

EndPractice
End Practice and
and See
See Results
Results Question 217 of 737 Answer
Next Question
Submit Answer
Submit

Math 
Calculator  Flag 
0:02

Enter the answer in the blank.

How many nations in Europe have a


The two charts below summarize 2013 population more than 500,000, are
information for the 44 nations that occupy members of NATO, and do not use the
the continent of Europe.  (The diagram euro as their primary currency?
excludes Russia, which occupies both
Europe & Asia.) The top chart includes
"larger" nations (populations > 500,000),
and the bottom chart includes "smaller"
nations (populations < 500,000).  Those
nations in the "NATO member" row, as of
2013, were members of the NATO military
alliance. Those nations in the "Euro"
column, as of 2013, used the Euro as their
primary currency.

EndPractice
End Practice and
and See
See Results
Results Question 218 of 737 Submit Answer
Submit Answer Next Question

Math 
Calculator  Flag 
0:01

Choose the option that best answers the question.

Consider the single nation in the upper


The two charts below summarize 2013 right-hand box of the lower chart.  This
information for the 44 nations that occupy has to be which of the following
the continent of Europe.  (The diagram nations?
excludes Russia, which occupies both
Europe & Asia.) The top chart includes Iceland (population = 103,000);
"larger" nations (populations > 500,000), NATO member; primary currency =
and the bottom chart includes "smaller" krona
nations (populations < 500,000).  Those
Latvia (population = 2,067,900);
nations in the "NATO member" row, as of
NATO member; primary currency =
2013, were members of the NATO military
lats
alliance. Those nations in the "Euro"
column, as of 2013, used the Euro as their
Luxembourg (population =
primary currency.
448,569); NATO member; primary
currency = euro

Montenegro (population =
616,258); not a NATO member;
primary currency = euro

Vatican City (population = 900); not


a NATO member; primary currency
= euro

EndPractice
End Practice and
and See
See Results
Results Question 219 of 737 Submit Answer
Submit Answer Next Question

Math 
Calculator  Flag 
0:03

Choose the option that best answers the question.


What is the least number of nations
The two charts below summarize 2013 currently not members of NATO that
information for the 44 nations that occupy would have to join NATO so that more
the continent of Europe.  (The diagram than 50% of the nations in Europe
excludes Russia, which occupies both would be members of NATO?
Europe & Asia.) The top chart includes
"larger" nations (populations > 500,000), 1
and the bottom chart includes "smaller"
2
nations (populations < 500,000).  Those
nations in the "NATO member" row, as of
3
2013, were members of the NATO military
alliance. Those nations in the "Euro" 4
column, as of 2013, used the Euro as their
primary currency. 5

EndPractice
End Practice and
and See
See Results
Results Question 220 of 737 Submit Answer
Submit Answer Next Question
Math 
Calculator  Flag 
0:01

Choose the option that best answers the question.

10
An office has 6 employees; there are 5
female employees and 1 male employee.
12
In how many ways can a 3-person
committee be created if the committee 15
must include the male employee?
24

30

EndPractice
End Practice and
and See
See Results
Results Question 221 of 737 Submit Answer
Submit Answer Next Question

Math 
Calculator  Flag 
0:01

Choose the correct statement.

The quantity in Column A is greater

The quantity in Column B is greater

The two quantities are equal

The relationship cannot be


determined from the information
given
Column A Column B
Length of AO Length of AB

EndPractice
End Practice and
and See
See Results
Results Question 222 of 737 Answer
Next Question
Submit Answer
Submit

Math 
Calculator  Flag 
0:01

Choose the option that best answers the question.

–8
If the line passes through the origin, what is
the value of k?
–12.5

–18

–24.5

–28

EndPractice
End Practice and
and See
See Results
Results Question 223 of 737 Answer
Next Question
Submit Answer
Submit

Math 
Calculator  Flag 
0:02

Choose the correct statement.


The median of x , y , 8 and 11 is 19. The quantity in Column A is greater

The quantity in Column B is greater


Column A Column B

x 23 The two quantities are equal

The relationship cannot be


determined from the information
given

EndPractice
End Practice and
and See
See Results
Results Question 224 of 737 Submit Answer
Submit Answer Next Question

Math 
Calculator  Flag 
0:01

Choose the option that best answers the question.

540
Sue planted 4 times as many apple seeds
as she planted orange seeds. 15 percent of
600
the apple seeds grew into trees, and 10
percent of the orange seeds grew into 660
trees. If a total of 420 apple trees and
orange trees grew from the seeds, how 720
many orange seeds did Sue plant?
760

EndPractice
End Practice and
and See
See Results
Results Question 225 of 737 Submit Answer
Submit Answer Next Question

Math 
Calculator  Flag 
0:03

Consider each of the choices separately and select all that apply.
1.1
Point A (−4, 2) and Point B (2, 4) lie in
the
xy-coordinate
plane. If point C lies in the
3.9
first quadrant and contains the coordinates
(p,
q), where p < 2 and q < 4, which of the 11.9
following could be the area of
triangle
ABC?

Indicate all such


numbers

EndPractice
End Practice and
and See
See Results
Results Question 226 of 737 Submit Answer
Submit Answer Next Question

Math 
Calculator  Flag 
0:01

Enter the answer in the blank.

Cinthia moved into her apartment in 2014.


 In 2015, the rent increased 30% from the
previous year's value.  In 2016, the rent
increased 10% from the previous year's
value. In 2017, the rent increased 40%
from the previous year's value.   The value
of her rent in 2017 was what percent
greater than her rent in 2014?

Give your answer to the nearest 0.1


percent.  

EndPractice
End Practice and
and See
See Results
Results Question 227 of 737 Submit Answer
Submit Answer Next Question
Math 
Calculator  Flag 
0:01

Choose the correct statement.

p−q 2 The quantity in Column A is greater


For positive numbers p and q,  ​ =​
p+q 3
The quantity in Column B is greater
Column A Column B
The two quantities are equal
p+q 5
The relationship cannot be
determined from the information
given

EndPractice
End Practice and
and See
See Results
Results Question 228 of 737 Submit Answer
Submit Answer Next Question

Math 
Calculator  Flag 
0:03

Choose the option that best answers the question.

60
It took Ellen 6 hours to ride her bike a total
distance of 120 miles. For the first part of
62.5
the trip, her speed was constantly 25 miles
per hour. For the second part of her trip, 66 2/3
her speed was constantly 15 miles per
hour. For how many miles did Ellen travel 75
at 25 miles per hour?
90

EndPractice
End Practice and
and See
See Results
Results Question 229 of 737 Submit Answer
Submit Answer Next Question
Math 
Calculator  Flag 
0:01

Choose the correct statement.

The quantity in Column A is greater


Column A Column B
The quantity in Column B is greater
43 percent of 207 85
The two quantities are equal

The relationship cannot be


determined from the information
given

EndPractice
End Practice and
and See
See Results
Results Question 230 of 737 Answer
Next Question
Submit Answer
Submit

Math 
Calculator  Flag 
0:02

Choose the option that best answers the question.

M 1
Walking a constant pace on flat ground, a ​ +​
5 5
hiker can cover M miles in 5 hours.  On
another day, if she walks again at this 6
same pace on flat ground, how many miles
M 5
can she cover in (1 + M) hours? ​ +​
5 M
5M

M+1

M2​ + M

5
EndPractice
End Practice and
and See
See Results
Results Question 231 of 737 Submit Answer
Submit Answer Next Question

Math 
Calculator  Flag 
0:02

Choose the correct statement.

The quantity in Column A is greater

The quantity in Column B is greater

The two quantities are equal

The relationship cannot be


Column A Column B
determined from the information
x y given

EndPractice
End Practice and
and See
See Results
Results Question 232 of 737 Submit Answer
Submit Answer Next Question

Math 
Calculator  Flag 
0:01

Choose the option that best answers the question.

4
w 2 w 8 (x + y) ​
If  ​ = ​ and ​ = ​ , then ​ = 5
x 3 y 15 y
6

5
7

5
8

5
9

5

EndPractice
End Practice and
and See
See Results
Results Question 233 of 737 Submit Answer
Submit Answer Next Question

Math 
Calculator  Flag 
0:01

Consider each of the choices separately and select all that apply.

4.16496
Which of the following when rounded to the
nearest hundredths, are rounded to 4.17?
4.16501

4.16849

4.17469

4.17496

4.17501

EndPractice
End Practice and
and See
See Results
Results Question 234 of 737 Submit Answer
Submit Answer Next Question

Math 
Calculator  Flag 
0:03

Enter the answer in the blank.

In Alioth Industries, 20% of the


employees
have advanced degrees and the others
have bachelor's degrees. The average
salary for the employees with
advanced
degree is $350,000, and the average salary
for employees with
bachelor's degrees is
$100,000. What is
the average salary, in
dollars, for all the employees at Alioth
Industries? 

EndPractice
End Practice and
and See
See Results
Results Question 235 of 737 Answer
Next Question
Submit Answer
Submit

Math 
Calculator  Flag 
0:02

Choose the correct statement.

The quantity in Column A is greater

The quantity in Column B is greater

The two quantities are equal

The relationship cannot be


determined from the information
given

Column A Column B

AB BC

EndPractice
End Practice and
and See
See Results
Results Question 236 of 737 Answer
Next Question
Submit Answer
Submit
Math 
Calculator  Flag 
0:01

Choose the option that best answers the question.

4 and 5
x is between

Note: Figure not drawn to scale 5 and 6

6 and 7

7 and 8

8 and 9

EndPractice
End Practice and
and See
See Results
Results Question 237 of 737 Submit Answer
Submit Answer Next Question

Math 
Calculator  Flag 
0:01

Choose the correct statement.

The quantity in Column A is greater


x and y are integers greater than 5.

x is y percent of x2 The quantity in Column B is greater

Column A Column B The two quantities are equal

x 10 The relationship cannot be


determined from the information
given

End Practice and See Results Question 238 of 737 Submit Answer Next Question
End Practice and See Results Submit Answer

Math 
Calculator  Flag 
0:01

Choose the option that best answers the question.

10
Joan has 100 candies to distribute among
10 children. If each child receives at least 1
34
candy and no two children receive the
same number of candies, what is the 39
maximum number of candies that a child
can receive? 45

55

EndPractice
End Practice and
and See
See Results
Results Question 239 of 737 Submit Answer
Submit Answer Next Question

Math 
Calculator  Flag 
0:01

Choose the correct statement.

Events A and B are independent. The quantity in Column A is greater

The probability that events A and B both The quantity in Column B is greater
occur is 0.6
The two quantities are equal
Column A Column
B The relationship cannot be
determined from the information
The probability that event A 0.3 given
occurs

EndPractice
End Practice and
and See
See Results
Results Question 240 of 737 Submit Answer
Submit Answer Next Question
Math 
Calculator  Flag 
0:01

Choose the option that best answers the question.

​ ​
​​2 + √​2​ 2 + 4√​2
​ ​ =
​2 − √​2​ ​
3 + 2√​2

4 + √​2

5 + √​2

8 − 2√​2

EndPractice
End Practice and
and See
See Results
Results Question 241 of 737 Submit Answer
Submit Answer Next Question

Math 
Calculator  Flag 
0:01

Consider each of the choices separately and select all that apply.

8
Square ABCD has a side of 20. Circle O
has a radius of r. If the circle has more area
9
than does the square, then which of the
following could be the value of r?  10
Indicate all possible values of the radius.
11

12

13
14 
15

EndPractice
End Practice and
and See
See Results
Results Question 242 of 737 Answer
Next Question
Submit Answer
Submit

Math 
Calculator  Flag 
0:01

Enter the answer as a fraction. Fractions do not need to be in simplest form

6 8 3
If ​  of k is  ​ , what is ​  of k?
11 41 11

EndPractice
End Practice and
and See
See Results
Results Question 243 of 737 Answer
Next Question
Submit Answer
Submit

Math 
Calculator  Flag 
0:02

Choose the correct statement.

The quantity in Column A is greater


a , b , c and d are different positive
numbers.
The quantity in Column B is greater
The average (arithmetic mean) of a and b
is 30. The two quantities are equal

The average of a , b , c and d is 40. The relationship cannot be


determined from the information
Column A Column given
B

The greatest possible value 99


of d

EndPractice
End Practice and
and See
See Results
Results Question 244 of 737 Submit Answer
Submit Answer Next Question

Math 
Calculator  Flag 
0:01

Enter the answer in the blank.

The ratio of people to televisions in


The following chart shows the population of Jenkinsville decreased by
Jenkinsville and the number of televisions approximately what percent from 1955
in the town through the middle of part of the to 1960?  
20th century.
Give your answer to the nearest integer
percent and do not enter the percent
sign.  

EndPractice
End Practice and
and See
See Results
Results Question 245 of 737 Submit Answer
Submit Answer Next Question


Calculator  Flag 
0:01
Math

Choose the option that best answers the question.

By approximately what percent did the


The following chart shows the population of number of televisions in Jenkinsville
Jenkinsville and the number of televisions increase from 1945 to 1950?
in the town through the middle of part of the
20th century. 20%

33%

40%

67%

80%

EndPractice
End Practice and
and See
See Results
Results Question 246 of 737 Submit Answer
Submit Answer Next Question

Math 
Calculator  Flag 
0:02

Consider each of the choices separately and select all that apply.

In which of the following years had the


The following chart shows the population of population of Jenkinsville increased by
Jenkinsville and the number of televisions more than 20% from five years
in the town through the middle of part of the before?  
20th century.
Indicate all such years.

1940
1945

1950

1955

1960

EndPractice
End Practice and
and See
See Results
Results Question 247 of 737 Answer
Next Question
Submit Answer
Submit

Math 
Calculator  Flag 
0:01

Choose the option that best answers the question.

Which of the following is closest to the


The following chart shows the population of average percentage increase over a
Jenkinsville and the number of televisions five-year span in the number of TVs in
in the town through the middle of part of the Jenkinsville from 1935 to 1960?
20th century.
18%

56%

96%

127%

838%

EndPractice
End Practice and
and See
See Results
Results Question 248 of 737 Answer
Next Question
Submit Answer
Submit
Math 
Calculator  Flag 
0:03

Consider each of the choices separately and select all that apply.

For which of the following years was


The following chart shows the population of the ratio of population to televisions in
Jenkinsville and the number of televisions Jenkinsville greater than 20?  
in the town through the middle of part of the
20th century. Indicate all such years.

1935

1940

1945

1950

1955

1960

EndPractice
End Practice and
and See
See Results
Results Question 249 of 737 Submit Answer
Submit Answer Next Question

Math 
Calculator  Flag 
0:01

Choose the option that best answers the question.

From 1955 to 1960 in Jenkinsville, what


The following chart shows the population of is the ratio of the percentage
Jenkinsville and the number of televisions increase in televisions to the
in the town through the middle of part of the percentage increase in population?
20th century.
0.75

1.2

1.75

3.5

12.5

EndPractice
End Practice and
and See
See Results
Results Question 250 of 737 Answer
Next Question
Submit Answer
Submit

Math 
Calculator  Flag 
0:01

Choose the option that best answers the question.

7/
A box contains 6 black balls and 4 white 90

balls. If two balls are selected at random 3/


25
without replacement, what is the probability
that both balls are white? 2/
15

4/
25

4/
9

EndPractice
End Practice and
and See
See Results
Results Question 251 of 737 Answer
Next Question
Submit Answer
Submit

Math 
Calculator  Flag 
0:01
Choose the correct statement.

The Quantity A is greater


Quantity A Quantity B
The Quantity B is greater
the number of the number of
multiples of 3 multiples of 7 The two quantities are equal
between 331 and between 721 and
341  731    The relationship cannot be
determined from the information
given

EndPractice
End Practice and
and See
See Results
Results Question 252 of 737 Submit Answer
Submit Answer Next Question

Math 
Calculator  Flag 
0:01

Choose the option that best answers the question.

What is the area of the circle? 9π

12π

16π

24π

36π

EndPractice
End Practice and
and See
See Results
Results Question 253 of 737 Submit Answer
Submit Answer Next Question
Math 
Calculator  Flag 
0:01

Choose the correct statement.

A certain company has 200 employees, all The quantity in Column A is greater
of whom are either programmers or
The quantity in Column B is greater
marketers.  Among these, 20% of the
programmers own pets, and 23% of the The two quantities are equal
marketers own pets.
The relationship cannot be
Column A Column determined from the information
B given

the total number of employees 43


who own pets

EndPractice
End Practice and
and See
See Results
Results Question 254 of 737 Submit Answer
Submit Answer Next Question

Math 
Calculator  Flag 
0:01

Choose the option that best answers the question.

199
If the sum of three consecutive integers is
K , then which of the following is a possible
200
value of K?
201

202

203
EndPractice
End Practice and
and See
See Results
Results Question 255 of 737 Answer
Next Question
Submit Answer
Submit

Math 
Calculator  Flag 
0:01

Consider each of the choices separately and select all that apply.

75
On a certain standardized test, the mean is
180 and the standard deviation is 35.
100
 Which of the following is within 2 standard
deviations of the mean? 150
Indicated all such numbers.
200

225

275

EndPractice
End Practice and
and See
See Results
Results Question 256 of 737 Answer
Next Question
Submit Answer
Submit

Math 
Calculator  Flag 
0:01

Enter the answer in the blank.

Dharik lives in a house on a straight street.


 For years, there have been 16 houses on
his street to the right of his house and 17
houses on his street to the left of his house.
 Last year, 5 new houses were built on the
same street even further to the left of those
houses to the left of Dharik’s house. If
these are the only houses on this street,
how many houses are on this street?

EndPractice
End Practice and
and See
See Results
Results Question 257 of 737 Submit Answer
Submit Answer Next Question

Math 
Calculator  Flag 
0:03

Choose the correct statement.

The quantity in Column A is greater


Column A Column B
The quantity in Column B is greater
22 percent of x 2/9 of x
The two quantities are equal

The relationship cannot be


determined from the information
given

EndPractice
End Practice and
and See
See Results
Results Question 258 of 737 Submit Answer
Submit Answer Next Question

Math 
Calculator  Flag 
0:02

Choose the option that best answers the question.

7ab
Point A in the xy-coordinate system is ​
2
shown below. Given two other points B (4a,
b) and C (2a, 5b), what is the area of 9ab

triangle ABC in terms of a and b? 2
15ab

2

4ab

6ab

EndPractice
End Practice and
and See
See Results
Results Question 259 of 737 Submit Answer
Submit Answer Next Question

Math 
Calculator  Flag 
0:01

Choose the correct statement.

Four consecutive positive integers have a The quantity in Column A is greater


sum of 802.  
The quantity in Column B is greater

Column A Column B
The two quantities are equal
the least of the four integers 199
The relationship cannot be
determined from the information
given

EndPractice
End Practice and
and See
See Results
Results Question 260 of 737 Submit Answer
Submit Answer Next Question
Math 
Calculator  Flag 
0:01

Choose the option that best answers the question.

1 hour and 20 minutes


Machine A can make 350 widgets in 1
hour, and machine B can make 250
1 hour and 24 minutes
widgets in 1 hour. If both machines work
together, how much time will it take them to 1 hour and 30 minutes
make a total of 1000 widgets?
1 hour and 36 minutes

1 hour and 40 minutes

EndPractice
End Practice and
and See
See Results
Results Question 261 of 737 Submit Answer
Submit Answer Next Question

Math 
Calculator  Flag 
0:01

Choose the correct statement.

If 148 tickets had been sold, the total The quantity in Column A is greater
revenue for an event would be three times
The quantity in Column B is greater
the cost of sponsoring the event. (Ticket
sales was the only source of revenue.)  All The two quantities are equal
tickets were the same price.  
The relationship cannot be
Column A Column B determined from the information
given
the revenue from the the cost of
sale of 50 tickets sponsoring the
event

EndPractice
End Practice and
and See
See Results
Results Question 262 of 737 Submit Answer
Submit Answer Next Question

Math 
Calculator  Flag 
0:01

Choose the option that best answers the question.

R(T − R)
If the retail price of a shirt is R dollars, and ​
100
the price including sales tax is T dollars
then the sales tax, as a percent, is T −R

100T
100T − R

T
T − 100R

T
100(T − R)

R

EndPractice
End Practice and
and See
See Results
Results Question 263 of 737 Submit Answer
Submit Answer Next Question

Math 
Calculator  Flag 
0:01

Consider each of the choices separately and select all that apply.

(−4, −2)
In the standard x,y-plane, a circle has a
center = (1, –2) and a radius r = 5. Which
(−3, 1)
of the following points are on the circle?

Indicate all possible points. (−1, −6)

(1, −7)

(3, 2)

(4, −6)

(5, 1)

(6, −2)

EndPractice
End Practice and
and See
See Results
Results Question 264 of 737 Submit Answer
Submit Answer Next Question

Math 
Calculator  Flag 
0:03

Enter the answer in the blank.

Give your answer to the nearest 0.1


In a certain set of numbers, 12.5 is 1.5
units of standard deviation above the
mean, and 8.9 is 0.5 units of standard
deviation below the mean. What is the
mean of the set?

EndPractice
End Practice and
and See
See Results
Results Question 265 of 737 Submit Answer
Submit Answer Next Question

Math 
Calculator  Flag 
0:01
Choose the correct statement.

The quantity in Column A is greater


In a certain parking lot at 10:00 am, the
ratio of trucks to cars was 1 to 3.  Between
The quantity in Column B is greater
10:00 am and 11:00 am, no vehicle left the
parking lot, and, for some integer N > 0, N The two quantities are equal
more cars and N more trucks entered the
lot and parked there.   The relationship cannot be
determined from the information
Column A Column given
B

the ratio of trucks to cars in the 1



lot at 11:00 3

EndPractice
End Practice and
and See
See Results
Results Question 266 of 737 Answer
Next Question
Submit Answer
Submit

Math 
Calculator  Flag 
0:02

Choose the option that best answers the question.

1/4
Jack has 5 cats and 1 dog. If the dog’s
weight is 3 times the average (arithmetic
1/3
mean) weight of the cats, then the dog’s
weight is what fraction of the total weight of 3/8
all 6 animals?
3/7

3/5

EndPractice
End Practice and
and See
See Results
Results Question 267 of 737 Answer
Next Question
Submit Answer
Submit
Math 
Calculator  Flag 
0:01

Choose the correct statement.

The quantity in Column A is greater


The numbers p and q are both positive
integers.
The quantity in Column B is greater

Column A Column B
The two quantities are equal
p p 2
​ ( ​ )​
q The relationship cannot be
q
determined from the information
given

EndPractice
End Practice and
and See
See Results
Results Question 268 of 737 Answer
Next Question
Submit Answer
Submit

Math 
Calculator  Flag 
0:01

Choose the option that best answers the question.

1 24 1 k 1 8
If ( ​ )​ ( ​ )​ = ​ 24 , then k =
2 81 18 ​
12

16

24

36
EndPractice
End Practice and
and See
See Results
Results Question 269 of 737 Submit Answer
Submit Answer Next Question

Math 
Calculator  Flag 
0:01

Choose the correct statement.

The quantity in Column A is greater


Column A Column B
The quantity in Column B is greater
Area of circle Area of circle with

with radius diameter √​1​4 The two quantities are equal

√​7​
The relationship cannot be
determined from the information
given

EndPractice
End Practice and
and See
See Results
Results Question 270 of 737 Submit Answer
Submit Answer Next Question

Math 
Calculator  Flag 
0:01

Choose the option that best answers the question.

5100
Before noon on Tuesday,  there was a
certain number of hex nuts in a bin.  After a
5200
delivery at noon that day, the number of
hex nuts in the bin increased by 30%. 5400
 Which of the following could be the
number of hex nuts in the bin after that 5500
delivery?
5600
EndPractice
End Practice and
and See
See Results
Results Question 271 of 737 Answer
Next Question
Submit Answer
Submit

Math 
Calculator  Flag 
0:02

Consider each of the choices separately and select all that apply.

4x+5
If x>0, and two sides of a certain triangle
have lengths 2x+1 and 3x+4 respectively,
x+2
which of the following could be the length
of the third side of the triangle? 6x+1
Indicate all possible lengths.
5x+6

2x+17

EndPractice
End Practice and
and See
See Results
Results Question 272 of 737 Answer
Next Question
Submit Answer
Submit

Math 
Calculator  Flag 
0:01

Enter the answer in the blank.

16,000 has how many positive divisors?

EndPractice
End Practice and
and See
See Results
Results Question 273 of 737 Answer
Next Question
Submit Answer
Submit
Math 
Calculator  Flag 
0:01

Choose the correct statement.

The average (arithmetic mean) of 7 The quantity in Column A is greater


different numbers is 5
The quantity in Column B is greater

Column A Column B
The two quantities are equal
Median of the 7 numbers 5
The relationship cannot be
determined from the information
given

EndPractice
End Practice and
and See
See Results
Results Question 274 of 737 Submit Answer
Submit Answer Next Question

Math 
Calculator  Flag 
0:01

Choose the option that best answers the question.

What is the size of the Interquartile


The following boxplot shows the 2012 Range (IQR) of this distribution?
season runs batted in (RBIs) of 280
American League (AL) batters (the top 280 25
batters in terms of number of plate
47
appearances).

56

83

Five-Number Summary for AL RBIs in 139


2012:

Minimum = 0
First Quartile = 9

Median = 25

Third Quartile = 56

Maximum = 139

EndPractice
End Practice and
and See
See Results
Results Question 275 of 737 Answer
Next Question
Submit Answer
Submit

Math 
Calculator  Flag 
0:01

Choose the option that best answers the question.

If no batter hit exactly 25 RBIs, then


The following boxplot shows the 2012 how many AL hitters hit more than 25
season runs batted in (RBIs) of 280 RBIs in 2012?
American League (AL) batters (the top 280
batters in terms of number of plate 9
appearances).
56

83

114
Five-Number Summary for AL RBIs in
2012: 140
Minimum = 0

First Quartile = 9

Median = 25

Third Quartile = 56

Maximum = 139
EndPractice
End Practice and
and See
See Results
Results Question 276 of 737 Submit Answer
Submit Answer Next Question

Math 
Calculator  Flag 
0:02

Choose the option that best answers the question.

B. J. Upton, who played on the Tampa


The following boxplot shows the 2012 Bay Rays that season, hit 78 RBIs in
season runs batted in (RBIs) of 280 2012; this is the 90th percentile value
American League (AL) batters (the top 280 on this chart. How many players hit
batters in terms of number of plate RBIs totaling 56 or more and less
appearances). than 78?

14

22
Five-Number Summary for AL RBIs in
28
2012:

Minimum = 0 34

First Quartile = 9 42
Median = 25

Third Quartile = 56

Maximum = 139

EndPractice
End Practice and
and See
See Results
Results Question 277 of 737 Submit Answer
Submit Answer Next Question

Math 
Calculator  Flag 
0:01
Choose the option that best answers the question.

2
If k is a positive integer, what is the
smallest possible value of k such that
5
1040k is the square of an integer?
10

15

65

EndPractice
End Practice and
and See
See Results
Results Question 278 of 737 Submit Answer
Submit Answer Next Question

Math 
Calculator  Flag 
0:01

Choose the correct statement.

The quantity in Column A is greater


x and y are positive.

30 percent of x is y. The quantity in Column B is greater

Column A Column B The two quantities are equal

x The relationship cannot be


​ 3
y
determined from the information
given

EndPractice
End Practice and
and See
See Results
Results Question 279 of 737 Submit Answer
Submit Answer Next Question

Math 
Calculator  Flag 
0:01
Choose the option that best answers the question.

​ y
If √​2x2​ + 2xy + 13y2​ = x + 3y , then x = ​
2
y2​

2

2y

y−2

y+2

EndPractice
End Practice and
and See
See Results
Results Question 280 of 737 Submit Answer
Submit Answer Next Question
Math 
Calculator  Flag 
0:01

Choose the correct statement.

The quantity in Column A is greater


Column A Column B
The quantity in Column B is greater
The percent The percent
increase from 11 to decrease from 16 The two quantities are equal
16 to 11
The relationship cannot be
determined from the information
given

EndPractice
End Practice and
and See
See Results
Results Question 281 of 737 Submit Answer
Submit Answer Next Question

Math 
Calculator  Flag 
0:01

Choose the option that best answers the question.

111
If x is the greatest common divisor of 90
and 18, and y is the least common multiple
120
of 51 and 34, then x + y =
213

222

231
EndPractice
Practice and
and See
See Results Question 282 of 737 Submit Answer Next Question
End Results Submit Answer

Math 
Calculator  Flag 
0:02

Consider each of the choices separately and select all that apply.

5x2​ + 65x + 60 5x + 5 –60


If  ​ = ​  , then which of
x2​ + 10x − 24 x−2
the following are possible values of x? –12

Indicate all such values. –1

EndPractice
End Practice and
and See
See Results
Results Question 283 of 737 Submit Answer
Submit Answer Next Question

Math 
Calculator  Flag 
0:02

Enter the answer in the blank.

The sum of the pre-tax costs of


Item A and
Item B is $300. In Alumba,
each item would
be charged a flat 7%. In
Aplandia, Item A is
subject to 5% tax and Item B is subject to
10% tax. If the tax in Aplandia on the
purchase of
both items is exactly $3 more
than it is in Alumba, then what is the pre-
tax
price of Item A? 

EndPractice
End Practice and
and See
See Results
Results Question 284 of 737 Submit Answer
Submit Answer Next Question

Math 
Calculator  Flag 
0:01

Choose the correct statement.

An integer is randomly selected from the The quantity in Column A is greater


integers from 200 to 900 inclusive.
The quantity in Column B is greater

Column A Column
The two quantities are equal
B
The relationship cannot be
Probability that the number is 14/13
determined from the information
either even or prime.
given

EndPractice
End Practice and
and See
See Results
Results Question 285 of 737 Submit Answer
Submit Answer Next Question

Math 
Calculator  Flag 
0:03

Choose the option that best answers the question.

6
For integers p and q, (2p)(5q) = 40000.
 Which of the following is the value of p +
10
q?
12

15

18

EndPractice
End Practice and
and See
See Results
Results Question 286 of 737 Submit Answer
Submit Answer Next Question

Math 
Calculator  Flag 
0:01

Choose the correct statement.

The greatest prime factor of 144 is x The quantity in Column A is greater

The greatest prime factor of 96 is y The quantity in Column B is greater

Column A Column B The two quantities are equal

x y The relationship cannot be


determined from the information
given

EndPractice
End Practice and
and See
See Results
Results Question 287 of 737 Submit Answer
Submit Answer Next Question

Math 
Calculator  Flag 
0:01

Choose the option that best answers the question.

According to the histogram, the lowest


The following two histograms show the possible score from the 40 juniors from
distribution of SAT scores of all forty juniors Celeraville Academy would be 1010.  A
at each of two schools. The Newboard score of 1010 would be what percentile
Free School is a public school, with rank among juniors at Newboard Free
students of a variety of ability levels. The School if no junior there got that exact
Celeraville Academy is an elite college- score?
prep private school for gifted students.
25th percentile
A note on rounding: SAT scores are always
multiples of 10.  In the histograms below, a 30th percentile
score divisible by 100 would be included in
the column below that score: thus, for 55th percentile
example, a score of exactly 1400 would be
70th percentile
included as part of the column between
1300 and 1400.
95th percentile

EndPractice
End Practice and
and See
See Results
Results Question 288 of 737 Submit Answer
Submit Answer Next Question

Math 
Calculator  Flag 
0:01

Enter the answer in the blank.


In how many of the columns is the
The following two histograms show the number of students with a score in that
distribution of SAT scores of all forty juniors category from the Celeraville Academy
at each of two schools. The Newboard greater than the number of students
Free School is a public school, with with a score in that category from the
students of a variety of ability levels. The Newboard Free School?
Celeraville Academy is an elite college-
prep private school for gifted students.

A note on rounding: SAT scores are always


multiples of 10.  In the histograms below, a
score divisible by 100 would be included in
the column below that score: thus, for
example, a score of exactly 1400 would be
included as part of the column between
1300 and 1400.

EndPractice
End Practice and
and See
See Results
Results Question 289 of 737 Submit Answer
Submit Answer Next Question

Math 
Calculator  Flag 
0:02
Choose the option that best answers the question.

The first quartile SAT score among the


The following two histograms show the forty scores at the Newboard Free
distribution of SAT scores of all forty juniors School is in which score range?
at each of two schools. The Newboard
Free School is a public school, with 600−700
students of a variety of ability levels. The
700−800
Celeraville Academy is an elite college-
prep private school for gifted students.
800−900
A note on rounding: SAT scores are always
900−1000
multiples of 10.  In the histograms below, a
score divisible by 100 would be included in
1000−1100
the column below that score: thus, for
example, a score of exactly 1400 would be
included as part of the column between
1300 and 1400.

EndPractice
End Practice and
and See
See Results
Results Question 290 of 737 Submit Answer
Submit Answer Next Question
Math 
Calculator  Flag 
0:01

Choose the option that best answers the question.

2k + 3j
If the average (arithmetic mean) of a and b ​
6
is j, and the average of c, d, and e is k,
what is the average of a, b, c, d, e and j ? k + 2j

3
k+j

2
2k + 3j

5
k + 2j

4

EndPractice
End Practice and
and See
See Results
Results Question 291 of 737 Submit Answer
Submit Answer Next Question

Math 
Calculator  Flag 
0:01

Choose the correct statement.

The quantity in Column A is greater


For positive numbers a, b, and, 
a×b c
c, ​ = 1 and  ​ = 4 The quantity in Column B is greater
c a

Column A Column B The two quantities are equal

b 4 The relationship cannot be


determined from the information
given

EndPractice
End Practice and
and See
See Results
Results Question 292 of 737 Submit Answer
Submit Answer Next Question
Math 
Calculator  Flag 
0:01

Choose the option that best answers the question.

3.5
If x ≠ 2.5 and 2x = |15 - 4x|, then x =

4.5

5.5

6.5

7.5

EndPractice
End Practice and
and See
See Results
Results Question 293 of 737 Submit Answer
Submit Answer Next Question

Math 
Calculator  Flag 
0:01

Consider each of the choices separately and select all that apply.

13
In list S, there are four numbers. Three of
the numbers are 13, 29, and 41, and the
15
fourth number is X. If the mean of the list is
less than 25, what could be the value of X? 17
Indicate all possible values of X.
19

21

23
25

27

EndPractice
End Practice and
and See
See Results
Results Question 294 of 737 Submit Answer
Submit Answer Next Question

Math 
Calculator  Flag 
0:01

Enter the answer as a fraction. Fractions do not need to be in simplest form

30% of 50 is what fraction of 75%


of 80? 

EndPractice
End Practice and
and See
See Results
Results Question 295 of 737 Submit Answer
Submit Answer Next Question

Math 
Calculator  Flag 
0:01

Choose the correct statement.

 y = 5 × 6 ×  14 The quantity in Column A is greater


× 15
The quantity in Column B is greater

Column A Column B
The two quantities are equal
Remainder when y Remainder when y
The relationship cannot be
is divided by 18 is divided by 40
determined from the information
given
EndPractice
End Practice and
and See
See Results
Results Question 296 of 737 Submit Answer
Submit Answer Next Question

Math 
Calculator  Flag 
0:02

Choose the option that best answers the question.

x -10
If 2x – y = 10 and ​  = 3, then x =
y
2

12

EndPractice
End Practice and
and See
See Results
Results Question 297 of 737 Submit Answer
Submit Answer Next Question

Math 
Calculator  Flag 
0:06

Choose the correct statement.

The quantity in Column A is greater

The quantity in Column B is greater

The two quantities are equal

The relationship cannot be


determined from the information
given

O is the center of the semicircle. 

AO = OB

Column A Column B

Area of Area of triangular


semicircular region ABC
region

EndPractice
End Practice and
and See
See Results
Results Question 298 of 737 Submit Answer
Submit Answer Next Question

Math 
Calculator  Flag 
0:03

Choose the option that best answers the question.

1.6x + 3.2
The average (arithmetic mean) of five
numbers is 3x + 4. If one of the numbers is
1.6x + 4.8
7x – 4, what is the average of the other four
numbers? 2x + 4
2x + 6

4x + 8

EndPractice
End Practice and
and See
See Results
Results Question 299 of 737 Submit Answer
Submit Answer Next Question

Math 
Calculator  Flag 
0:06

Choose the correct statement.

The quantity in Column A is greater

The quantity in Column B is greater

The two quantities are equal

The relationship cannot be


determined from the information
Column A Column B given

a 2 + b2 c2

EndPractice
End Practice and
and See
See Results
Results Question 300 of 737 Submit Answer
Submit Answer Next Question
Math 
Calculator  Flag 
0:01

Choose the option that best answers the question.

–4
If 5x – 3y = 7 and 2y – 4x = 3, then 2x – 2y
=
4

16

20

EndPractice
End Practice and
and See
See Results
Results Question 301 of 737 Submit Answer
Submit Answer Next Question

Math 
Calculator  Flag 
0:01

Consider each of the choices separately and select all that apply.

x+y
10x + 10y + 10z = n, where x, y, and z
are
positive integers
y−z
Which of the following could be the
number
of zeroes, to the left of the decimal point, z
contained in n?

Indicate all such


answers

End Practice and See Results Question 302 of 737 Submit Answer Next Question
End Practice and See Results Submit Answer

Math 
Calculator  Flag 
0:02

Enter the answer in the blank.

What is the sum of each distinct


prime
factor of 9999?

EndPractice
End Practice and
and See
See Results
Results Question 303 of 737 Submit Answer
Submit Answer Next Question

Math 
Calculator  Flag 
0:01

Choose the correct statement.

Car X can come with any of these 5 The quantity in Column A is greater
additional features: sunroof, stereo, tinted
The quantity in Column B is greater
windows, leather seats and cruise control.
Any car might have none of those, or The two quantities are equal
any one of those, or any combination
of more than one, or even all five The relationship cannot be
together.   determined from the information
given
Column A Column
B

Number of different 25
combinations possible
EndPractice
End Practice and
and See
See Results
Results Question 304 of 737 Submit Answer
Submit Answer Next Question

Math 
Calculator  Flag 
0:01

Enter the answer as a fraction. Fractions do not need to be in simplest form

Among the students in this trial who


Fifteen college seniors, none of whom took studied for 20 hours or more, what
any math classes in college, were selected fraction of them performed worse than
for a clinical trial.  An online GRE Math expected, according to the best-fit line
Course was made available to them, and model?  Express your answer as a
these students were allowed to study for fraction
however many hours they wanted.  Then,
each took the GRE for their first time.  The
graph below records how many hours each
student studied, using the online material,
and that student's GRE Quant score.   The
best-fit line model shows the best
prediction, given such a student's hours of
study, of what her GRE Q score might be. 

EndPractice
End Practice and
and See
See Results
Results Question 305 of 737 Submit Answer
Submit Answer Next Question
Math 
Calculator  Flag 
0:02

Choose the option that best answers the question.

For the students in this trial with the


Fifteen college seniors, none of whom took three highest scores, approximately
any math classes in college, were selected what is the average number of hours
for a clinical trial.  An online GRE Math they spend studying? 
Course was made available to them, and
these students were allowed to study for 29
however many hours they wanted.  Then,
32
each took the GRE for their first time.  The
graph below records how many hours each
35
student studied, using the online material,
and that student's GRE Quant score.   The 39
best-fit line model shows the best
prediction, given such a student's hours of 43
study, of what her GRE Q score might be. 

EndPractice
End Practice and
and See
See Results
Results Question 306 of 737 Submit Answer
Submit Answer Next Question

Math 
Calculator  Flag 
0:01
Choose the option that best answers the question.

Among the students in this trial who


Fifteen college seniors, none of whom took studied less than 25 hours, what is the
any math classes in college, were selected highest GRE Quant score achieved? 
for a clinical trial.  An online GRE Math
Course was made available to them, and 146
these students were allowed to study for
149
however many hours they wanted.  Then,
each took the GRE for their first time.  The
154
graph below records how many hours each
student studied, using the online material, 157
and that student's GRE Quant score.   The
best-fit line model shows the best 165
prediction, given such a student's hours of
study, of what her GRE Q score might be. 

EndPractice
End Practice and
and See
See Results
Results Question 307 of 737 Submit Answer
Submit Answer Next Question

Math 
Calculator  Flag 
0:02

Choose the option that best answers the question.

3
If the mean of list A is 6.8 and the standard
deviation is 3.6, then how many elements
4
of list A are within 1 unit of standard
deviation of the mean? 5
A = {2, 9, 2, 6, 9, 10, 7, 4, 5, 14}
6

EndPractice
End Practice and
and See
See Results
Results Question 308 of 737 Submit Answer
Submit Answer Next Question

Math 
Calculator  Flag 
0:01

Choose the correct statement.

p + q 10 The quantity in Column A is greater


For positive numbers p and q,  ​ =​
p 7
The quantity in Column B is greater
Column A Column B
The two quantities are equal
p−q 5
​ ​
q 3 The relationship cannot be
determined from the information
given

EndPractice
End Practice and
and See
See Results
Results Question 309 of 737 Submit Answer
Submit Answer Next Question

Math 
Calculator  Flag 
0:01

Choose the option that best answers the question.


1:√​2
If the ratio of the volume of cube A to the
volume of cube B is 1 to 8, what is the ratio
1:2
of the surface area of cube A to the surface
area of cube B?

1:2√​2

1:4

1:8

EndPractice
End Practice and
and See
See Results
Results Question 310 of 737 Submit Answer
Submit Answer Next Question

Math 
Calculator  Flag 
0:01

Choose the correct statement.

In the Endymion neighborhood, there are The quantity in Column A is greater


60 houses: some are ranch-style (i.e. one
The quantity in Column B is greater
level) and the rest are split-levels.   The
average age of the 40 ranch-style houses The two quantities are equal
is 60 years, and the average age of the 20
split-level houses is 70 years. The relationship cannot be
determined from the information
Column A Column given
B

the average age of all 60 65


houses in the Endymion
neighborhood

EndPractice
End Practice and
and See
See Results
Results Question 311 of 737 Submit Answer
Submit Answer Next Question
Math 
Calculator  Flag 
0:01

Choose the option that best answers the question.

31
If xy = 7 and x – y = 5, then x2 + y2 =

39

41

45

58

EndPractice
End Practice and
and See
See Results
Results Question 312 of 737 Submit Answer
Submit Answer Next Question

Math 
Calculator  Flag 
0:02

Consider each of the choices separately and select all that apply.

the total time of the trip


Kemi drove the 240 miles from Houston to
Dallas in two segments: she kept one
average of the two speeds of the
constant speed for the first segment and
two segments
then another constant speed for the second
segment.   the distance of each segment
Which of the following statements
the time of each segment
individually provide(s) sufficient additional
information to determine the average
the speed on each segment
velocity of her trip from Houston to Dallas.

Indicate all such statements.


EndPractice
End Practice and
and See
See Results
Results Question 313 of 737 Submit Answer
Submit Answer Next Question

Math 
Calculator  Flag 
0:01

Enter the answer in the blank.

Let N be a positive number.  The positive


number P is the result when the decimal
point of N is moved 9 spaces to the right.  If
P equals 8 times the reciprocal of N2, what
is the value of N?

EndPractice
End Practice and
and See
See Results
Results Question 314 of 737 Submit Answer
Submit Answer Next Question

Math 
Calculator  Flag 
0:08

Choose the correct statement.

1/3 + 2/5 = N, and, in lowest terms, N = The quantity in Column A is greater
p/q, where p and q are positive integers.
The quantity in Column B is greater

Column A Column B
The two quantities are equal
q 10
The relationship cannot be
determined from the information
given
EndPractice
End Practice and
and See
See Results
Results Question 315 of 737 Submit Answer
Submit Answer Next Question

Math 
Calculator  Flag 
0:01

Choose the option that best answers the question.

A is the center of the circle, and the length 4(4 − π)



of AB is 4√​2. The blue shaded region is a
4(8 − π)
square. What is the area of the shaded
region? 8(2 − π)

8(8 − π)

16(4 − π)

EndPractice
End Practice and
and See
See Results
Results Question 316 of 737 Submit Answer
Submit Answer Next Question

Math 
Calculator  Flag 
0:01

Choose the correct statement.

The quantity in Column A is greater


Four friends win $120,000 in the lottery,
and they divided the winnings in a 1:2:4:5
The quantity in Column B is greater
ratio.
The two quantities are equal
Column A Column
B The relationship cannot be
determined from the information
The difference between the $40,000
given
greatest and least share.

EndPractice
End Practice and
and See
See Results
Results Question 317 of 737 Answer
Next Question
Submit Answer
Submit

Math 
Calculator  Flag 
0:01

Choose the option that best answers the question.

The town that spent $61,000 on the


Crime Initiative had a total number of
violent crimes that was approximately
what percentage of the total number of
violent crime of the town that spent
$40,000 on the Crime Initiative?

6.4%

27.3%

34.4%

41.3%

58.7%

EndPractice
End Practice and
and See
See Results
Results Question 318 of 737 Answer
Next Question
Submit Answer
Submit
Math 
Calculator  Flag 
0:01

Choose the option that best answers the question.

Among these 17 towns, the median


amount invested in the statewide Crime
Initiative is

$37,000

$42,000

$53,000

$62,000

$90,000

EndPractice
End Practice and
and See
See Results
Results Question 319 of 737 Submit Answer
Submit Answer Next Question

Math 
Calculator  Flag 
0:01

Choose the option that best answers the question.

Of the four towns shown each with


fewer than twenty violent crimes in
2009, the average amount they
invested in the statewide Crime
Initiative in 2009 is

$29,250

$41,000

$49,500
$58,750

$64,250

EndPractice
End Practice and
and See
See Results
Results Question 320 of 737 Submit Answer
Submit Answer Next Question
Math 
Calculator  Flag 
0:01

Choose the option that best answers the question.

x
Dimitri weighs x pounds more than Allen y− ​
2
weighs. Together, Allen and Dimitri weigh a
y
total of y pounds. Which of the following 2x − ​
2
represents Allen’s weight?
y−x

2

y − 2x

x − 2y

EndPractice
End Practice and
and See
See Results
Results Question 321 of 737 Submit Answer
Submit Answer Next Question

Math 
Calculator  Flag 
0:01

Choose the correct statement.

The quantity in Column A is greater

The quantity in Column B is greater

The two quantities are equal

The relationship cannot be


determined from the information
given
Column A Column B

Length of arc ABC 6

EndPractice
End Practice and
and See
See Results
Results Question 322 of 737 Answer
Next Question
Submit Answer
Submit

Math 
Calculator  Flag 
0:01

Choose the option that best answers the question.

K must be even
M is a positive two-digit number. When the
digits are reversed, the number is N. If K =
K cannot be square
M + N, which of the following is true?
K cannot be divisible by 13

K must be divisible by 11

If M is even then K must be even

EndPractice
End Practice and
and See
See Results
Results Question 323 of 737 Answer
Next Question
Submit Answer
Submit

Math 
Calculator  Flag 
0:01

Consider each of the choices separately and select all that apply.

(5, 4)

(5, 5)
(5, 6)

(8, 2)

(8, 3)

(8, 4)

The point (2, 7) is shown in the coordinate


plane above.  Point K (not shown) and the
point (2, 7) are on the line Q, which has a
3
slope less than − ​ .  Which of the following
5
could be the coordinates of point K?

Indicated all such coordinates.

EndPractice
End Practice and
and See
See Results
Results Question 324 of 737 Answer
Next Question
Submit Answer
Submit

Math 
Calculator  Flag 
0:01

Enter the answer in the blank.

A librarian has three identical


copies of a
single cookbook and four different novels
that he wants to
display. Assuming all
seven books will
be in a single row, how
many different arrangements can he
make?

EndPractice
End Practice and
and See
See Results
Results Question 325 of 737 Answer
Next Question
Submit Answer
Submit
Math 
Calculator  Flag 
0:02

Choose the correct statement.

Cleve is 4 times as old as Al. Bob is 3 The quantity in Column A is greater


years younger than Al. The sum of their
The quantity in Column B is greater
ages is 81.
The two quantities are equal
Column A Column B
The relationship cannot be
Al's age 13
determined from the information
given

EndPractice
End Practice and
and See
See Results
Results Question 326 of 737 Answer
Next Question
Submit Answer
Submit

Math 
Calculator  Flag 
0:03

Choose the option that best answers the question.

8−x -3
If  ​ = x , then x2 + 2x - 3 =
x+1
1

Cannot be determined
EndPractice
End Practice and
and See
See Results
Results Question 327 of 737 Submit Answer
Submit Answer Next Question

Math 
Calculator  Flag 
0:03

Choose the correct statement.

Triangle A has sides with length 5, 5 and 8. The quantity in Column A is greater

Triangle B has sides with length 8, 8 and 5. The quantity in Column B is greater

Column A Column B The two quantities are equal

The average The average The relationship cannot be


(arithmetic mean) (arithmetic mean) determined from the information
measure of the 3 measure of the 3 given
angles of triangle angles of triangle
A. B.

EndPractice
End Practice and
and See
See Results
Results Question 328 of 737 Submit Answer
Submit Answer Next Question

Math 
Calculator  Flag 
0:02

Choose the option that best answers the question.

$240
After her birthday, Fiona had D dollars in
gift money.  She spent 3/5 of this on an
$300
electric skateboard. The next day, she
spent 1/3 of what was left on a movie $320
passcard, and then finally put the last $80
in the bank.  If she made no other $360
purchases with that money, then what was
the value of D? $480

EndPractice
End Practice and
and See
See Results
Results Question 329 of 737 Answer
Next Question
Submit Answer
Submit

Math 
Calculator  Flag 
0:03

Choose the correct statement.

The quantity in Column A is greater


2n + 2n + 2n + 2n = 4n + 3

The quantity in Column B is greater


Column A Column B

The two quantities are equal


n 4

The relationship cannot be


determined from the information
given

EndPractice
End Practice and
and See
See Results
Results Question 330 of 737 Answer
Next Question
Submit Answer
Submit

Math 
Calculator  Flag 
0:03

Choose the option that best answers the question.

19/
If the numbers 19/36, 5/11, 12/25, 6/11, and 36
8/ were arranged from least to greatest,
18 12
/25
which number would be in the middle?
6/
11

5/
11

8/
18

EndPractice
End Practice and
and See
See Results
Results Question 331 of 737 Answer
Next Question
Submit Answer
Submit

Math 
Calculator  Flag 
0:02

Consider each of the choices separately and select all that apply.

6
For positive integers P, Q, and R, 1 ≤ P < Q
< 15.  P is odd, Q is even, and P + Q = R.
11
Which of the following could be the value of
the sum P + Q + R?    15
Indicated all such numbers.
18

20

37

46

58

EndPractice
End Practice and
and See
See Results
Results Question 332 of 737 Answer
Next Question
Submit Answer
Submit


Calculator  Flag 
0:03
Math

Enter the answer in the blank.

Give your answer to the nearest 0.1


If x and y are positive numbers and 
​ x
√​x2​ − y2​ = 3y − x , what is the value of  ​ ?
y

EndPractice
End Practice and
and See
See Results
Results Question 333 of 737 Submit Answer
Submit Answer Next Question

Math 
Calculator  Flag 
0:06

Choose the correct statement.

A number, x, is randomly selected from the The quantity in Column A is greater


integers from 42 to 92 inclusive.
The quantity in Column B is greater

Column A Column B
The two quantities are equal
The probability The probability that
The relationship cannot be
that x is odd. x is even.
determined from the information
given

EndPractice
End Practice and
and See
See Results
Results Question 334 of 737 Submit Answer
Submit Answer Next Question

Math 
Calculator  Flag 
0:03
Choose the option that best answers the question.

For the 15 colleges shown, the graph


supports which of the following
statements

I. tuition income is positively correlated


with student enrollment

II. investment income is negatively


correlated with student enrollment

III. all colleges with over 20,000


In the diagram above, each of fifteen
students have less than $500 million in
private colleges is represented by a dot
investment income
and an X on a vertical line.  The X indicates
the college's annual income from tuition in I only
2008.  The dot, above or below on the
same dashed vertical line, indicates the I and II only
college's annual income in 2008 from
investments such as endowments. The I and III only
base of the vertical dashed line indicates
II and III only
the number of students at that college in
2008.    I, II, and III

EndPractice
End Practice and
and See
See Results
Results Question 335 of 737 Submit Answer
Submit Answer Next Question

Math 
Calculator  Flag 
0:02

Choose the option that best answers the question.

The college that is drawing the most


investment income in 2008 takes in
approximately how much in mean total
income per student in 2008? (Total
income = tuition + investments)
$5,600

$28,000

In the diagram above, each of fifteen $36,000


private colleges is represented by a dot
and an X on a vertical line.  The X indicates $56,000
the college's annual income from tuition in
2008.  The dot, above or below on the $230,000
same dashed vertical line, indicates the
college's annual income in 2008 from
investments such as endowments. The
base of the vertical dashed line indicates
the number of students at that college in
2008.   

EndPractice
End Practice and
and See
See Results
Results Question 336 of 737 Submit Answer
Submit Answer Next Question

Math 
Calculator  Flag 
0:02

Choose the option that best answers the question.

If the tuition income at a college


exceeds its investment income, then
that college is said to be "tuition
driven." How many colleges shown
here were tuition driven in 2008?

four

five
In the diagram above, each of fifteen six
private colleges is represented by a dot
and an X on a vertical line.  The X indicates seven
the college's annual income from tuition in
2008.  The dot, above or below on the eight
same dashed vertical line, indicates the
college's annual income in 2008 from
investments such as endowments. The
base of the vertical dashed line indicates
the number of students at that college in
2008.   

EndPractice
End Practice and
and See
See Results
Results Question 337 of 737 Submit Answer
Submit Answer Next Question

Math 
Calculator  Flag 
0:02

Choose the option that best answers the question.

The college shown with the highest


tuition income in 2008 has how much
investment income?

$190 million

$340 million

$590 million
In the diagram above, each of fifteen $610 million
private colleges is represented by a dot
and an X on a vertical line.  The X indicates $640 million
the college's annual income from tuition in
2008.  The dot, above or below on the
same dashed vertical line, indicates the
college's annual income in 2008 from
investments such as endowments. The
base of the vertical dashed line indicates
the number of students at that college in
2008.   

EndPractice
End Practice and
and See
See Results
Results Question 338 of 737 Submit Answer
Submit Answer Next Question

Math 
Calculator  Flag 
0:03

Choose the option that best answers the question.

For how many colleges shown is the


investment income in 2008 more than
double the same college's tuition
income in 2008?

none

one
two
In the diagram above, each of fifteen
private colleges is represented by a dot three
and an X on a vertical line.  The X indicates
the college's annual income from tuition in four
2008.  The dot, above or below on the
same dashed vertical line, indicates the
college's annual income in 2008 from
investments such as endowments. The
base of the vertical dashed line indicates
the number of students at that college in
2008.   

EndPractice
End Practice and
and See
See Results
Results Question 339 of 737 Submit Answer
Submit Answer Next Question

Math 
Calculator  Flag 
0:03

Choose the option that best answers the question.

The college with the highest 2008


mean investment income per student
enrolled generates how much annual
tuition income in 2008.

$160 million

$360 million
$450 million
In the diagram above, each of fifteen
private colleges is represented by a dot $560 million
and an X on a vertical line.  The X indicates
the college's annual income from tuition in $620 million
2008.  The dot, above or below on the
same dashed vertical line, indicates the
college's annual income in 2008 from
investments such as endowments. The
base of the vertical dashed line indicates
the number of students at that college in
2008.   

EndPractice
End Practice and
and See
See Results
Results Question 340 of 737 Submit Answer
Submit Answer Next Question
Math 
Calculator  Flag 
0:02

Choose the option that best answers the question.

1550
Appleton’s population is 400 greater than
Berryville’s population. If Berryville’s
1650
population were reduced by 900 people,
then Appleton’s population would be 3 1750
times as large as Berryville’s population.
What is Berryville’s current population? 1850

1950

EndPractice
End Practice and
and See
See Results
Results Question 341 of 737 Submit Answer
Submit Answer Next Question

Math 
Calculator  Flag 
0:03

Choose the correct statement.

The quantity in Column A is greater

The quantity in Column B is greater

The two quantities are equal

The relationship cannot be


determined from the information
DC = AB = 9 given

DB < 3
Column A Column B 
Perimeter of triangle ABC 36

EndPractice
End Practice and
and See
See Results
Results Question 342 of 737 Answer
Next Question
Submit Answer
Submit

Math 
Calculator  Flag 
0:03

Choose the option that best answers the question.

RW + Q
When Q is divided by W, the quotient is R
and the remainder is E. Which of the
RW − Q
following expressions is equal to E?
Q − RW

QW − R

Q
/RW

EndPractice
End Practice and
and See
See Results
Results Question 343 of 737 Answer
Next Question
Submit Answer
Submit

Math 
Calculator  Flag 
0:02

Choose the correct statement.

The quantity in Column A is greater


The quantity in Column B is greater

The two quantities are equal

The relationship cannot be


Column A Column B determined from the information
given
AB 8

EndPractice
End Practice and
and See
See Results
Results Question 344 of 737 Answer
Next Question
Submit Answer
Submit

Math 
Calculator  Flag 
0:02

Choose the option that best answers the question.

16
From a total of 5 boys and 4 girls, how
many 4-person committees can be
24
selected if the committee must have
exactly 2 boys and 2 girls? 60

120

240

EndPractice
End Practice and
and See
See Results
Results Question 345 of 737 Answer
Next Question
Submit Answer
Submit

Math 
Calculator  Flag 
0:03
Consider each of the choices separately and select all that apply.

The ratio of DE to EF = 1 : √2
Two sides of triangle DEF are equal to 3.
Which of the following, taken alone, would
The sum of angles DEF and EFD is
be sufficient in finding the area of
triangle
135 degrees
DEF?

Indicate all such


statements The sum of angles DEF and FDE is
90 degrees

EndPractice
End Practice and
and See
See Results
Results Question 346 of 737 Answer
Next Question
Submit Answer
Submit

Math 
Calculator  Flag 
0:02

Enter the answer in the blank.

When the decimal point of positive number


N is moved 2 places to the left, the result is
6
equal to ​ .  What is the value of N?
N −1

EndPractice
End Practice and
and See
See Results
Results Question 347 of 737 Answer
Next Question
Submit Answer
Submit

Math 
Calculator  Flag 
0:03

Choose the correct statement.

The quantity in Column A is greater


Column A Column B
The quantity in Column B is greater
The number of The number of
prime numbers prime numbers The two quantities are equal
divisible by 13 divisible by 2
The relationship cannot be
determined from the information
given

EndPractice
End Practice and
and See
See Results
Results Question 348 of 737 Answer
Next Question
Submit Answer
Submit

Math 
Calculator  Flag 
0:03

Choose the option that best answers the question.

72
Working alone, pump A can empty a pool
in 3 hours. Working alone, pump B can
75
empty the same pool in 2 hours. Working
together, how many minutes will it take 84
pump A and pump B to empty the pool?
96

108

EndPractice
End Practice and
and See
See Results
Results Question 349 of 737 Answer
Next Question
Submit Answer
Submit

Math 
Calculator  Flag 
0:03

Choose the correct statement.


The quantity in Column A is greater

The quantity in Column B is greater

The two quantities are equal

The relationship cannot be


Column A Column B determined from the information
given
Area of triangle 16

EndPractice
End Practice and
and See
See Results
Results Question 350 of 737 Submit Answer
Submit Answer Next Question

Math 
Calculator  Flag 
0:03

Choose the option that best answers the question.

4k
2k years ago Frank was 3k years old. In k
years Frank's age, in years, will be
5k

6k

7k

8k

EndPractice
End Practice and
and See
See Results
Results Question 351 of 737 Submit Answer
Submit Answer Next Question


Calculator  Flag 
0:03
Math

Choose the correct statement.

The greatest prime factor of 40,002 is x The quantity in Column A is greater

The greatest prime factor of 80,004 is y The quantity in Column B is greater

Column A Column B The two quantities are equal

x y The relationship cannot be


determined from the information
given

EndPractice
End Practice and
and See
See Results
Results Question 352 of 737 Submit Answer
Submit Answer Next Question

Math 
Calculator  Flag 
0:02

Choose the option that best answers the question.

5
If the average (arithmetic mean) of x, y and
15 is 9, and the average of x, 2y and 2 is 7,
6
then y =
7

EndPractice
End Practice and
and See
See Results
Results Question 353 of 737 Submit Answer
Submit Answer Next Question
Math 
Calculator  Flag 
0:06

Consider each of the choices separately and select all that apply.

If w is even, then x must be even.


If x and y are integers, and w=x2y+x+3y,
which of the following statements must be
If x is odd, then w must be odd.
true?

Indicate all such statements. If y is odd, then w must be odd.

If w is odd, then y must be odd.

EndPractice
End Practice and
and See
See Results
Results Question 354 of 737 Submit Answer
Submit Answer Next Question

Math 
Calculator  Flag 
0:02

Enter the answer in the blank.

Give your answer to the nearest 0.01


The figure shows the graph of the equation
y = k − x2, where k is a constant. If the area
of triangle ABC is 1/8, what is the value of
k?
EndPractice
End Practice and
and See
See Results
Results Question 355 of 737 Submit Answer
Submit Answer Next Question

Math 
Calculator  Flag 
0:03

Choose the correct statement.

An office has 6 employees. The manager The quantity in Column A is greater


must create a committee consisting of 3
The quantity in Column B is greater
employees.
The two quantities are equal
Column A Column
B The relationship cannot be
determined from the information
Number of different 40
given
committees possible.

EndPractice
End Practice and
and See
See Results
Results Question 356 of 737 Submit Answer
Submit Answer Next Question

Math 
Calculator  Flag 
0:02

Choose the option that best answers the question.

10! − 8! 232
​ =
7!
352

472
552

712

EndPractice
End Practice and
and See
See Results
Results Question 357 of 737 Answer
Next Question
Submit Answer
Submit

Math 
Calculator  Flag 
0:03

Choose the correct statement.

The revenue generated by Company X is The quantity in Column A is greater


divided between Doug and Moira in a 6 to 5
The quantity in Column B is greater
ratio respectively.
The two quantities are equal
Column A Column
B The relationship cannot be
determined from the information
Moira's share when the $7900
given
revenue generated by
Company X is $15,700

EndPractice
End Practice and
and See
See Results
Results Question 358 of 737 Answer
Next Question
Submit Answer
Submit

Math 
Calculator  Flag 
0:02

Choose the option that best answers the question.

2.4 x 1050
​ ) ∗ (8 × 1030
(3 × 1020 ​ )=

2.4 x 1051

2.4 x 1060

2.4 x 1061

2.4 x 10301

EndPractice
End Practice and
and See
See Results
Results Question 359 of 737 Submit Answer
Submit Answer Next Question

Math 
Calculator  Flag 
0:35

Choose the correct statement.

A certain taxi charges $0.85 for the first ½ The quantity in Column A is greater
mile and $0.25 for every ½ mile after that.
The quantity in Column B is greater
The total cost of a trip was $8.85
The two quantities are equal
Column A Column B
The relationship cannot be
The trip's distance in miles 16 determined from the information
given

EndPractice
End Practice and
and See
See Results
Results Question 360 of 737 Submit Answer
Submit Answer Next Question
Math 
Calculator  Flag 
0:06

Choose the option that best answers the question.

3 18
In the Antares Corporation, ​  of the
7
managers are female. If there are 42 24
female managers, how many managers in
total are there? 60

66

98

EndPractice
End Practice and
and See
See Results
Results Question 361 of 737 Submit Answer
Submit Answer Next Question

Math 
Calculator  Flag 
0:02

Consider each of the choices separately and select all that apply.

100
The Sargon Corporation offers an optional
stock-option buy-in program to its
200
employees. Of the employees with salaries
greater than or equal to $100,000, 85% 350
choose to participate in this plan.  Of the
employees with salaries less than 460
$100,000, 77% choose to participate in this
plan. Which of the following could be the 525
total number of employees?
640
Indicate all possible values for the number
of employees. 750 
880

EndPractice
End Practice and
and See
See Results
Results Question 362 of 737 Answer
Next Question
Submit Answer
Submit

Math 
Calculator  Flag 
0:05

Enter the answer in the blank.

x2 – y2 < 8
x+y>3

If x and y are integers in the above


inequalities and 0 < y < x, what is the
greatest possible value of x?

EndPractice
End Practice and
and See
See Results
Results Question 363 of 737 Answer
Next Question
Submit Answer
Submit

Math 
Calculator  Flag 
0:03

Choose the correct statement.

The quantity in Column A is greater

The quantity in Column B is greater

The two quantities are equal


Column A Column B The relationship cannot be
determined from the information
a+b+c+d+e+f+g 1080 given

EndPractice
End Practice and
and See
See Results
Results Question 364 of 737 Answer
Next Question
Submit Answer
Submit

Math 
Calculator  Flag 
0:03

Enter the answer in the blank.

For years from 2004 onward, for how


many years shown on the chart was the
unemployment rate higher than it was
in each of the previous two years?

EndPractice
End Practice and
and See
See Results
Results Question 365 of 737 Answer
Next Question
Submit Answer
Submit

Math 
Calculator  Flag 
0:05

Choose the option that best answers the question.

The percent increase in unemployment


rate from 2008 to 2009 is approximately
3.5%

12.6%

23.7%

37.5%

59.9%

EndPractice
End Practice and
and See
See Results
Results Question 366 of 737 Answer
Next Question
Submit Answer
Submit

Math 
Calculator  Flag 
0:02

Choose the option that best answers the question.

The US unemployment rate in 2007


was approximately

3.5%

4.6%

5.2%

5.8%

7.2%

EndPractice
End Practice and
and See
See Results
Results Question 367 of 737 Answer
Next Question
Submit Answer
Submit
Math 
Calculator  Flag 
0:02

Choose the option that best answers the question.

120
A certain restaurant offers 8 different
salads, 5 different main courses, 6 different
240
desserts. If customers choose one salad,
one main course and two different desserts 480
for their meal, how many different meals
are possible? 600

1200

EndPractice
End Practice and
and See
See Results
Results Question 368 of 737 Answer
Next Question
Submit Answer
Submit

Math 
Calculator  Flag 
0:02

Choose the correct statement.

Cam is 20 percent taller than Bea, and Bea The quantity in Column A is greater
is 20 percent taller than Ann.
The quantity in Column B is greater

Column A Column B
The two quantities are equal
Cam’s height Bea’s height minus
The relationship cannot be
minus Bea’s height. Ann’s height.
determined from the information
given

EndPractice
End Practice and
and See
See Results
Results Question 369 of 737 Answer
Next Question
Submit Answer
Submit
Math 
Calculator  Flag 
0:02

Choose the option that best answers the question.

4
If the average (arithmetic mean) of five
consecutive negative integers is 2k – 1,
4k
what is the difference between the greatest
and least of the five integers? 4k + 4

4 - 4k

4k2 - 4k

EndPractice
End Practice and
and See
See Results
Results Question 370 of 737 Answer
Next Question
Submit Answer
Submit

Math 
Calculator  Flag 
0:02

Choose the correct statement.

The sum of 5 consecutive even integers is The quantity in Column A is greater


0.
The quantity in Column B is greater

Column A Column B
The two quantities are equal
The product of the 5 integers 0
The relationship cannot be
determined from the information
given

EndPractice
End Practice and
and See
See Results
Results Question 371 of 737 Answer
Next Question
Submit Answer
Submit
Math 
Calculator  Flag 
0:03

Choose the option that best answers the question.

12
In the xy-coordinate system, line k has y- −​
5
intercept 12 and an x-intercept greater than
zero. If the area of the triangular region 6
−​
5
enclosed by line k and the two axes is 30,
what is the slope of line k? 6

5
3

2
12

5

EndPractice
End Practice and
and See
See Results
Results Question 372 of 737 Answer
Next Question
Submit Answer
Submit

Math 
Calculator  Flag 
0:02

Consider each of the choices separately and select all that apply.

Week 3
Edward and Klara are both nurses who are
good friends with each other.  Edward
Week 4
works at Brigade Hospital, where, in every
10 day cycle, he works the first 7 days and Week 5
has three days off.  Klara works at Cedar
Mountain Hospital, where, in every 7 day Week 6
cycle, she works the first 5 days, and has 2
days off.  Assume that both start their Week 7
respective cycles on Day 1 of Week 1. In
Week 8
which weeks do they have the same two
days off?  
Week 9
Indicate all weeks in which they have the
same two days off. Week 10

EndPractice
End Practice and
and See
See Results
Results Question 373 of 737 Answer
Next Question
Submit Answer
Submit

Math 
Calculator  Flag 
0:26

Enter the answer in the blank.

Square ABCD has side length of 4.  Point E


is the midpoint of AD and the center of the
semicircular arc from point F to point G.  If
the area of the shaded region is π K − 16,
what is the value of K? 

EndPractice
End Practice and
and See
See Results
Results Question 374 of 737 Answer
Next Question
Submit Answer
Submit

Math 
Calculator  Flag 
0:05
Choose the correct statement.

Cylindrical tank A has radius x and height The quantity in Column A is greater
y.
The quantity in Column B is greater
Cylindrical tank B has radius y and height
x. The two quantities are equal

x = 2y The relationship cannot be


determined from the information
Column A Column B given

Volume of tank A Volume of tank B

EndPractice
End Practice and
and See
See Results
Results Question 375 of 737 Answer
Next Question
Submit Answer
Submit

Math 
Calculator  Flag 
0:03

Choose the option that best answers the question.

2 2/ 5
If xy = 5 and x2 + y2 = 12, then x/y + y/x =

3 1/ 7

5 1/ 3

60

EndPractice
End Practice and
and See
See Results
Results Question 376 of 737 Answer
Next Question
Submit Answer
Submit
Math 
Calculator  Flag 
0:28

Choose the correct statement.

The quantity in Column A is greater


Column A Column B
The quantity in Column B is greater
Perimeter of Length of one side of
square with rectangle with The two quantities are equal
sides length 5 perimeter 40
The relationship cannot be
determined from the information
given

EndPractice
End Practice and
and See
See Results
Results Question 377 of 737 Answer
Next Question
Submit Answer
Submit

Math 
Calculator  Flag 
0:02

Consider each of the choices separately and select all that apply.

Which BMI groups have


The graph below shows the Body Mass representatives on this graph with BMR
Index (BMI) and Basal Metabolic Rate > 1600 kcal/day?
(BMR) of fifteen males between the ages of
43 and 65. Severely underweight

Underweight

Normal

Overweight

Obese
EndPractice
End Practice and
and See
See Results
Results Question 378 of 737 Answer
Next Question
Submit Answer
Submit

Math 
Calculator  Flag 
0:03

Choose the option that best answers the question.

The individual on this chart with the


The graph below shows the Body Mass highest BMI has a BMR of
Index (BMI) and Basal Metabolic Rate approximately
(BMR) of fifteen males between the ages of
43 and 65. 1204

1444

1563

1702

1853

EndPractice
End Practice and
and See
See Results
Results Question 379 of 737 Answer
Next Question
Submit Answer
Submit

Math 
Calculator  Flag 
0:02

Enter the answer in the blank.

The trendline on the graph indicates,


The graph below shows the Body Mass for a given BMI, what the expected
Index (BMI) and Basal Metabolic Rate BMR would be for that individual. In this
(BMR) of fifteen males between the ages of group, how many individuals have a
43 and 65. BMR higher than predicted by this
trendline?

EndPractice
End Practice and
and See
See Results
Results Question 380 of 737 Answer
Next Question
Submit Answer
Submit
Magoosh

Math 
Calculator  Flag 
0:03

Choose the option that best answers the question.

9%
Two
sweaters are originally the same price.
Both are discounted 10%. Then one of
the
10%
sweaters is discounted an additional 10%.
By approximately what percent
would the 11%
price of the cheaper of the two sweaters
have to be increased so that
the sweaters 15%
once again sell for the same
price? 
20%

EndPractice
End Practice and
and See
See Results
Results Question 381 of 737 Answer
Next Question
Submit Answer
Submit

Math 
Calculator  Flag 
0:06

Choose the correct statement.

The quantity in Column A is greater


Column A Column B
The quantity in Column B is greater
0.918/0.919 1
The two quantities are equal

The relationship cannot be


determined from the information
given
EndPractice
Practice and
and See
See Results Question 382 of 737 Answer
Next Question
Submit Answer
End Results Submit

Math 
Calculator  Flag 
0:02

Choose the option that best answers the question.

54
How many multiples of 5 are there between
81 and 358?
55

56

57

58

EndPractice
End Practice and
and See
See Results
Results Question 383 of 737 Answer
Next Question
Submit Answer
Submit

Math 
Calculator  Flag 
0:05

Consider each of the choices separately and select all that apply.

Square S (not shown) has one vertex at


(1, 3) and has an area of 25; one pair of
the sides of square S have a slope of 
3
​ .  Not counting the vertex at (1, 3),
                                   4
square S has three other vertices.  In
which quadrants could any other vertex
of Square S fall? Note that points on
the x- or y-axis are not in any quadrant.
The point (1, 3) is shown in the xy-plane
above.  The quadrants are also shown.  Indicate all such quadrants

Quadrant I

Quadrant II

Quadrant III

Quadrant IV

EndPractice
End Practice and
and See
See Results
Results Question 384 of 737 Answer
Next Question
Submit Answer
Submit

Math 
Calculator  Flag 
0:02

Enter the answer as a fraction. Fractions do not need to be in simplest form

The first term in a certain sequence is 1,


the 2nd term in the sequence is 3, and for
all integers n ≥ 3, the nth term in the
sequence is the sum of the reciprocals of
the first (n − 1) terms in the sequence.
 What is the value of the fifth term in the
sequence?

Give your answer as a fraction.

EndPractice
End Practice and
and See
See Results
Results Question 385 of 737 Answer
Next Question
Submit Answer
Submit

Math 
Calculator  Flag 
0:02

Choose the correct statement.


The decimal r = 2.666666 continues The quantity in Column A is greater
forever in that repeating decimal pattern.
The quantity in Column B is greater
When written as a fraction in lowest terms,
r = a/b, where a and b are positive The two quantities are equal
numbers.
The relationship cannot be
Column A Column B determined from the information
given
a+b 10

EndPractice
End Practice and
and See
See Results
Results Question 386 of 737 Answer
Next Question
Submit Answer
Submit

Math 
Calculator  Flag 
0:02

Choose the option that best answers the question.

20
If y is 80 percent greater than x, then x is
what percent less than y?
25

33 1/3

44 4/9

80

EndPractice
End Practice and
and See
See Results
Results Question 387 of 737 Answer
Next Question
Submit Answer
Submit
Math 
Calculator  Flag 
0:02

Choose the correct statement.

Set A: {0.2, 0.4, 0.6, 0.8} The quantity in Column A is greater

Set B: {2, 4, 6, 8} The quantity in Column B is greater

Column A Column B The two quantities are equal

Standard deviation Standard deviation The relationship cannot be


of set A of set B determined from the information
given

EndPractice
End Practice and
and See
See Results
Results Question 388 of 737 Answer
Next Question
Submit Answer
Submit

Math 
Calculator  Flag 
0:02

Choose the option that best answers the question.

26
A purse contains 5-cent coins and 10-cent
coins worth a total of $1.75. If the 5-cent
27
coins were replaced with 10-cent coins and
the 10-cent coins were replaced with 5-cent 28
coins, the coins would be worth a total of
$2.15. How many coins are in the purse? 29

30

EndPractice
End Practice and
and See
See Results
Results Question 389 of 737 Answer
Next Question
Submit Answer
Submit
Math 
Calculator  Flag 
0:02

Choose the correct statement.

The average (arithmetic mean) weight of The quantity in Column A is greater


18 automobiles is 3500 pounds.
The quantity in Column B is greater
No automobile weighs exactly 3500
pounds. The two quantities are equal

Column A Column B The relationship cannot be


determined from the information
Number of Number of given
automobiles that automobiles that
weigh more than weigh less than
3500 pounds.   3500 pounds.

EndPractice
End Practice and
and See
See Results
Results Question 390 of 737 Answer
Next Question
Submit Answer
Submit

Math 
Calculator  Flag 
0:05

Choose the option that best answers the question.

75π

150π

300π

600π
900π

In the diagram above, arc AB has a central


angle of 135° and an arclength of 15π .
 What is the area of the shaded sector?  

EndPractice
End Practice and
and See
See Results
Results Question 391 of 737 Answer
Next Question
Submit Answer
Submit

Math 
Calculator  Flag 
0:02

Consider each of the choices separately and select all that apply.

1
Among all the students at a certain high ​
2
school, the probability of picking a left-
1 2
handed student is ​ , and the probability of ​
4 3
picking a student who is learning Spanish
2 3
is ​ .  Which of the following could be the

3 4
probability of picking a student who is 5
either left-handed or learning Spanish or ​
6
both?
7

Indicate all such numbers. 8

EndPractice
End Practice and
and See
See Results
Results Question 392 of 737 Answer
Next Question
Submit Answer
Submit
Math 
Calculator  Flag 
0:03

Enter the answer as a fraction. Fractions do not need to be in simplest form

In a class of 40 students, 12 are left-


handed and the other 28 are right-handed.
 If two students are chosen at random,
what’s the probability that one is left-
handed and one is right-handed?

Give your answer as a fraction.   

EndPractice
End Practice and
and See
See Results
Results Question 393 of 737 Answer
Next Question
Submit Answer
Submit

Math 
Calculator  Flag 
0:03

Choose the correct statement.

Set X: {5, 6, 9} The quantity in Column A is greater

Set Y: {0, 1, 4} The quantity in Column B is greater

Column A Column B The two quantities are equal

Standard deviation Standard deviation The relationship cannot be


of set X of set Y determined from the information
given

EndPractice
End Practice and
and See
See Results
Results Question 394 of 737 Answer
Next Question
Submit Answer
Submit
Math 
Calculator  Flag 
0:01

Choose the option that best answers the question.

​ ​ ​
6√​6
(3 + √​6 )2 − (3 − √​6 )2 = 

12√​6

12

18 − 6√​6

18 − 12√​6

EndPractice
End Practice and
and See
See Results
Results Question 395 of 737 Answer
Next Question
Submit Answer
Submit

Math 
Calculator  Flag 
0:03

Choose the correct statement.

The quantity in Column A is greater


The speed of light is approximately 3 × 105
kilometers per second.
The quantity in Column B is greater

Column A Column B
The two quantities are equal

Approximate number of 1.08 × 108


The relationship cannot be
kilometers that light can
determined from the information
travel in 1 hour.
given
EndPractice
End Practice and
and See
See Results
Results Question 396 of 737 Answer
Next Question
Submit Answer
Submit

Math 
Calculator  Flag 
0:03

Choose the option that best answers the question.

1
If 4n + 4n + 4n + 4n = 416 , then n =

12

15

EndPractice
End Practice and
and See
See Results
Results Question 397 of 737 Answer
Next Question
Submit Answer
Submit

Math 
Calculator  Flag 
0:02

Choose the correct statement.

The sales tax at a certain store is 15 The quantity in Column A is greater


percent. The total price of an item,
The quantity in Column B is greater
including sales tax, is $45.
The two quantities are equal
Column A Column
B The relationship cannot be
determined from the information
Price of item excluding sales $39
given
tax

EndPractice
End Practice and
and See
See Results
Results Question 398 of 737 Answer
Next Question
Submit Answer
Submit

Math 
Calculator  Flag 
0:02

Choose the option that best answers the question.

​ ​
If ABCD is a square, what are the (√​3, √​3)
coordinates of C? ​ ​
(√​3, 1 + √​3)
​ ​
(2√​3 , √​3 )
​ ​
(1 + √​3 , √​3)
​ ​
(√​3, 2√​3 )

EndPractice
End Practice and
and See
See Results
Results Question 399 of 737 Answer
Next Question
Submit Answer
Submit

Math 
Calculator  Flag 
0:02

Consider each of the choices separately and select all that apply.

Which of the following statements


The diagram below shows the layout of the
individually provide(s) sufficient
quadrants in the xy-plane.  Line L (not additional information to determine
shown) is in the xy-plane, has a non- whether Line L passes through
negative slope, and passes through Quadrant IV?
Quadrant I.
Indicate all such statements.

Line L has a positive slope

Line L has a positive y-intercept

Line L passes through Quadrant II

Line L passes through Quadrant III

EndPractice
End Practice and
and See
See Results
Results Question 400 of 737 Answer
Next Question
Submit Answer
Submit
Magoosh

Math 
Calculator  Flag 
0:03

Enter the answer in the blank.

A knockoff website requires users to create


a password using letters from the word
MAGOSH. If each password must have at
least 4 letters and no repeated letters are
allowed, how many different passwords are
possible?

EndPractice
End Practice and
and See
See Results
Results Question 401 of 737 Answer
Next Question
Submit Answer
Submit

Math 
Calculator  Flag 
0:02

Choose the correct statement.

The quantity in Column A is greater


Column A Column B
The quantity in Column B is greater
The Twice the length of the
perimeter of diagonal of the same The two quantities are equal
a rectangle rectangle
The relationship cannot be
determined from the information
given

EndPractice
End Practice and
and See
See Results
Results Question 402 of 737 Answer
Next Question
Submit Answer
Submit

Math 
Calculator  Flag 
0:03

Choose the option that best answers the question.

x
2x + y years ago, Roberto was 3x + y years
old. How many years old was Roberto x
4x + 2y
years ago?
5x + 2y

6x + 2y

6x + y

EndPractice
End Practice and
and See
See Results
Results Question 403 of 737 Answer
Next Question
Submit Answer
Submit

Math 
Calculator  Flag 
0:02

Choose the correct statement.

In 12 years, Murray will be 4 times as old The quantity in Column A is greater


as he is now.
The quantity in Column B is greater

Column A Column
The two quantities are equal
B
The relationship cannot be
Number of years until Murray 24
determined from the information
is 8 times as old as he is now
given
EndPractice
End Practice and
and See
See Results
Results Question 404 of 737 Answer
Next Question
Submit Answer
Submit

Math 
Calculator  Flag 
0:02

Choose the option that best answers the question.

3y2
Which of the following is equal to

1
12y16
​ √​​ ​  for all values of y?  3y4
4y16​

6y2

6y4

6y8

EndPractice
End Practice and
and See
See Results
Results Question 405 of 737 Answer
Next Question
Submit Answer
Submit

Math 
Calculator  Flag 
0:03

Choose the correct statement.

Gene is 7 years older than Roberta. The quantity in Column A is greater

6 years ago, Gene was twice as old as The quantity in Column B is greater
Roberta.
The two quantities are equal
Column A Column B
The relationship cannot be
Roberta’s current age 12 determined from the information
given

EndPractice
End Practice and
and See
See Results
Results Question 406 of 737 Answer
Next Question
Submit Answer
Submit

Math 
Calculator  Flag 
0:03

Choose the option that best answers the question.

–3x + 2y = 6
Line k is in the rectangular coordinate
system. If the x-intercept of k is  –2, and
3x + 2y = –6
the y-intercept is 3, which of the following is
an equation of line k? 3x – 2y = 6

2x – 3y = 6

–2x – 3y = 6

EndPractice
End Practice and
and See
See Results
Results Question 407 of 737 Answer
Next Question
Submit Answer
Submit

Math 
Calculator  Flag 
0:02

Consider each of the choices separately and select all that apply.

3
Constantine forms the following hypothesis.
  Let n be any non-negative number that
4
meets the following condition: when n is
divided by 5, the remainder cannot equal 2. 5
 For such values of n, the quantity Q = 97 −
6n is a prime number so long as Q > 0. 6
 Which of the following values of n would
provide a counterexample to this 8
hypothesis?
9
Indicate all such values.
10

EndPractice
End Practice and
and See
See Results
Results Question 408 of 737 Answer
Next Question
Submit Answer
Submit

Math 
Calculator  Flag 
0:02

Enter the answer as a fraction. Fractions do not need to be in simplest form

In basketball, when Monica takes


her first
free throw, she has a 50% chance of
scoring. If she takes an additional throw,
she has a
75% chance of scoring on this
throw if she scored on the immediately
previous
throw, but only a 25% chance of
scoring on this throw if she didn't score on
the immediately previous throw. Suppose
she has to take a set of three free throws in
a row, and she doesn't score on
the first
one. What is the probability
that she scores
at least once on one of the subsequent two
throws?

EndPractice
End Practice and
and See
See Results
Results Question 409 of 737 Answer
Next Question
Submit Answer
Submit
Math 
Calculator  Flag 
0:02

Choose the correct statement.

Yesterday, Carl had 40 percent more CDs The quantity in Column A is greater
than Karen had. Today, Carl gave 20
The quantity in Column B is greater
percent of his CDs to Karen.
The two quantities are equal
Column A Column B
The relationship cannot be
Number of CDs Number of CDs that
determined from the information
that Carl now has Karen now has
given

EndPractice
End Practice and
and See
See Results
Results Question 410 of 737 Answer
Next Question
Submit Answer
Submit

Math 
Calculator  Flag 
0:05

Choose the option that best answers the question.

​ ​
If x and y are both positive then √​72x3​ y16
​ =  8xy8​ √​2x

6xy4​ √​2x

6xy8​ √​2x

6y4​ √​8x

9xy8​ √​8x

EndPractice
End Practice and
and See
See Results
Results Question 411 of 737 Answer
Next Question
Submit Answer
Submit
Math 
Calculator  Flag 
0:03

Choose the correct statement.

The quantity in Column A is greater

The quantity in Column B is greater

The two quantities are equal

The relationship cannot be


determined from the information
AC = BC = 8 given

Column A Column B

x y

EndPractice
End Practice and
and See
See Results
Results Question 412 of 737 Answer
Next Question
Submit Answer
Submit

Math 
Calculator  Flag 
0:05

Choose the option that best answers the question.

k:3
If k:3 is the ratio of the circumference to the
diameter of a circle with radius x, then what
k:6
is the ratio of the circumference to the
diameter of a circle with radius 2x? 2k:3

2k:9
4k:9

EndPractice
End Practice and
and See
See Results
Results Question 413 of 737 Answer
Next Question
Submit Answer
Submit

Math 
Calculator  Flag 
0:02

Choose the correct statement.

Two sides of a triangle have lengths 7 and The quantity in Column A is greater
4
The quantity in Column B is greater

Column A Column
The two quantities are equal
B
The relationship cannot be
Length of third side of 3
determined from the information
triangle
given

EndPractice
End Practice and
and See
See Results
Results Question 414 of 737 Answer
Next Question
Submit Answer
Submit

Math 
Calculator  Flag 
0:02

Choose the option that best answers the question.

x2​ 2
If f(x) = 12 − ​  and f(2k) = 2k, what is one
2
possible value for k? 3
4

EndPractice
End Practice and
and See
See Results
Results Question 415 of 737 Answer
Next Question
Submit Answer
Submit

Math 
Calculator  Flag 
0:03

Consider each of the choices separately and select all that apply.

13.5
{N, 11, 13, 15, 18, 19}

In the list above, N is some positive integer. 14


 Which of the following would be, for some
value of N, a possible median of the list? 14.5

Indicate all possible values for the median. 15

15.5

16

16.5

17

17.5

EndPractice
End Practice and
and See
See Results
Results Question 416 of 737 Answer
Next Question
Submit Answer
Submit


Calculator  Flag 
0:03
Math

Enter the answer as a fraction. Fractions do not need to be in simplest form

                


Equilateral triangle ABC has an area of √​3 .
 If the shaded region has an area of π K − 

√​3 , what is the value of K?

Give your answer as a fraction.   

EndPractice
End Practice and
and See
See Results
Results Question 417 of 737 Answer
Next Question
Submit Answer
Submit

Math 
Calculator  Flag 
0:03

Choose the correct statement.

The quantity in Column A is greater

The quantity in Column B is greater

The two quantities are equal

The relationship cannot be


Each edge of the above cube has length 1.
determined from the information
given
Column A Column B

Length of diagonal AB √​3​
EndPractice
End Practice and
and See
See Results
Results Question 418 of 737 Answer
Next Question
Submit Answer
Submit

Math 
Calculator  Flag 
0:02

Choose the option that best answers the question.

-480
If a, b, c and d are different integers
between -6 and 10 inclusive, what is the
-720
least possible value of the product abcd?
-1200

-3600

-4320

EndPractice
End Practice and
and See
See Results
Results Question 419 of 737 Answer
Next Question
Submit Answer
Submit

Math 
Calculator  Flag 
0:02

Choose the correct statement.

wxy ≠ 0 The quantity in Column A is greater

3w = 4x, 4y = 3x The quantity in Column B is greater

Column A Column B The two quantities are equal


The ratio of w to y 1 The relationship cannot be
determined from the information
given

EndPractice
End Practice and
and See
See Results
Results Question 420 of 737 Answer
Next Question
Submit Answer
Submit
Magoosh

Math 
Calculator  Flag 
0:02

Choose the option that best answers the question.

In the figure below, ∠ADE = 60°, ∠EFC = 75

40°, and ∠ DAE = 55°.  If AB is parallel


85
to CD, what is the value of x?

Note: Figure not drawn to scale 95

105

115

EndPractice
End Practice and
and See
See Results
Results Question 421 of 737 Answer
Next Question
Submit Answer
Submit

Math 
Calculator  Flag 
0:02

Choose the correct statement.

The quantity in Column A is greater


v+w–4=x+y–5

The quantity in Column B is greater


Column A Column B
The two quantities are equal
Average (arithmetic Average 
mean) of v and w (arithmetic mean)
The relationship cannot be
of x and y
determined from the information
given

EndPractice
End Practice and
and See
See Results
Results Question 422 of 737 Answer
Next Question
Submit Answer
Submit

Math 
Calculator  Flag 
0:05

Choose the option that best answers the question.

29
If 2k = 3, then 23k+2 =

54

81

83

108

EndPractice
End Practice and
and See
See Results
Results Question 423 of 737 Answer
Next Question
Submit Answer
Submit

Math 
Calculator  Flag 
0:02

Consider each of the choices separately and select all that apply.

Which of the following is equal to 8 24


​  ? 296

Indicate all possible values. 436

1212

1618

248​

3215

EndPractice
End Practice and
and See
See Results
Results Question 424 of 737 Answer
Next Question
Submit Answer
Submit

Math 
Calculator  Flag 
0:02

Enter the answer in the blank.

In the
triangle above, BD is perpendicular
to AC. If CD = 20, BC = 25, and AB = 17,
what is the length of AD?

EndPractice
End Practice and
and See
See Results
Results Question 425 of 737 Answer
Next Question
Submit Answer
Submit

Math 
Calculator  Flag 
0:01
Choose the correct statement.

The quantity in Column A is greater


For positive numbers, X, Y ,  and 
Z 35 Z 35
Z, ​ = ​  and  ​ = ​ The quantity in Column B is greater
Y 6 X 8

Column A Column B The two quantities are equal

X Y The relationship cannot be


determined from the information
given

EndPractice
End Practice and
and See
See Results
Results Question 426 of 737 Answer
Next Question
Submit Answer
Submit

Math 
Calculator  Flag 
0:02

Choose the option that best answers the question.

85​ × 46​ -22


If  ​ = 321−n
​  then n =
16n​
-11

11

22

EndPractice
End Practice and
and See
See Results
Results Question 427 of 737 Answer
Next Question
Submit Answer
Submit
Math 
Calculator  Flag 
0:02

Choose the correct statement.

The quantity in Column A is greater


Column A Column B
The quantity in Column B is greater
101 6001
​ ​
150 9000
The two quantities are equal

The relationship cannot be


determined from the information
given

EndPractice
End Practice and
and See
See Results
Results Question 428 of 737 Answer
Next Question
Submit Answer
Submit

Math 
Calculator  Flag 
0:02

Choose the option that best answers the question.

If the hypotenuse of an isosceles right 4


triangle has length of 8, then the area of ​
4√​2
the triangle is
8

8√​2

16

EndPractice
End Practice and
and See
See Results
Results Question 429 of 737 Answer
Next Question
Submit Answer
Submit
Math 
Calculator  Flag 
0:02

Choose the correct statement.

The quantity in Column A is greater


Column A Column
B The quantity in Column B is greater

Sum of integers from 1 to 40 800 The two quantities are equal


inclusive
The relationship cannot be
determined from the information
given

EndPractice
End Practice and
and See
See Results
Results Question 430 of 737 Answer
Next Question
Submit Answer
Submit

Math 
Calculator  Flag 
0:05

Choose the option that best answers the question.

–16
If 2 ÷ 2 ÷ 2 ÷ 2 ÷ 2 = 2x, then x =

–8

–5

–4

–3

EndPractice
End Practice and
and See
See Results
Results Question 431 of 737 Answer
Next Question
Submit Answer
Submit
Math 
Calculator  Flag 
0:03

Consider each of the choices separately and select all that apply.

3
For which values of x is 5x + 9 < 7x + 6?  ​
2
Indicate all possible values of x. 4

3
7

4
8

5
10

7
13

8
16

11
26

17

EndPractice
End Practice and
and See
See Results
Results Question 432 of 737 Answer
Next Question
Submit Answer
Submit

Math 
Calculator  Flag 
0:02

Enter the answer in the blank.

Luke drives the first 300 miles of


a trip at
60 miles an hour. How fast
does he have to
drive, in miles per hour, on the final 200
miles of the trip if
the total time of the trip is
to equal 7 hours? 

EndPractice
End Practice and
and See
See Results
Results Question 433 of 737 Answer
Next Question
Submit Answer
Submit

Math 
Calculator  Flag 
0:03

Choose the correct statement.

The quantity in Column A is greater


Column A Column B
The quantity in Column B is greater
The circumference of The perimeter of
a circle with a square with The two quantities are equal
​ ​
diameter √​5​0 side √​5​0
The relationship cannot be
determined from the information
given

EndPractice
End Practice and
and See
See Results
Results Question 434 of 737 Answer
Next Question
Submit Answer
Submit

Math 
Calculator  Flag 
0:03

Choose the option that best answers the question.

190
What is the average (arithmetic mean) of
all multiples of 10 from 10 to 400 inclusive?
195
200

205

210

EndPractice
End Practice and
and See
See Results
Results Question 435 of 737 Answer
Next Question
Submit Answer
Submit

Math 
Calculator  Flag 
0:03

Choose the correct statement.

5 The quantity in Column A is greater


For positive numbers P and Q, ​ P = Q
3
The quantity in Column B is greater
Column A Column B
The two quantities are equal
P (0.55)Q
The relationship cannot be
determined from the information
given

EndPractice
End Practice and
and See
See Results
Results Question 436 of 737 Answer
Next Question
Submit Answer
Submit

Math 
Calculator  Flag 
0:02

Choose the option that best answers the question.

10,000
What is the sum of all integers from 45 to
155 inclusive?
10,100

11,000

11,100

13,200

EndPractice
End Practice and
and See
See Results
Results Question 437 of 737 Answer
Next Question
Submit Answer
Submit

Math 
Calculator  Flag 
0:02

Choose the correct statement.

The average (arithmetic mean) of x , y and The quantity in Column A is greater


15 is 9.
The quantity in Column B is greater

Column A Column B
The two quantities are equal
Average of x and y 6
The relationship cannot be
determined from the information
given

EndPractice
End Practice and
and See
See Results
Results Question 438 of 737 Answer
Next Question
Submit Answer
Submit

Math 
Calculator  Flag 
0:02

Choose the option that best answers the question.


1 3
A certain barrel is  ​  full. When k liters of ​ k
5 8
liquid are added to the barrel, it becomes  7
2 ​ k
​  full. In terms of k, what is the capacity of 15
3
the barrel, in liters? 15
​ k
7
7
​ k
3
8
​ k
3

EndPractice
End Practice and
and See
See Results
Results Question 439 of 737 Answer
Next Question
Submit Answer
Submit

Math 
Calculator  Flag 
0:03

Consider each of the choices separately and select all that apply.

21
a, b and c are positive integers. If b equals
the square root of a, and if c equals the
30
sum of a and b, which of the following
could be the value of c? 45
Indicate all such values.
72

100

331

EndPractice
End Practice and
and See
See Results
Results Question 440 of 737 Answer
Next Question
Submit Answer
Submit
Magoosh

Math 
Calculator  Flag 
0:03

Enter the answer in the blank.

A machine is making thermometers at a


rate of 135 every 18 minutes. How many
thermometers will this machine make in an
hour?

EndPractice
End Practice and
and See
See Results
Results Question 441 of 737 Answer
Next Question
Submit Answer
Submit

Math 
Calculator  Flag 
0:03

Choose the correct statement.

The first six terms of an infinite sequence The quantity in Column A is greater
are 2, 4, 4, 3, 7, 5 and these six terms
The quantity in Column B is greater
repeat in the same order. (e.g., 2, 4, 4, 3, 7,
5, 2, 4, 4, 3, 7, 5 . . . ) The two quantities are equal

Column A Column B The relationship cannot be


determined from the information
Term 49 Term 50
given

EndPractice
End Practice and
and See
See Results
Results Question 442 of 737 Answer
Next Question
Submit Answer
Submit

Math 
Calculator  Flag 
0:02

Choose the option that best answers the question.


If ABCD is a rectangle, BC = x and AB = πx√​3
2x, then the circumference of the circle, in ​
πx√​5
terms of x, is

π√​3x

π√​5x

5x2​ π

EndPractice
End Practice and
and See
See Results
Results Question 443 of 737 Answer
Next Question
Submit Answer
Submit

Math 
Calculator  Flag 
0:02

Choose the option that best answers the question.

y y y
​ +​ =
1+x 1−x
y

x
2y

1 − x2​
2xy

1 − x2​
y − 2x

1 − x2​

EndPractice
End Practice and
and See
See Results
Results Question 444 of 737 Answer
Next Question
Submit Answer
Submit

Math 
Calculator  Flag 
0:03

Choose the option that best answers the question.

2
If 1/x + 1/3 + 1/4 = 1, then x =
24

11
12

5

EndPractice
End Practice and
and See
See Results
Results Question 445 of 737 Answer
Next Question
Submit Answer
Submit

Math 
Calculator  Flag 
0:05

Consider each of the choices separately and select all that apply.

1296
Which of the following numbers is divisible
by 36? 
2160
Indicate all possible values.
3438
4608

5346

6144

7000

8244

EndPractice
End Practice and
and See
See Results
Results Question 446 of 737 Answer
Next Question
Submit Answer
Submit

Math 
Calculator  Flag 
0:02

Enter the answer in the blank.

Melanie invests P dollars in a certificate of


deposit that increases in value at 6%
compounding annually each year for five
years.  At the end of five years, the
certificate is worth $1000. What was the
value of P?

Give your answer to the nearest 0.01


dollars.  

EndPractice
End Practice and
and See
See Results
Results Question 447 of 737 Answer
Next Question
Submit Answer
Submit

Math 
Calculator  Flag 
0:03
Choose the option that best answers the question.

If ∠CF D = 85∘​ , then what is the value of x? 125°

Note: Figure not drawn to scale 130°

135°

140°

145°

EndPractice
End Practice and
and See
See Results
Results Question 448 of 737 Answer
Next Question
Submit Answer
Submit

Math 
Calculator  Flag 
0:02

Choose the option that best answers the question.

3
If 92x+5 = 273x-10, then x =

12

15

EndPractice
End Practice and
and See
See Results
Results Question 449 of 737 Answer
Next Question
Submit Answer
Submit
Math 
Calculator  Flag 
0:02

Choose the option that best answers the question.

​ 4
2
If 3 + √​ ​ x 2​ + 1 = 8 , what is the value of x? 
3
5

12

EndPractice
End Practice and
and See
See Results
Results Question 450 of 737 Answer
Next Question
Submit Answer
Submit

Math 
Calculator  Flag 
0:06

Consider each of the choices separately and select all that apply.

3x 1 10
For which values of x is    ​ − ​ < 0?
x2​ + 2x 5
11
Indicate all possible values of x.
12

13

14

15

16
EndPractice
End Practice and
and See
See Results
Results Question 451 of 737 Answer
Next Question
Submit Answer
Submit

Math 
Calculator  Flag 
0:02

Enter the answer as a fraction. Fractions do not need to be in simplest form

A weighted die, numbered one


through six,
has a probability of 1/4 of rolling a six. If
this die is rolled three times, and each
roll
is independent, what is the probability of
rolling at least two sixes?

EndPractice
End Practice and
and See
See Results
Results Question 452 of 737 Answer
Next Question
Submit Answer
Submit

Math 
Calculator  Flag 
0:02

Choose the option that best answers the question.

(5 58​ ) 1 14​
​ 1
(4 2​ ) ​5
1 16
​9
1 16
​5
1 64
2 14​
EndPractice
End Practice and
and See
See Results
Results Question 453 of 737 Answer
Next Question
Submit Answer
Submit

Math 
Calculator  Flag 
0:03

Choose the option that best answers the question.

40
Kim is taking a math class, and the teacher
gives a multiple choice test consisting of 8
400
questions. If each question has 5 answer
choices, and Kim answers every question, 58
in how many different ways can she
complete the test? 85

4040

EndPractice
End Practice and
and See
See Results
Results Question 454 of 737 Answer
Next Question
Submit Answer
Submit

Math 
Calculator  Flag 
0:02

Choose the option that best answers the question.

2y – 3x
If 3x < 2y < 0, which of the following must
be the greatest?
3x – 2y

–(3x – 2y)

–(3x + 2y)

0
EndPractice
End Practice and
and See
See Results
Results Question 455 of 737 Answer
Next Question
Submit Answer
Submit

Math 
Calculator  Flag 
0:02

Consider each of the choices separately and select all that apply.

2
Which of the following lines are y = ​ x+8
3
perpendicular to the line 3x + 2y = 7? 
2
y = −​ x−6
Indicate all possible values lines. 3
3
y = ​ x+5
2
3 4
y = −​ x− ​
2 7

2x + 3y = 19

2x − 3y = −2

EndPractice
End Practice and
and See
See Results
Results Question 456 of 737 Answer
Next Question
Submit Answer
Submit

Math 
Calculator  Flag 
0:02

Enter the answer in the blank.

Let N = 60! + 55! + 50! The unit digit of N


and a number of digits to the left of the
units digits are consecutive zeros before
we come to the first non-zero digit. How
many such consecutive zeros are there
until the first non-zero digit?

EndPractice
End Practice and
and See
See Results
Results Question 457 of 737 Answer
Next Question
Submit Answer
Submit

Math 
Calculator  Flag 
0:03

Choose the option that best answers the question.

{C, D, A, B}
If A, B, C and D are positive integers such
that 4A = 9B, 17C = 11D, and 5C = 12A,
{B, A, C, D}
then the arrangement of the four numbers
from greatest to least is {D, C, A, B}

{D, C, B, A}

{B, D, A, C}

EndPractice
End Practice and
and See
See Results
Results Question 458 of 737 Answer
Next Question
Submit Answer
Submit

Math 
Calculator  Flag 
0:01

Choose the option that best answers the question.

3
If 22n + 22n + 22n + 22n = 424, then n =

6
12

23

24

EndPractice
End Practice and
and See
See Results
Results Question 459 of 737 Answer
Next Question
Submit Answer
Submit

Math 
Calculator  Flag 
0:02

Choose the option that best answers the question.

1/3
If 8n+1 + 8n = 36, then n =

1/2

3/5

2/3

4/5

EndPractice
End Practice and
and See
See Results
Results Question 460 of 737 Answer
Next Question
Submit Answer
Submit
Magoosh

Math 
Calculator  Flag 
0:03

Consider each of the choices separately and select all that apply.

30

40

50

75

In triangle FGH, the base has a length of 80


FH = 126.3, and the altitude from G to FH
is constructed and has a length of h. What 100
could be the value of h if we know that the
125
area of the triangle is an integer? 

Indicate all possible values of h.       150

EndPractice
End Practice and
and See
See Results
Results Question 461 of 737 Answer
Next Question
Submit Answer
Submit

Math 
Calculator  Flag 
0:05

Enter the answer in the blank.

From the letters in MAGOOSH, we are


going to make three-letter "words." Any set
of three letters counts
as a word, and
different arrangements of the same three
letters (such as
"MAG" and "AGM") count

as different words. How many different
three-letter words can be
made from the
seven letters in MAGOOSH? 

EndPractice
End Practice and
and See
See Results
Results Question 462 of 737 Answer
Next Question
Submit Answer
Submit

Math 
Calculator  Flag 
0:02

Choose the option that best answers the question.

4
Points
Q, R, S, and T are integers that lie
on a number line in that order. The
6
distance between Q and R is twice the
distance between R and T. If point S is
the 10
midpoint of R and T, then which of the
following could be the distance
between Q 12
and S?
14

EndPractice
End Practice and
and See
See Results
Results Question 463 of 737 Answer
Next Question
Submit Answer
Submit

Math 
Calculator  Flag 
0:02

Choose the option that best answers the question.

​ 56
√​1​ 2 × 32 × 54

72√​2

96

96√​2

144

EndPractice
End Practice and
and See
See Results
Results Question 464 of 737 Answer
Next Question
Submit Answer
Submit

Math 
Calculator  Flag 
0:02

Choose the option that best answers the question.

10
In a group of 40 people, 15 have visited
Iceland and 23 have visited Norway. If 11
11
people have visited both Iceland and
Norway, how many people have visited 12
neither country?
13

14

EndPractice
End Practice and
and See
See Results
Results Question 465 of 737 Answer
Next Question
Submit Answer
Submit

Math 
Calculator  Flag 
0:02

Consider each of the choices separately and select all that apply.
1296
Which of the following numbers are
divisible by 6 but not by 18?
2744
Indicate all possible values.
3072

4356

5832

6000

7290

8112

EndPractice
End Practice and
and See
See Results
Results Question 466 of 737 Answer
Next Question
Submit Answer
Submit

Math 
Calculator  Flag 
0:02

Enter the answer as a fraction. Fractions do not need to be in simplest form

Positive number N is a fraction less than


one.  Twice N plus the reciprocal of
N equals 3. What is the value of N?  

Give your answer as a fraction.

EndPractice
End Practice and
and See
See Results
Results Question 467 of 737 Answer
Next Question
Submit Answer
Submit

Math 
Calculator  Flag 
0:02
Choose the option that best answers the question.

0.00016
What is the decimal equivalent of (2/5)5 ?

0.00032

0.00256

0.00512

0.01024

EndPractice
End Practice and
and See
See Results
Results Question 468 of 737 Answer
Next Question
Submit Answer
Submit

Math 
Calculator  Flag 
0:05

Choose the option that best answers the question.

2
In the diagram, AE = 20, AD = 14, CD = 6
and EB = 17. What is the length of line
3
segment BC?

Note: Figure not drawn to scale 4

EndPractice
End Practice and
and See
See Results
Results Question 469 of 737 Answer
Next Question
Submit Answer
Submit


Calculator  Flag 
0:02
Math

Choose the option that best answers the question.

16
When 12 marbles are added to a
rectangular aquarium, the water in the
18
aquarium rises 1 ½ inches. In total, how
many marbles must be added to the 20
aquarium to raise the water 2 ¾ inches?
22

24

EndPractice
End Practice and
and See
See Results
Results Question 470 of 737 Answer
Next Question
Submit Answer
Submit

Math 
Calculator  Flag 
0:03

Consider each of the choices separately and select all that apply.

​ ​
Which of the following is greater than √​79 ?
2√​19

Indicate all possible values. 3√​10

4√​5

5√​3

6√​2

10
EndPractice
End Practice and
and See
See Results
Results Question 471 of 737 Answer
Next Question
Submit Answer
Submit

Math 
Calculator  Flag 
0:02

Enter the answer in the blank.

                  

In the diagram above, AB = BC, and AC =


DC = BD.  What is the measure, in
degrees, of  ∠DAC?

EndPractice
End Practice and
and See
See Results
Results Question 472 of 737 Answer
Next Question
Submit Answer
Submit

Math 
Calculator  Flag 
0:02

Choose the option that best answers the question.

–6
If x is a number such that x2 + 2x – 24 = 0
and x2 + 5x – 6 = 0, then x =
–4

–3

3
6

EndPractice
End Practice and
and See
See Results
Results Question 473 of 737 Answer
Next Question
Submit Answer
Submit

Math 
Calculator  Flag 
0:02

Choose the option that best answers the question.

462 4
If k is an integer and k = ​ , then which of
n
the following could be the value of n? 5

13

22

EndPractice
End Practice and
and See
See Results
Results Question 474 of 737 Answer
Next Question
Submit Answer
Submit

Math 
Calculator  Flag 
0:02

Choose the option that best answers the question.

2 hours and 2 minutes


To reach her destination, Jeanette must
drive 90 miles. If she drives 5 miles every 7
2 hours and 6 minutes
minutes, how much time will it take her to
reach her destination? 2 hours and 10 minutes
2 hours and 12 minutes

2 hours and 15 minutes

EndPractice
End Practice and
and See
See Results
Results Question 475 of 737 Answer
Next Question
Submit Answer
Submit

Math 
Calculator  Flag 
0:02

Consider each of the choices separately and select all that apply.

21 1
If  x > ​ , then x could be which of the

40 2
following? 11

Indicate all possible values of x.  20
13

25
19

36
31

60
38

75
41

80

EndPractice
End Practice and
and See
See Results
Results Question 476 of 737 Answer
Next Question
Submit Answer
Submit

Math 
Calculator  Flag 
0:02
Enter the answer in the blank.

In 2012, Meg bought a house for $400,000.


 In 2015, when she was beginning to sell
her house, her asking price was 40%
higher than what she paid, but the buyer
offered her $70,000 more than her asking
price.  Assuming that she accepted this
offer, what percent profit did she make on
the sale of this house?

Give your answer to the nearest 0.1 of a


percent.   

EndPractice
End Practice and
and See
See Results
Results Question 477 of 737 Answer
Next Question
Submit Answer
Submit

Math 
Calculator  Flag 
0:02

Choose the option that best answers the question.

x x 18
If  ​ + ​ + 15 = x , then x =
3 4
24

36

48

60

EndPractice
End Practice and
and See
See Results
Results Question 478 of 737 Answer
Next Question
Submit Answer
Submit
Math 
Calculator  Flag 
0:02

Choose the option that best answers the question.

18
If a truck is traveling at a constant rate of
90 kilometers per hour, how many seconds
24
will it take the truck to travel a distance of
600 meters? 30
(1 kilometer = 1000 meters)
36

48

EndPractice
End Practice and
and See
See Results
Results Question 479 of 737 Answer
Next Question
Submit Answer
Submit

Math 
Calculator  Flag 
0:02

Choose the option that best answers the question.

x+y
In a certain class, x students are 10 years ​
21
old, and the remaining y students are 11
10x + 11y
years old. What is the average (arithmetic ​
21
mean) age of all students in the class?
110xy

x+y

110xy

xy

10x + 11y

x+y
EndPractice
End Practice and
and See
See Results
Results Question 480 of 737 Answer
Next Question
Submit Answer
Submit
Magoosh

Math 
Calculator  Flag 
0:06

Consider each of the choices separately and select all that apply.

1
The length, width, and a height of a box are
different integer lengths. If the volume is
5
125 cubic centimeters, then which of the
following could be the length of the height 25
in centimeters? 
125

EndPractice
End Practice and
and See
See Results
Results Question 481 of 737 Answer
Next Question
Submit Answer
Submit

Math 
Calculator  Flag 
0:02

Enter the answer in the blank.

In a large bucket of screws, the


ratio of slot
screws to Phillips screws is 11 to 4. There
are no other varieties of screws in the
bucket. If there are 320 Phillips screws
in
the bucket, what is the total number of
screws in the bucket?

EndPractice
End Practice and
and See
See Results
Results Question 482 of 737 Answer
Next Question
Submit Answer
Submit

Math 
Calculator  Flag 
0:02

Choose the option that best answers the question.

504
How many three-digit numbers are there
such that all three digits are different and
648
the first digit is not zero?
720

729

810

EndPractice
End Practice and
and See
See Results
Results Question 483 of 737 Answer
Next Question
Submit Answer
Submit

Math 
Calculator  Flag 
0:02

Choose the option that best answers the question.

–5
If f(x) = x4 – 3x3 – 2x2 + 5x , then f(–1) =

–3

–1

EndPractice
End Practice and
and See
See Results
Results Question 484 of 737 Answer
Next Question
Submit Answer
Submit
Math 
Calculator  Flag 
0:02

Choose the option that best answers the question.

4
The Greatest Common Factor (GCF) of 48
and 72 is
6

12

24

48

EndPractice
End Practice and
and See
See Results
Results Question 485 of 737 Answer
Next Question
Submit Answer
Submit

Math 
Calculator  Flag 
0:02

Consider each of the choices separately and select all that apply.

4.4
In a population of chickens, the average
(arithmetic mean) weight is 6.3 pounds,
4.6
and the standard deviation is 1.2 pounds.
Which of the following weights (in pounds) 5.1
are within 1.5 units of standard deviation of
the mean? 5.2

Indicate all weights.


6.9

7.6
7.7

8.2

EndPractice
End Practice and
and See
See Results
Results Question 486 of 737 Answer
Next Question
Submit Answer
Submit

Math 
Calculator  Flag 
0:02

Enter the answer in the blank.

                 

What is the area of the region shown


above?

EndPractice
End Practice and
and See
See Results
Results Question 487 of 737 Answer
Next Question
Submit Answer
Submit

Math 
Calculator  Flag 
0:02
Choose the option that best answers the question.

2/5
When 6 is multiplied by x, the result is the
same as when x is added to 9. What is the
3/5
value of x/3 ?
6/5

8/5

9/5

EndPractice
End Practice and
and See
See Results
Results Question 488 of 737 Answer
Next Question
Submit Answer
Submit

Math 
Calculator  Flag 
0:02

Choose the option that best answers the question.

480
Walking at a constant rate of 8 kilometers
per hour, Juan can cross a bridge in 6
600
minutes. What is the length of the bridge in
meters? (1 kilometer = 1000 meters) 720

750

800

EndPractice
End Practice and
and See
See Results
Results Question 489 of 737 Answer
Next Question
Submit Answer
Submit

Math 
Calculator  Flag 
0:03
Choose the option that best answers the question.

3x –3
If  ​  = y, and 2 – 3y = y + 2, then x =
2
–2

EndPractice
End Practice and
and See
See Results
Results Question 490 of 737 Answer
Next Question
Submit Answer
Submit

Math 
Calculator  Flag 
0:07

Consider each of the choices separately and select all that apply.

a*(b + c) = a*b + a*c


If a, b, and c are real numbers, and a ≠ 0,
which of the following must be true?
a*(b – c) = a*b – a*c

(b + c)/a = b/a + c/a

(b – c)/a = b/a – c/a

a/(b + c) = a/b + a/c

a/(b – c) = a/b – a/c

(b + c)a = ba + ca

(b - c)a = ba – ca
EndPractice
End Practice and
and See
See Results
Results Question 491 of 737 Answer
Next Question
Submit Answer
Submit

Math 
Calculator  Flag 
0:02

Enter the answer as a fraction. Fractions do not need to be in simplest form

5 3 2
​ + ​ x = 2+ ​ x
3 2 3
What is
the value of x? Give your answer
as
fraction. 

EndPractice
End Practice and
and See
See Results
Results Question 492 of 737 Answer
Next Question
Submit Answer
Submit

Math 
Calculator  Flag 
0:02

Choose the option that best answers the question.

18%
In a certain company, there are 160
researchers and 240 marketers; these are
20%
the only two types of employees in this
company. If 15% of the researchers are 21%
left-handed and 25% of the marketers are
left-handed, what percent of the company’s 23%
entire employee population is left-handed?
24%

EndPractice
End Practice and
and See
See Results
Results Question 493 of 737 Answer
Next Question
Submit Answer
Submit
Math 
Calculator  Flag 
0:02

Choose the option that best answers the question.

35+3k
9k x 272k =

38k

311k

312k

2
312k

EndPractice
End Practice and
and See
See Results
Results Question 494 of 737 Answer
Next Question
Submit Answer
Submit

Math 
Calculator  Flag 
0:02

Choose the option that best answers the question.

90
On a certain high school athletic team, the
ratio of freshmen to sophomores to juniors
140
to seniors is 1:3:4:6. If there are 60 juniors
on the team, how many students in total 150
are on the team?
180

210

End Practice and See Results Question 495 of 737 Submit Answer
Next Question
End Practice and See Results Submit Answer

Math 
Calculator  Flag 
0:01

Consider each of the choices separately and select all that apply.

P+Q+R
The integers P, Q, and R are all positive
odd integers.  Which of the following also
P×Q× R
must be odd?
PQ + PR

PQ − RQ

PQ

QP+R

EndPractice
End Practice and
and See
See Results
Results Question 496 of 737 Answer
Next Question
Submit Answer
Submit

Math 
Calculator  Flag 
0:02

Enter the answer in the blank.

If Michael can shovel all the snow


off a
standard driveway in 12 minutes, and
Eamon can
shovel all the snow off a
standard driveway in 36 minutes, then
working
together, how many minutes would
it take for them both to shovel all the snow
off a standard driveway?

EndPractice
End Practice and
and See
See Results
Results Question 497 of 737 Answer
Next Question
Submit Answer
Submit

Math 
Calculator  Flag 
0:06

Choose the option that best answers the question.

0
If |x + 5| = 3 and |2y - 1|/3 = 5, then |x + y|
could equal each of the following EXCEPT
6

15

EndPractice
End Practice and
and See
See Results
Results Question 498 of 737 Answer
Next Question
Submit Answer
Submit

Math 
Calculator  Flag 
0:02

Choose the option that best answers the question.

xy/
It takes 1 pound of flour to make y cakes. w

The price of flour is w dollars for x pounds. y/


wx
In terms of w, x and y, what is the dollar
cost of the flour required to make 1 cake? w/
xy
wx/
y

wxy

EndPractice
End Practice and
and See
See Results
Results Question 499 of 737 Answer
Next Question
Submit Answer
Submit

Math 
Calculator  Flag 
0:03

Choose the option that best answers the question.

6.25
Captown is the capital city of Maltania. If
the population of Captown is 25 percent of
10
the rest of the population of Maltania, then
the population of Captown is what percent 12.5
of the entire population of Maltania?
20

25

EndPractice
End Practice and
and See
See Results
Results Question 500 of 737 Answer
Next Question
Submit Answer
Submit
Magoosh

Math 
Calculator  Flag 
0:02

Consider each of the choices separately and select all that apply.

P/(Q + S)
Both P and Q are positive numbers, and S
is a negative number. Which of the
Q/(P + S)
following fractions could be undefined?
S/(P + Q)

Q/(S − P)

S/(P − Q)

EndPractice
End Practice and
and See
See Results
Results Question 501 of 737 Answer
Next Question
Submit Answer
Submit

Math 
Calculator  Flag 
0:05

Enter the answer as a fraction. Fractions do not need to be in simplest form

3
2+ ​
If  ​ n = 5​ , what is the value of n?
2 4
3+ ​
n

EndPractice
End Practice and
and See
See Results
Results Question 502 of 737 Answer
Next Question
Submit Answer
Submit

Math 
Calculator  Flag 
0:02

Choose the option that best answers the question.

−4
2 − [1 − (1 − [2 − 3] − 2) + 3] =

−2

EndPractice
End Practice and
and See
See Results
Results Question 503 of 737 Answer
Next Question
Submit Answer
Submit

Math 
Calculator  Flag 
0:03

Choose the option that best answers the question.

3/4
If square ABCD has area 25, and the area
of the larger shaded square is 9 times the
5/4
area of the smaller shaded square, what is
the length of one side of the smaller 6/5
shaded square?
4/3
Note: Figure not drawn to scale

5/3
EndPractice
End Practice and
and See
See Results
Results Question 504 of 737 Answer
Next Question
Submit Answer
Submit

Math 
Calculator  Flag 
0:05

Choose the option that best answers the question.

25
At the moment there are 54,210 tagged
birds in a certain wildlife refuge. If exactly
30
20 percent of all birds in the refuge are
tagged, what percent of the untagged birds 33 1/3
must be tagged so that half of all birds in
the refuge are tagged? 37 1/2

50

EndPractice
End Practice and
and See
See Results
Results Question 505 of 737 Answer
Next Question
Submit Answer
Submit

Math 
Calculator  Flag 
0:02

Consider each of the choices separately and select all that apply.
7x − 3y = 10
Which of the following is an equation of a
7
line with a slope of + ​ ? 3x − 7y = 11
3
Indicate all such equations.
7x + 3y = 12

3x + 7y = 13

3x = 7y +14

7x = 3y +15

3
x = ​ y +16
7
7
x = ​ y +17
3

EndPractice
End Practice and
and See
See Results
Results Question 506 of 737 Answer
Next Question
Submit Answer
Submit

Math 
Calculator  Flag 
0:03

Enter the answer in the blank.

2−n​ 3−n​ 1
If ( ​ )( ​ ) = ​ , what is the value of n?
3 2 36

EndPractice
End Practice and
and See
See Results
Results Question 507 of 737 Answer
Next Question
Submit Answer
Submit

Math 
Calculator  Flag 
0:06
Choose the option that best answers the question.

40%
In Dewey Elementary School, there are two
second-grade classes: class A has 35
42.5%
students and class B has 45 students. If
40% of the students in class A walk to 60%
school, and 80% of the students in class B
walk to school, what percent of all the 62.5%
students in the second-grade at Dewey
Elementary walk to school? 75%

EndPractice
End Practice and
and See
See Results
Results Question 508 of 737 Answer
Next Question
Submit Answer
Submit

Math 
Calculator  Flag 
0:02

Choose the option that best answers the question.

6:5

5:3

9:25

5:6
3
If
AB = BD and AB is
​ of AC, what is the 25:36
5
ratio of circumference of the larger
semicircle to that of the combined
circumference of the two semicircles?

EndPractice
End Practice and
and See
See Results
Results Question 509 of 737 Answer
Next Question
Submit Answer
Submit
Math 
Calculator  Flag 
0:02

Choose the option that best answers the question.


In the xy-coordinate system, a circle with 2 + √​26

radius √​30  and center (2, 1) intersects the ​
2 + √​29
x-axis at (k, 0). One possible value of k is

2 + √​31

2 + √​34

2 + √​35

EndPractice
End Practice and
and See
See Results
Results Question 510 of 737 Answer
Next Question
Submit Answer
Submit

Math 
Calculator  Flag 
0:02

Consider each of the choices separately and select all that apply.

1
From a group of 8 people, it is possible to
create exactly 56 different k-person
2
committees. Which of the following could
be the value of k ? 3
Indicate all such values.
4

7
EndPractice
End Practice and
and See
See Results
Results Question 511 of 737 Answer
Next Question
Submit Answer
Submit

Math 
Calculator  Flag 
0:02

Enter the answer in the blank.

4x 2
If x > 0 and   ​ − ​ = 0, what is
the
x2​ − 3x 7
value of x?

EndPractice
End Practice and
and See
See Results
Results Question 512 of 737 Answer
Next Question
Submit Answer
Submit

Math 
Calculator  Flag 
0:03

Choose the option that best answers the question.

22
Every person in a certain group is either a
Dodgers fan or a Yankees fan, but not
88
both. The ratio of Yankees fans to Dodgers
fans is 5 to 3. If 22 Yankees fans change 128
teams to become Dodgers fans, the ratio of
Dodgers fans to Yankees fans will be 1 to 144
1. How many people are in the group?
176

EndPractice
End Practice and
and See
See Results
Results Question 513 of 737 Answer
Next Question
Submit Answer
Submit
Math 
Calculator  Flag 
0:02

Choose the option that best answers the question.


O is the center of the semicircle. If  4√​3

∠BCO = 30∘​  and BC = 6√​3 , what is the ​
6√​3
area of triangle ABO?

9√​3

12√​3

24√​3

EndPractice
End Practice and
and See
See Results
Results Question 514 of 737 Answer
Next Question
Submit Answer
Submit

Math 
Calculator  Flag 
0:03

Choose the option that best answers the question.

726
If 724 is the greatest common divisor of
positive integers A and B, and 726 is the
7210
least common multiple of A and B, then
AB= 7212

7224

724096
EndPractice
End Practice and
and See
See Results
Results Question 515 of 737 Answer
Next Question
Submit Answer
Submit

Math 
Calculator  Flag 
0:03

Consider each of the choices separately and select all that apply.

3
Which of the following lines intersects the y= ​ x
5
vertical line x = 3 between (3, 1) and (3,
2)?  3
y = ​ x+1
7
Indicate all possible lines.
1 6
y = ​ x+ ​
5 5
3
y = −​ x+5
2
1
y = −​ x+3
4

EndPractice
End Practice and
and See
See Results
Results Question 516 of 737 Answer
Next Question
Submit Answer
Submit

Math 
Calculator  Flag 
0:03

Enter the answer in the blank.

If 16x = 8, what is the value of x? 

EndPractice
End Practice and
and See
See Results
Results Question 517 of 737 Answer
Next Question
Submit Answer
Submit
Math 
Calculator  Flag 
0:06

Choose the option that best answers the question.

I only
Note: Figure not drawn to scale

III only

If x and y are numbers on the number line I and II


above, which of the following statements
must be true? I and III

I. |x + y| < y II and III

II. x + y < 0

III. xy < 0

EndPractice
End Practice and
and See
See Results
Results Question 518 of 737 Answer
Next Question
Submit Answer
Submit

Math 
Calculator  Flag 
0:03

Choose the option that best answers the question.

$10079.44
Cindy invests $10000 in an account that
pays an annual rate of 3.96%,
$10815.83
compounding semi-annually.
Approximately how much does she have in $12652.61
her account after two years?
$14232.14
$20598.11

EndPractice
End Practice and
and See
See Results
Results Question 519 of 737 Answer
Next Question
Submit Answer
Submit

Math 
Calculator  Flag 
0:02

Choose the option that best answers the question.

3 × 1015​ 5 × 107​
​ =
6 × 10 −7​
2 × 108​

5 × 1021

2 × 1022

1.8 × 1023

EndPractice
End Practice and
and See
See Results
Results Question 520 of 737 Answer
Next Question
Submit Answer
Submit
Magoosh

Math 
Calculator  Flag 
0:02

Consider each of the choices separately and select all that apply.

–60
If y – 3x > 12 and x – y > 38, which of the
following are possible values of x?
–30
Indicate all such values.
–6

20

40

80

EndPractice
End Practice and
and See
See Results
Results Question 521 of 737 Answer
Next Question
Submit Answer
Submit

Math 
Calculator  Flag 
0:02

Enter the answer in the blank.

​ ​ ​
If √​28 + √​175 = a√​b where a and b
are
integers each greater than one, then what
is the value of (a + b)?

End Practice and See Results Question 522 of 737 Submit Answer
Next Question
End Practice and See Results Submit Answer

Math 
Calculator  Flag 
0:02

Choose the option that best answers the question.

(1.3333)(0.6666)(1.125) 1/2
The value of   ​ is
(0.75)(0.8)(0.8333)
closest to 2/3

3/2

EndPractice
End Practice and
and See
See Results
Results Question 523 of 737 Answer
Next Question
Submit Answer
Submit

Math 
Calculator  Flag 
0:03

Choose the option that best answers the question.

2
In the below addition A, B, C, D, E, F, and
G represent the digits 0, 1, 2, 3, 4, 5 and 6.
3
If each variable has a different value, and E
≠ 0, then G equals 4

6
EndPractice
End Practice and
and See
See Results
Results Question 524 of 737 Answer
Next Question
Submit Answer
Submit

Math 
Calculator  Flag 
0:03

Choose the option that best answers the question.

1
The functions f(x) and g(x) are defined by ​
2
f(x) = x2 – 1 and g(x) = 1 – 2x. Given that ​
f(g(k)) = 3,  which of the following could be ​​√​3​

the value of k? ​2

3

2

–1

EndPractice
End Practice and
and See
See Results
Results Question 525 of 737 Answer
Next Question
Submit Answer
Submit

Math 
Calculator  Flag 
0:02

Enter the answer as a fraction. Fractions do not need to be in simplest form


In the diagram above, B is the
center of the
larger circle. AB is the
radius of the larger
circle, and it is the diameter of the smaller
circle. The shaded area is what fraction of
the
larger circle?

EndPractice
End Practice and
and See
See Results
Results Question 526 of 737 Answer
Next Question
Submit Answer
Submit

Math 
Calculator  Flag 
0:02

Choose the option that best answers the question.

​ 53
√​(​ 49)(137) − (56)(49) =

57

63

67

73
EndPractice
End Practice and
and See
See Results
Results Question 527 of 737 Answer
Next Question
Submit Answer
Submit

Math 
Calculator  Flag 
0:02

Choose the option that best answers the question.

​ 0.1
Given that 0.​k  represents a decimal in
which the digit k repeats without end, then
​1 1
what is the value of ​ ​ ​ ?
​(0.5) − (0.​6​ )2​
4.5

18

EndPractice
End Practice and
and See
See Results
Results Question 528 of 737 Answer
Next Question
Submit Answer
Submit

Math 
Calculator  Flag 
0:02

Choose the option that best answers the question.


If the circle with center O has area 9π , what 9√​3
is the area of equilateral triangle ABC?
18

12√​3

24

16√​3
EndPractice
End Practice and
and See
See Results
Results Question 529 of 737 Answer
Next Question
Submit Answer
Submit

Math 
Calculator  Flag 
0:03

Enter the answer in the blank.

Tuk weighs 60 percent more than Kim, Lee


weighs 50 percent less than Tuk, and Pat
weighs 25 percent more than Lee. If Pat
weighs 126 pounds, what is Kim’s weight?

EndPractice
End Practice and
and See
See Results
Results Question 530 of 737 Answer
Next Question
Submit Answer
Submit

Math 
Calculator  Flag 
0:02

Choose the option that best answers the question.


10
If 24 kilograms of flour are required to ​
3
make 300 tarts, how many kilograms of
flour are required to make 45 tarts? 32

9
18

5
15

4
64

15

EndPractice
End Practice and
and See
See Results
Results Question 531 of 737 Answer
Next Question
Submit Answer
Submit

Math 
Calculator  Flag 
0:02

Choose the option that best answers the question.

x2​
If the area of the outer square is 4x2, then ​
2
the area of the inner square is
x2​  

√​2 x2​

2x2​  

2√​2x2​

EndPractice
End Practice and
and See
See Results
Results Question 532 of 737 Answer
Next Question
Submit Answer
Submit
Math 
Calculator  Flag 
0:03

Choose the option that best answers the question.

138
While driving from A-ville to B-town, Harriet
drove at a constant speed of 115
148
kilometers per hour. Upon arriving in B-
town, Harriet immediately turned and drove 150
back to A-ville at a constant speed of 135
kilometers per hour. If the entire trip took 5 162
hours, how many minutes did it take Harriet
to drive from A-ville to B-town? 168

EndPractice
End Practice and
and See
See Results
Results Question 533 of 737 Answer
Next Question
Submit Answer
Submit

Math 
Calculator  Flag 
0:02

Enter the answer in the blank.

Whenever Art Dealer sells a sculpture, he


earns a 20 percent commission on the first
$12,000 of the sale price plus 15 percent of
the sale price in excess of $12,000. If Art
earned a $3,900 commission on the sale of
a certain sculpture, what was the sale
price?

EndPractice
End Practice and
and See
See Results
Results Question 534 of 737 Answer
Next Question
Submit Answer
Submit
Math 
Calculator  Flag 
0:02

Choose the option that best answers the question.

12
In a 30-60-90 triangle, the hypotenuse has

a length of 24√​3 . What is the length of the ​
12√​3
leg opposite the 60° angle?
24

36

36√​3

EndPractice
End Practice and
and See
See Results
Results Question 535 of 737 Answer
Next Question
Submit Answer
Submit

Math 
Calculator  Flag 
0:02

Choose the option that best answers the question.

273
The nth term (tn) of a certain sequence is
defined as tn = tn – 1 + 4. 
277
 If t1 = –7 then t71 =
281

283

287

EndPractice
End Practice and
and See
See Results
Results Question 536 of 737 Answer
Next Question
Submit Answer
Submit
Math 
Calculator  Flag 
0:02

Choose the option that best answers the question.

54
A certain school district has specified 5
different novels and 4 different non-fiction
60
books from which teachers can choose to
assemble a summer reading list.  Each 72
summer reading list must have exactly 3
novels and 2 non-fiction books. How many 120
different summer reading lists are
possible? 240

EndPractice
End Practice and
and See
See Results
Results Question 537 of 737 Answer
Next Question
Submit Answer
Submit

Math 
Calculator  Flag 
0:01

Enter the answer in the blank.

In the figure below, ABC is a sector with


center A. If arc BC has length 4π, what is
the length of AC?

End Practice and See Results Question 538 of 737 Submit Answer
Next Question
End Practice and See Results Submit Answer

Math 
Calculator  Flag 
0:02

Choose the option that best answers the question.

1/20
Set K consists of all fractions of the form
x/(x+2) where x is a positive even integer
1/10
less than 20. What is the product of all the
fractions in Set K ? 1/9

1/2

8/9

EndPractice
End Practice and
and See
See Results
Results Question 539 of 737 Answer
Next Question
Submit Answer
Submit

Math 
Calculator  Flag 
0:05

Choose the option that best answers the question.

P 5
If A is the initial amount put into an 4A( ​ )​
100
account, P is the annual percentage rate of
interest, which remains fixed, and the P 20
A( ​ )​
account compounds quarterly, which of the
400
following is an expression, in terms of A P 5
A(1 + ​ )​
and P, for the amount in the account after 5 100
years? P 20
A(1 + ​ )​
25
P 20
A(1 + ​ )​
400

EndPractice
End Practice and
and See
See Results
Results Question 540 of 737 Answer
Next Question
Submit Answer
Submit
Magoosh

Math 
Calculator  Flag 
0:03

Choose the option that best answers the question.

0
What is the units digit of 1847?

EndPractice
End Practice and
and See
See Results
Results Question 541 of 737 Answer
Next Question
Submit Answer
Submit

Math 
Calculator  Flag 
0:03

Enter the answer in the blank.

In the quadrilateral shown here, a+b+c+d=



EndPractice
End Practice and
and See
See Results
Results Question 542 of 737 Answer
Next Question
Submit Answer
Submit

Math 
Calculator  Flag 
0:02

Choose the option that best answers the question.

Astronomers determine that a certain 5 × 103​


square region in interstellar space has an
2.5 × 105​
area of approximately 2.4 × 107​  (light-
years)2, where a light-year is a unit 2.5 × 103​
of astronomical distance.  Assuming that
this region is a square, then a single side of 5 × 105​
the square would be approximately how
many light-years long?   6.25 × 103​

EndPractice
End Practice and
and See
See Results
Results Question 543 of 737 Answer
Next Question
Submit Answer
Submit

Math 
Calculator  Flag 
0:02

Choose the option that best answers the question.

2x + 2y – 180
What is the value of w in terms of x and y?

Note: Figure not drawn to scale 180 – x – y

360 – 2x – 2y

360 – 2x – 3y
180 + x – 2y

EndPractice
End Practice and
and See
See Results
Results Question 544 of 737 Answer
Next Question
Submit Answer
Submit

Math 
Calculator  Flag 
0:01

Choose the option that best answers the question.

8
n is a positive integer, and k is the product
of all integers from 1 to n inclusive. If k is a
12
multiple of 1440, then the smallest possible
value of n is 16

18

24

EndPractice
End Practice and
and See
See Results
Results Question 545 of 737 Answer
Next Question
Submit Answer
Submit

Math 
Calculator  Flag 
0:02

Enter the answer in the blank.


City A and City B are connected by a
straight 420-mile road.  At noon, Nigel left
City A, traveling toward City B, and Phyllis
left City B, traveling toward City A.  If Nigel
travels at 49 miles per hour and Phyllis
travels at 56 miles per hour, how many
miles apart will Nigel and Phyllis be 1
hour before they meet?

EndPractice
End Practice and
and See
See Results
Results Question 546 of 737 Answer
Next Question
Submit Answer
Submit

Math 
Calculator  Flag 
0:02

Choose the option that best answers the question.

66
Every day at noon, a bus leaves for
Townville and travels at a speed of x
72
kilometers per hour. Today, the bus left 30
minutes late. If the driver drives 7/6 times 80
as fast as usual, she will arrive in Townville
at the regular time. If the distance to 84
Townville is 280 kilometers, what is the
value of x? 90

EndPractice
End Practice and
and See
See Results
Results Question 547 of 737 Answer
Next Question
Submit Answer
Submit

Math 
Calculator  Flag 
0:02
Choose the option that best answers the question.

0
What is the Greatest Common Factor
(GCF) of 25x2 and 16y4?
1

xy2

x2y4

400x2y4

EndPractice
End Practice and
and See
See Results
Results Question 548 of 737 Answer
Next Question
Submit Answer
Submit

Math 
Calculator  Flag 
0:02

Choose the option that best answers the question.

None
The system of equations has how many
solutions?
Exactly 1
3x − 6y = 9
Exactly 2
2y − x − 3 = 0
Exactly 3

Infinitely many

EndPractice
End Practice and
and See
See Results
Results Question 549 of 737 Answer
Next Question
Submit Answer
Submit


Calculator  Flag 
0:02
Math

Enter the answer in the blank.

Helen mixes a first solution, 4


liters of 40%
concentrated sulfuric acid solution, with a
second solution, 5
liters of a sulfuric acid
solution with a stronger concentration, and
the
resultant solution is 9 liters of 50%
concentrated sulfuric acid solution. What
was the concentration, as a percent, of
the
second solution?

EndPractice
End Practice and
and See
See Results
Results Question 550 of 737 Answer
Next Question
Submit Answer
Submit

Math 
Calculator  Flag 
0:02

Choose the option that best answers the question.

60ks
A computer can perform c calculations in s ​
c
seconds. How many minutes will it take the
computer to perform k calculations? ks

c
ks

60c
60c

ks
k

60cs

End Practice and See Results Question 551 of 737 Submit Answer
Next Question
End Practice and See Results Submit Answer

Math 
Calculator  Flag 
0:02

Choose the option that best answers the question.

2
Working together, 7 identical pumps can 4​
3
empty a pool in 6 hours. How many hours
will it take 4 pumps to empty the same 1
9​
4
pool?
1
9​
3
3
9​
4
1
10 ​
2

EndPractice
End Practice and
and See
See Results
Results Question 552 of 737 Answer
Next Question
Submit Answer
Submit

Math 
Calculator  Flag 
0:02

Choose the option that best answers the question.

0.064
The probability is 0.6 that an “unfair” coin
will turn up tails on any given toss. If the
0.36
coin is tossed 3 times, what is the
probability that at least one of the tosses 0.64
will turn up tails?
0.784
0.936

EndPractice
End Practice and
and See
See Results
Results Question 553 of 737 Answer
Next Question
Submit Answer
Submit

Math 
Calculator  Flag 
0:02

Enter the answer in the blank.

What is the y-intercept of the graph of the


equation
y = 2|4x – 4| – 10?

EndPractice
End Practice and
and See
See Results
Results Question 554 of 737 Answer
Next Question
Submit Answer
Submit

Math 
Calculator  Flag 
0:02

Choose the option that best answers the question.

​ ​
√​0.00001 = ​​√​1​0

​100
1

100

​​ √​1​0

​1000
1

1000

​​ ​√​1​0
​10000

EndPractice
End Practice and
and See
See Results
Results Question 555 of 737 Answer
Next Question
Submit Answer
Submit

Math 
Calculator  Flag 
0:06

Choose the option that best answers the question.

​ {2, 3}
y  − 13√​y + 36 = 0

Which of the follow is a set of all possible {±2, ±3}


values of y that satisfy this equation.
{4, 9}

{−4, −9}

{16, 81}

EndPractice
End Practice and
and See
See Results
Results Question 556 of 737 Answer
Next Question
Submit Answer
Submit

Math 
Calculator  Flag 
0:02

Choose the option that best answers the question.

20
If a right triangle has area 28 and
hypotenuse 12, what is its perimeter?
24

28
32

36

EndPractice
End Practice and
and See
See Results
Results Question 557 of 737 Answer
Next Question
Submit Answer
Submit

Math 
Calculator  Flag 
0:02

Enter the answer in the blank.

AB is the diameter of the circle. If AF = BF



= 3√​2 and AC = 5, what is the area of
square BCDE?

EndPractice
End Practice and
and See
See Results
Results Question 558 of 737 Answer
Next Question
Submit Answer
Submit

Math 
Calculator  Flag 
0:03

Choose the option that best answers the question.


2
If a, b, c, d, e and f are integers and (ab +
cdef) < 0, then what is the maximum
3
number of integers that can be negative?
4

EndPractice
End Practice and
and See
See Results
Results Question 559 of 737 Answer
Next Question
Submit Answer
Submit

Math 
Calculator  Flag 
0:02

Choose the option that best answers the question.

$5800
In 2004, Cindy had $4000 in a mutual fund
account. In 2005, the amount in the same
$6000
account was $5000. If the percent increase
from 2004 to 2005 was the same as the $6250
percent increase from 2005 to 2006, how
much did Cindy have in this account in $7500
2006?
$9000

EndPractice
End Practice and
and See
See Results
Results Question 560 of 737 Answer
Next Question
Submit Answer
Submit
Magoosh

Math 
Calculator  Flag 
0:02

Choose the option that best answers the question.

14
At a certain store, one can buy 6 cans of
juice for $8. How many cans of this same
36
juice could one buy with $48?
48

64

96

EndPractice
End Practice and
and See
See Results
Results Question 561 of 737 Answer
Next Question
Submit Answer
Submit

Math 
Calculator  Flag 
0:01

Enter the answer as a fraction. Fractions do not need to be in simplest form

In the standard x,y plane, what is


the slope
of the line that is perpendicular to the
line 12x + 3y = 7?

EndPractice
End Practice and
and See
See Results
Results Question 562 of 737 Answer
Next Question
Submit Answer
Submit
Math 
Calculator  Flag 

0:02

Choose the option that best answers the question.

10784
732 + 742 =

10777

10779

10801

10805

EndPractice
End Practice and
and See
See Results
Results Question 563 of 737 Answer
Next Question
Submit Answer
Submit

Math 
Calculator  Flag 
0:02

Choose the option that best answers the question.

$91.00
At the close of the market on Monday, the
price of a certain volatile stock was exactly
$92.40
$100.00 per share. By close of the day on
Tuesday, the stock was 20% up from its $100.00
start that day. By the close of the day on
Wednesday, the stock was 10% up from its $101.07
start that day. By the close of the day on
Thursday, the stock was 30% down from its $109.89
start that day. What was the price per
share by the end of the day on Thursday?

EndPractice
End Practice and
and See
See Results
Results Question 564 of 737 Submit Answer
Submit Answer Next Question

Math 
Calculator  Flag 
0:02

Choose the option that best answers the question.

4
If 20 percent of 3k is 6, what is 40 percent
of k?
12

24

36

72

EndPractice
End Practice and
and See
See Results
Results Question 565 of 737 Answer
Next Question
Submit Answer
Submit

Math 
Calculator  Flag 
0:03

Choose the option that best answers the question.

40
The sum of k consecutive integers is 41. If
the least integer is -40, then k =
41

80

81

82
EndPractice
End Practice and
and See
See Results
Results Question 566 of 737 Answer
Next Question
Submit Answer
Submit

Math 
Calculator  Flag 
0:02

Choose the option that best answers the question.

k2 – 9
If the average (arithmetic mean) of seven
consecutive integers is k + 2, then the
k2 – 2k + 1
product of the greatest and least integer is
k2 + 4k – 12

k2 + 6k + 9

k2 + 4k – 5

EndPractice
End Practice and
and See
See Results
Results Question 567 of 737 Answer
Next Question
Submit Answer
Submit

Math 
Calculator  Flag 
0:01

Choose the option that best answers the question.

Cannot be determined
On a certain multiple-choice test, 9 points
are awarded for each correct answer, and
16
7 points are deducted for each incorrect or
unanswered question. Sally received a total 19
score of 0 points on the test. If the test has
fewer than 30 questions, how many 21
questions are on the test?
24

EndPractice
End Practice and
and See
See Results
Results Question 568 of 737 Answer
Next Question
Submit Answer
Submit

Math 
Calculator  Flag 
0:02

Choose the option that best answers the question.

4
In a certain sock drawer, there are 4 pairs
of black socks, 3 pairs of gray socks and 2
7
pairs of orange socks. If socks are
removed at random without replacement, 9
what is the minimum number of socks that
must be removed in order to ensure that 10
two socks of the same color have been
removed? 11

EndPractice
End Practice and
and See
See Results
Results Question 569 of 737 Answer
Next Question
Submit Answer
Submit

Math 
Calculator  Flag 
0:03

Choose the option that best answers the question.

250/3
Solution X is 10 percent alcohol by volume,
and solution Y is 30 percent alcohol by
500/3
volume. How many milliliters of solution Y
must be added to 200 milliliters of solution 400
X to create a solution that is 25 percent
alcohol by volume? 480

600

EndPractice
End Practice and
and See
See Results
Results Question 570 of 737 Answer
Next Question
Submit Answer
Submit

Math 
Calculator  Flag 
0:05

Choose the option that best answers the question.

1
If the ratio of x to y is 4 times the ratio of y ​
y 4
to x, then ​ could be
x 1

2

EndPractice
End Practice and
and See
See Results
Results Question 571 of 737 Answer
Next Question
Submit Answer
Submit

Math 
Calculator  Flag 
0:02

Choose the option that best answers the question.

3
If k is the greatest positive integer such that
3k is a divisor of 15! then k =
4
5

EndPractice
End Practice and
and See
See Results
Results Question 572 of 737 Answer
Next Question
Submit Answer
Submit

Math 
Calculator  Flag 
0:02

Choose the option that best answers the question.

1 48
A
ball, when dropped, will bounce ​  of the
3
height from which it was dropped. If
the ball 78
is dropped from 54 feet, what is the total
distance in feet the ball
will have traveled 90
the moment it hits the ground for the third
102
time?

106

EndPractice
End Practice and
and See
See Results
Results Question 573 of 737 Answer
Next Question
Submit Answer
Submit

Math 
Calculator  Flag 
0:02

Choose the option that best answers the question.

37.5
Solution Y is 40 percent sugar by volume,
and solution X is 20 percent sugar by
75
volume. How many gallons of solution X
must be added to 150 gallons of solution Y 150
to create a solution that is 25 percent sugar
by volume? 240

450

EndPractice
End Practice and
and See
See Results
Results Question 574 of 737 Answer
Next Question
Submit Answer
Submit

Math 
Calculator  Flag 
0:02

Choose the option that best answers the question.

4/3
A container currently holds 4 quarts of
alcohol and 4 quarts of water. How many
5/3
quarts of water must be added to the
container to create a mixture that is 3 parts 7/3
alcohol to 5 parts water by volume?
8/3

10/3

EndPractice
End Practice and
and See
See Results
Results Question 575 of 737 Answer
Next Question
Submit Answer
Submit

Math 
Calculator  Flag 
0:02

Choose the option that best answers the question.

1
/9
Each circle has center O. The radius of the
smaller circle is 2 and the radius of the 1/
6
larger circle is 6. If a point is selected at
random from the larger circular region, 2/
3
what is the probability that the point will lie
in the shaded region? 5/
6

8/
9

EndPractice
End Practice and
and See
See Results
Results Question 576 of 737 Answer
Next Question
Submit Answer
Submit

Math 
Calculator  Flag 
0:04

Choose the option that best answers the question.

15
For the first 5 hours of a trip, a plane
averaged 120 kilometers per hour. For the
20
remainder of the trip, the plane travelled an
average speed of 180 kilometers per hour. 25
If the average speed for the entire trip was
170 kilometers per hour, how many hours 30
long was the entire trip?
35

EndPractice
End Practice and
and See
See Results
Results Question 577 of 737 Answer
Next Question
Submit Answer
Submit
Math 
Calculator  Flag 
0:02

Choose the option that best answers the question.

4.1
In the xy-coordinate system, the distance
​ ​ ​ ​
between points (2√​3 , −√​2)   and (5√​3 , 3√​2)
5.9
is approximately
6.4

7.7

8.1

EndPractice
End Practice and
and See
See Results
Results Question 578 of 737 Answer
Next Question
Submit Answer
Submit

Math 
Calculator  Flag 
0:03

Choose the option that best answers the question.

For all numbers a and b, the operation ⊕ is II only

defined by a ⊕ b = a2 – ab. If xy ≠ 0, then


I and II only
which of the following can be equal to
zero? I and III only
I. x ⊕ y
II and III only
II. xy ⊕ y
All of the above
III. x ⊕ (x + y)

End Practice and See Results Question 579 of 737 Submit Answer
Next Question
End Practice and See Results Submit Answer

Math 
Calculator  Flag 
0:02

Choose the option that best answers the question.

4 2 8
(1 ​ )​ 1​
5 25
16
1​
25
3
2​
5
16
2​
25
6
3​
25

EndPractice
End Practice and
and See
See Results
Results Question 580 of 737 Answer
Next Question
Submit Answer
Submit
Magoosh

Math 
Calculator  Flag 
0:02

Choose the option that best answers the question.

6
2600 has how many positive divisors?

12

18

24

48

EndPractice
End Practice and
and See
See Results
Results Question 581 of 737 Answer
Next Question
Submit Answer
Submit

Math 
Calculator  Flag 
0:02

Choose the option that best answers the question.

6
W, X, Y and Z each represent a different
number. If the sum of each column is
7
shown beneath that column, and the sum
of each row is shown beside that row, then 8
n=
9

10

EndPractice
End Practice and
and See
See Results
Results Question 582 of 737 Answer
Next Question
Submit Answer
Submit

Math 
Calculator  Flag 
0:02

Choose the option that best answers the question.

x/
7 −5
The average (arithmetic mean) of y
numbers is x. If 30 is added to the set of x/
6 −6
numbers, then the average will be x − 5.
What is the value of y in terms of x ? x/ −5
6

x/
5 −7

x/
5 −6

EndPractice
End Practice and
and See
See Results
Results Question 583 of 737 Answer
Next Question
Submit Answer
Submit

Math 
Calculator  Flag 
0:02

Choose the option that best answers the question.

180
At Gallicum Enterprises, all employees are
in one of three categories: J, K, or L.  The
240
ratio of the numbers of employees in J to K
to L has been 1:3:5 for some time.  Last 260
month, 20 new J employees were hired,
and no employees left.  If the new ratio of J 320
to K is now 1:2 with these new J
employees, what is the new total number of 380
employees at Gallicum Enterprises?  

EndPractice
End Practice and
and See
See Results
Results Question 584 of 737 Answer
Next Question
Submit Answer
Submit

Math 
Calculator  Flag 
0:05

Choose the option that best answers the question.

0
What is the remainder when 4386 is divided
by 5?
1

EndPractice
End Practice and
and See
See Results
Results Question 585 of 737 Answer
Next Question
Submit Answer
Submit

Math 
Calculator  Flag 
0:01
Choose the option that best answers the question.

6
How many odd, positive divisors does 540
have?
8

12

15

24

EndPractice
End Practice and
and See
See Results
Results Question 586 of 737 Answer
Next Question
Submit Answer
Submit

Math 
Calculator  Flag 
0:02

Choose the option that best answers the question.

100
In a list of four positive integers, the range
is 60 and the median is 50.  What is the
105
highest possible value of the largest
number in the list?   108

109

110

EndPractice
End Practice and
and See
See Results
Results Question 587 of 737 Answer
Next Question
Submit Answer
Submit

Math 
Calculator  Flag 
0:02
Choose the option that best answers the question.

4
How many points (x, y) lie on the line
segment between (22, 12 2/3) and (7, 17
2/ ) such that x and y are both integers?
5
3

EndPractice
End Practice and
and See
See Results
Results Question 588 of 737 Answer
Next Question
Submit Answer
Submit

Math 
Calculator  Flag 
0:04

Choose the option that best answers the question.

1/
A box contains 10 pairs of shoes (20 shoes 190

in total). If two shoes are selected at 1/


20
random, what is the probability that they
are matching shoes? 1/
19

1/
10

1/
9

EndPractice
End Practice and
and See
See Results
Results Question 589 of 737 Answer
Next Question
Submit Answer
Submit
Math 
Calculator  Flag 
0:02

Choose the option that best answers the question.

27%
If a set of numbers is normally distributed
with a mean of 82 and a standard deviation
68%
of 6, approximately what percent of the
numbers are greater than 76 and less than 73%
94?
82%

95%

EndPractice
End Practice and
and See
See Results
Results Question 590 of 737 Answer
Next Question
Submit Answer
Submit

Math 
Calculator  Flag 
0:02

Choose the option that best answers the question.

10
Sid intended to type a seven-digit number,
but the two 3's he meant to type did not
16
appear. What appeared instead was the
five-digit number 52115. How many 21
different seven-digit numbers could Sid
have meant to type? 24

27

EndPractice
End Practice and
and See
See Results
Results Question 591 of 737 Answer
Next Question
Submit Answer
Submit
Math 
Calculator  Flag 
0:02

Choose the option that best answers the question.

3/
Set A: {1, 3, 4, 6, 9, 12, 15} 14

2/
If three numbers are randomly selected 7
from set A without replacement, what is the
9/
probability that the sum of the three 14

numbers is divisible by 3? 5/
7

11/
14

EndPractice
End Practice and
and See
See Results
Results Question 592 of 737 Answer
Next Question
Submit Answer
Submit

Math 
Calculator  Flag 
0:06

Choose the option that best answers the question.

75%
If the length of each side of an equilateral
triangle were increased by 50 percent,
100%
what would be the percent increase in the
area? 125%

150%

225%

EndPractice
End Practice and
and See
See Results
Results Question 593 of 737 Answer
Next Question
Submit Answer
Submit
Math 
Calculator  Flag 
0:02

Choose the option that best answers the question.

42
A professional driver drove a long linear
route at an average speed of 30 miles per
45
hour. Immediately after completing this
drive, the driver turned around drove back 50
on the same route at an average speed of
70 miles per hour.  If the round trip took 2 54
hours, how many miles long is this route?
60

EndPractice
End Practice and
and See
See Results
Results Question 594 of 737 Answer
Next Question
Submit Answer
Submit

Math 
Calculator  Flag 
0:06

Choose the option that best answers the question.

9/
A: {71,73,79,83,87} B:{57,59,61,67} 20

3/
If one number is selected at random from 5
set A, and one number is selected at
3/
random from set B, what is the probability 4

that both numbers are prime? 4/


5

EndPractice
End Practice and
and See
See Results
Results Question 595 of 737 Answer
Next Question
Submit Answer
Submit
Math 
Calculator  Flag 
0:02

Choose the option that best answers the question.

6.75
Noelle walks from point A to point B at an
average speed of 5 kilometers per hour. At
7
what speed, in kilometers per hour, must
Noelle walk from point B to point A so that 7.25
her average speed for the entire trip is 6
kilometers per hour? 7.5

7.75

EndPractice
End Practice and
and See
See Results
Results Question 596 of 737 Answer
Next Question
Submit Answer
Submit

Math 
Calculator  Flag 
0:03

Choose the option that best answers the question.

1/
A box contains 10 balls numbered from 1 to 100

10 inclusive. If Ann removes a ball at 1/


90
random and then returns to the box, and
then Jane removes a ball at random, what 1/
45
is the probability that both women removed
the same ball? 1/
10

41/
45
EndPractice
End Practice and
and See
See Results
Results Question 597 of 737 Answer
Next Question
Submit Answer
Submit

Math 
Calculator  Flag 
0:05

Choose the option that best answers the question.

12π
A cow is tethered to the corner of a
rectangular shed. If the length of the rope is
15π
5, and the shed has length 4 and width 3,
what is the maximum area that is 16π
accessible to the cow? (The cow cannot
enter the shed). 18π

20π

EndPractice
End Practice and
and See
See Results
Results Question 598 of 737 Answer
Next Question
Submit Answer
Submit

Math 
Calculator  Flag 
0:03

Choose the option that best answers the question.

0
In the coordinate plane, line L passes
above the points (50, 70) and (100, 89) but 1
below the point (80, 84).  Which of the ​
2
following could be the slope of line L?
1

4
2

5
6

7

EndPractice
End Practice and
and See
See Results
Results Question 599 of 737 Answer
Next Question
Submit Answer
Submit

Math 
Calculator  Flag 
0:02

Choose the option that best answers the question.


What is the perimeter of ABCDE? 36 + 6√​3

48

36 + 8√​3

36 + 12√​3

60

EndPractice
End Practice and
and See
See Results
Results Question 600 of 737 Answer
Next Question
Submit Answer
Submit
Magoosh

Math 
Calculator  Flag 
0:02

Choose the option that best answers the question.

30
After taxes, Sally takes home a salary of J
= $5000 every month.  She pays P percent
40
of this to her rent and all her fixed bills each
month, leaving her with K left.   She spends 50
half of K on groceries, leaving her with L
1 60
left.  If she spends ​ of L on gifts and puts
3
2
​ of L into her savings account, this would 70
5
leave her with $200 for miscellaneous
expenses.  What is the value of P?

EndPractice
End Practice and
and See
See Results
Results Question 601 of 737 Answer
Next Question
Submit Answer
Submit

Math 
Calculator  Flag 
0:03

Choose the option that best answers the question.

1
The ratio of two positive numbers is 3 to 4.
If k is added to each number the new ratio
13
will be 4 to 5, and the sum of the numbers
will be 117. What is the value of k? 14

18

21

EndPractice
End Practice and
and See
See Results
Results Question 602 of 737 Answer
Next Question
Submit Answer
Submit

Math 
Calculator  Flag 
0:02

Choose the option that best answers the question.

1/
When a certain coin is flipped, the 4

probability of heads is 0.5. If the coin is 1/


3
flipped 6 times, what is the probability that
there are exactly 3 heads? 5/
16

31/
64

1/
2

EndPractice
End Practice and
and See
See Results
Results Question 603 of 737 Answer
Next Question
Submit Answer
Submit

Math 
Calculator  Flag 
0:02

Choose the option that best answers the question.

450
The sum of all the digits of the integers
from 18 to 21 inclusive is 24 (1+8 + 1+9 +
810
2+0 + 2+1 = 24). What is the sum of all the
digits of the integers from 0 to 99 inclusive? 900

1000
1100

EndPractice
End Practice and
and See
See Results
Results Question 604 of 737 Answer
Next Question
Submit Answer
Submit

Math 
Calculator  Flag 
0:02

Choose the option that best answers the question.

30
At a certain farm the ratio of pigs to cows to
chickens is 7:8:10. If the total number of
90
pigs, cows and chickens is 300, how many
chickens are there? 120

180

200

EndPractice
End Practice and
and See
See Results
Results Question 605 of 737 Answer
Next Question
Submit Answer
Submit

Math 
Calculator  Flag 
0:02

Choose the option that best answers the question.

$1.08
If the sales tax on a $12.00 purchase is
$0.66, what is the sales tax on a $20.00
$1.10
purchase?
$1.16
$1.18

$1.20

EndPractice
End Practice and
and See
See Results
Results Question 606 of 737 Answer
Next Question
Submit Answer
Submit

Math 
Calculator  Flag 
0:02

Choose the option that best answers the question.

56
If 4x = 14 and xy = 1 then y =

7/2

2/7

1/5

1/56

EndPractice
End Practice and
and See
See Results
Results Question 607 of 737 Answer
Next Question
Submit Answer
Submit

Math 
Calculator  Flag 
0:01

Choose the option that best answers the question.

1440
A certain essay consists of 15 paragraphs.
Each paragraph contains at least 110
1540
words but not more than 120 words. Which
of the following could be the total number
1640
of words in the essay?
1740

1840

EndPractice
End Practice and
and See
See Results
Results Question 608 of 737 Answer
Next Question
Submit Answer
Submit

Math 
Calculator  Flag 
0:02

Choose the option that best answers the question.

x – 4y
The average (arithmetic mean) of two
numbers is 4x. If one of the numbers is y,
4x + 4y
then the value of the other number is
8x – 4y

4y – 8x

8x – y

EndPractice
End Practice and
and See
See Results
Results Question 609 of 737 Answer
Next Question
Submit Answer
Submit

Math 
Calculator  Flag 
0:02

Choose the option that best answers the question.

1
In a high school in a small town, there are ​
4
12 freshmen, 14 sophomores, 15 juniors,
and 19 seniors.  If a student is selected at 1

5
random, what is the probability that this
student will be a freshman?   1

6
3

8
1

12

EndPractice
End Practice and
and See
See Results
Results Question 610 of 737 Answer
Next Question
Submit Answer
Submit

Math 
Calculator  Flag 
0:03

Choose the option that best answers the question.

15
In Aldebaran Corporation, 35% of the
managers have an MBA. If there are 42
57
managers with MBA, what is the total
number of managers? 84

120

147

EndPractice
End Practice and
and See
See Results
Results Question 611 of 737 Answer
Next Question
Submit Answer
Submit

Math 
Calculator  Flag 
0:02

Choose the option that best answers the question.


3
There are 10 employees in an office, not
counting the office manager. The table
4
shows how many employees have 0, 1, 2
or 3 pets. If the office manager also were 5
included in the table, the average
(arithmetic mean) number of pets per 6
person would equal the median number of
pets per person. How many pets does the 7
office manager have?

EndPractice
End Practice and
and See
See Results
Results Question 612 of 737 Answer
Next Question
Submit Answer
Submit

Math 
Calculator  Flag 
0:06

Choose the option that best answers the question.

–100 and –100


Set X consists of 100 numbers. The
average (arithmetic mean) of set X is 10,
–10 and –10
and the standard deviation is 4.6. Which of
the following two numbers, when added to 0 and 0
set X, will decrease the set’s standard
deviation by the greatest amount? 0 and 20

10 and 10
EndPractice
End Practice and
and See
See Results
Results Question 613 of 737 Answer
Next Question
Submit Answer
Submit

Math 
Calculator  Flag 
0:02

Choose the option that best answers the question.

1
The Greatest Common Factor (GCF) of 18
and 24 is
2

EndPractice
End Practice and
and See
See Results
Results Question 614 of 737 Answer
Next Question
Submit Answer
Submit

Math 
Calculator  Flag 
0:02

Choose the option that best answers the question.

6
How many positive integers less than 100
have a remainder of 2 when divided by 13?
7

10
EndPractice
End Practice and
and See
See Results
Results Question 615 of 737 Answer
Next Question
Submit Answer
Submit

Math 
Calculator  Flag 
0:02

Choose the option that best answers the question.

A(R)3
If A is the initial amount put into an
account, R is the annual percentage of
A(R+R3)
interest written as a decimal, and the
interest compounds annually, then which of A(3R+3R2+R3)
the following would be an expression, in
terms of A and R, for the interest accrued 3A(R)3
in three years?
3A(R+R2+R3)

EndPractice
End Practice and
and See
See Results
Results Question 616 of 737 Answer
Next Question
Submit Answer
Submit

Math 
Calculator  Flag 
0:01

Choose the option that best answers the question.

16
If it takes Bill 8 minutes to peel 30 potatoes,
how many potatoes can he peel in one
120
hour?
144

225
240

EndPractice
End Practice and
and See
See Results
Results Question 617 of 737 Answer
Next Question
Submit Answer
Submit

Math 
Calculator  Flag 
0:06

Choose the option that best answers the question.

0.5
If f(x) = 5 − 2x and f(3k) = f(k + 1), then f(k)
=
1

EndPractice
End Practice and
and See
See Results
Results Question 618 of 737 Answer
Next Question
Submit Answer
Submit

Math 
Calculator  Flag 
0:02

Choose the option that best answers the question.

1.2%
Diana invested $61,293 in an account with
a fixed annual percent of interest,
4.5%
compounding quarterly. At the end of five
full years, she had $76,662.25 in principal 10%
plus interest. Approximately what was the
annual percent rate of interest for this 18%
account?
25.2%

EndPractice
End Practice and
and See
See Results
Results Question 619 of 737 Answer
Next Question
Submit Answer
Submit

Math 
Calculator  Flag 
0:02

Choose the option that best answers the question.

1
The average (arithmetic mean) of 4
different integers is 75. If the largest integer
19
is 90, what is the least possible value of the
smallest integer? 29

30

33

EndPractice
End Practice and
and See
See Results
Results Question 620 of 737 Answer
Next Question
Submit Answer
Submit
Magoosh

Math 
Calculator  Flag 
0:02

Choose the option that best answers the question.

164
In how many ways can 16 different gifts be
divided among four children such that each
(4!)4
child receives exactly four gifts?
16!/
(4!)4

16!/
4!

416

EndPractice
End Practice and
and See
See Results
Results Question 621 of 737 Answer
Next Question
Submit Answer
Submit

Math 
Calculator  Flag 
0:02

Choose the option that best answers the question.

12
Positive integers a, b, c, d and e are such
that a < b < c < d < e. If the average
17
(arithmetic mean) of the five numbers is 6
and d − b = 3, then what is the greatest 18
possible range of the five numbers?
19

20
EndPractice
End Practice and
and See
See Results
Results Question 622 of 737 Answer
Next Question
Submit Answer
Submit

Math 
Calculator  Flag 
0:02

Choose the option that best answers the question.

9 x 10-100
(3 × 1020
​ ) × (3 × 10−5
​ )

1 x 10-4

9 x 10-4

1 x 1015

9 x 1015

EndPractice
End Practice and
and See
See Results
Results Question 623 of 737 Answer
Next Question
Submit Answer
Submit

Math 
Calculator  Flag 
0:03

Choose the option that best answers the question.

36
x and y are positive integers such that x <
​ ​
y. If 6√​6 = x√​y , then xy could equal
48

54

96

108
EndPractice
End Practice and
and See
See Results
Results Question 624 of 737 Answer
Next Question
Submit Answer
Submit

Math 
Calculator  Flag 
0:02

Choose the option that best answers the question.

Zero
Peter invests $100,000 in an account that
pays 12% annual interest: the interest is
$68.25
paid once, at the end of the year. Martha
invests $100,000 in an account that pays $682.50
12% annual interest, compounding monthly
at the end of each month. At the end of one $6825.00
full year, compared to Peter's account,
approximately how much more does $68250.00
Martha’s account have?

EndPractice
End Practice and
and See
See Results
Results Question 625 of 737 Answer
Next Question
Submit Answer
Submit

Math 
Calculator  Flag 
0:02

Choose the option that best answers the question.

9! + 10!
A popular website requires users to create
a password consisting of digits only. If no
2 x 10!
digit may be repeated and each password
must be at least 9 digits long, how many 9! x 10!
passwords are possible?
19!

20!

EndPractice
End Practice and
and See
See Results
Results Question 626 of 737 Answer
Next Question
Submit Answer
Submit

Math 
Calculator  Flag 
0:06

Choose the option that best answers the question.

24
In how many different ways can 3 fiction
books and 3 non-fiction books be arranged
36
in a row of 6 books on a shelf such that the
fiction books are not separated, and the 72
non-fiction books are not separated?
144

288

EndPractice
End Practice and
and See
See Results
Results Question 627 of 737 Answer
Next Question
Submit Answer
Submit

Math 
Calculator  Flag 
0:02

Choose the option that best answers the question.

x
If AC = BC and CD = DE then, in terms of
x, the value of y is
180 − 2x
Note: Figure not drawn to scale
90 − 2x

4x − 180

45 + x/4

EndPractice
End Practice and
and See
See Results
Results Question 628 of 737 Answer
Next Question
Submit Answer
Submit

Math 
Calculator  Flag 
0:02

Choose the option that best answers the question.

285
Kevin drove from A to B at a constant
speed of 60 mph, turned immediately
300
around, and returned at a constant speed
of 80 mph.  Exactly 4 hours before the end 315
of his trip, he was still approaching B, only
15 miles away from it.  What is the distance 435
between A and B?  
450

EndPractice
End Practice and
and See
See Results
Results Question 629 of 737 Answer
Next Question
Submit Answer
Submit

Math 
Calculator  Flag 
0:02

Choose the option that best answers the question.


24
In how many ways can Ann, Bob, Chuck,
Don and Ed be seated in a row such that
48
Ann and Bob are not seated next to each
other? 56

72

96

EndPractice
End Practice and
and See
See Results
Results Question 630 of 737 Answer
Next Question
Submit Answer
Submit

Math 
Calculator  Flag 
0:02

Choose the option that best answers the question.

Region Q, shown here, is defined by 

(x − 6)2​ + (y − 4)2​ ≤ 100,

y ≥ 0,  and x ≥ 0.

What is the approximate area of Region


Q in square units?

between 75 and 125

between 125 and 175

between 175 and 225

between 225 and 275

between 275 and 325

EndPractice
End Practice and
and See
See Results
Results Question 631 of 737 Answer
Next Question
Submit Answer
Submit
Math 
Calculator  Flag 
0:02

Choose the option that best answers the question.

$70.40
The price of a pair of sneakers was $80 for
the last six months of last year. On January
$82.00
first, the price increased 20%. After the
price increase, an employee bought these $83.33
sneakers with a 10% employee discount.
What price did the employee pay? $86.40

$88.00

EndPractice
End Practice and
and See
See Results
Results Question 632 of 737 Answer
Next Question
Submit Answer
Submit

Math 
Calculator  Flag 
0:02

Choose the option that best answers the question.

None would increase the ratio


Erina has a bag that starts with 50 marbles
in it: 10 green marbles and 40 purple
I only
marbles.  She wants to increase the ratio of
green to purple marbles by adding some I and II only
more marbles to the bag.  Which of the
following additions would increase this I and III only
ratio?
I, II, and III
I. 3 green and 1 purple

II. 1 green and 3 purple

III. 3 green and 3 purple


EndPractice
End Practice and
and See
See Results
Results Question 633 of 737 Answer
Next Question
Submit Answer
Submit

Math 
Calculator  Flag 
0:02

Choose the option that best answers the question.

7/
In the coordinate plane, rectangular region 12

R has vertices at (0,0), (0,3), (4,3), and 5/


12
(4,0). If a point in region R is randomly
selected, what is the probability that the 3/
8
point's y-coordinate will be greater than its
x-coordinate? 1/
3

1/
4

EndPractice
End Practice and
and See
See Results
Results Question 634 of 737 Answer
Next Question
Submit Answer
Submit

Math 
Calculator  Flag 
0:06

Choose the option that best answers the question.

9x4y5
(2xy2) × (7x3y3) = 

14x4y5

9x3y6

14x3y6
14x6y6

EndPractice
End Practice and
and See
See Results
Results Question 635 of 737 Answer
Next Question
Submit Answer
Submit

Math 
Calculator  Flag 
0:02

Choose the option that best answers the question.

x < x2 < x3
If −1 < x < 0, which of the following is
correct?
x3 < x2 < x

x < x3 < x2

x2 < x3 < x

x3 < x < x2

EndPractice
End Practice and
and See
See Results
Results Question 636 of 737 Answer
Next Question
Submit Answer
Submit

Math 
Calculator  Flag 
0:05

Choose the option that best answers the question.

8 × 1040​ 2 x 104

1 × 1010​
8 x 104

4 x 1020
2 x 1030

8 x 1030

EndPractice
End Practice and
and See
See Results
Results Question 637 of 737 Answer
Next Question
Submit Answer
Submit

Math 
Calculator  Flag 
0:02

Choose the option that best answers the question.

x is not an integer
The right triangle ABC has side lengths
10 – x, 9 – x, and 8 – x. Given that only one
The perimeter of ABC must be
value of x is possible, which of the following
greater than 10
statements is true?
ABC is isosceles

The area of ABC cannot be


determined from the information
given

None of the above statements is


true

EndPractice
End Practice and
and See
See Results
Results Question 638 of 737 Answer
Next Question
Submit Answer
Submit

Math 
Calculator  Flag 
0:02

Choose the option that best answers the question.


20
In how many different ways can 3 identical
green shirts and 3 identical red shirts be
40
distributed among 6 children such that
each child receives a shirt? 216

720

729

EndPractice
End Practice and
and See
See Results
Results Question 639 of 737 Answer
Next Question
Submit Answer
Submit

Math 
Calculator  Flag 
0:02

Choose the option that best answers the question.

102
The points A(0, 0), B(0, 4a – 5) and C(2a +
1, 2a + 6) form a triangle. If ∠ABC = 90∘​ ,
120
what is the area of triangle ABC?
132

144

156

EndPractice
End Practice and
and See
See Results
Results Question 640 of 737 Answer
Next Question
Submit Answer
Submit
Magoosh

Math 
Calculator  Flag 
0:02

Choose the option that best answers the question.

Cannot be determined
If x and y are positive integers, and 1 is the
greatest common divisor of x and y, what is
1
the greatest common divisor of 2x and 3y?
2

EndPractice
End Practice and
and See
See Results
Results Question 641 of 737 Answer
Next Question
Submit Answer
Submit

Math 
Calculator  Flag 
0:02

Choose the option that best answers the question.

0
Q = x3 − x

Given that x is a positive integer such that x 1


≥ 75, which of the following is the
remainder when Q is divided by 6? 3

Cannot be determined by the


information provided.
EndPractice
End Practice and
and See
See Results
Results Question 642 of 737 Answer
Next Question
Submit Answer
Submit

Math 
Calculator  Flag 
0:02

Choose the option that best answers the question.

28
Given that n = 10a + 10b + 10c, where a, b,
and c are distinct positive integers, how
36
many different positive values of n result if
n is less than 1 billion (1,000,000,000) ?  56

84

120

EndPractice
End Practice and
and See
See Results
Results Question 643 of 737 Answer
Next Question
Submit Answer
Submit

Math 
Calculator  Flag 
0:02

Choose the option that best answers the question.

1/
If points A and B are randomly placed on 4

the circumference of a circle with radius 2, 1/


3
what is the probability that the length of
chord AB is greater than 2? 1/
2

2/
3

3
/4

EndPractice
End Practice and
and See
See Results
Results Question 644 of 737 Answer
Next Question
Submit Answer
Submit

Math 
Calculator  Flag 
0:02

Choose the option that best answers the question.

10
A sum of money was distributed among
Lyle, Bob and Chloe. First, Lyle received 4
20
dollars plus one-half of what remained.
Next, Bob received 4 dollars plus one-third 26
of what remained. Finally, Chloe received
the remaining $32. How many dollars did 40
Bob receive?
52

EndPractice
End Practice and
and See
See Results
Results Question 645 of 737 Answer
Next Question
Submit Answer
Submit

Math 
Calculator  Flag 
0:02

Choose the option that best answers the question.

24
Jevan must paint 3 rooms in a house.
Room A can be painted orange, red, or
26
green, Room B can be painted orange,
white, or red, and Room C can be painted 27
white, red, or green. The 3 rooms cannot
all be painted the same color.  In how many 63
different ways could Jevan paint the 3
rooms? 64

EndPractice
End Practice and
and See
See Results
Results Question 646 of 737 Answer
Next Question
Submit Answer
Submit

Math 
Calculator  Flag 
0:02

Choose the option that best answers the question.

3
If k is an integer and 121 < k2 < 225, then k
can have at most how many values?
4

EndPractice
End Practice and
and See
See Results
Results Question 647 of 737 Answer
Next Question
Submit Answer
Submit

Math 
Calculator  Flag 
0:02

Choose the option that best answers the question.

I only
If n = 2×3×5×7×11×13×17, then which of
the following statements must be true?
II only
I. n2 is divisible by 600
III only
II. n + 19 is divisible by 19
n+4 I and III
III.  ​ is even
2
None of the above

EndPractice
End Practice and
and See
See Results
Results Question 648 of 737 Answer
Next Question
Submit Answer
Submit

Math 
Calculator  Flag 
0:02

Choose the option that best answers the question.

10%
If three primes are randomly selected from
the prime numbers less than 30 and no
27%
prime can be chosen more than once, what
is the probability that the sum of the three 30%
prime numbers selected will be even? 
36.5%

42%

EndPractice
End Practice and
and See
See Results
Results Question 649 of 737 Answer
Next Question
Submit Answer
Submit

Math 
Calculator  Flag 
0:03

Choose the option that best answers the question.

26
The hypotenuse of a right triangle is 16 ft
longer than the length of the shorter leg.  If
32
the area of this triangle is exactly 120 ft2,
what is the length of the hypotenuse in 40
feet? 
64

80

EndPractice
End Practice and
and See
See Results
Results Question 650 of 737 Answer
Next Question
Submit Answer
Submit

Math 
Calculator  Flag 
0:06

Choose the option that best answers the question.

9
If A, B and C represent different digits in
the multiplication, then A + B + C =
12

14

15

17

EndPractice
End Practice and
and See
See Results
Results Question 651 of 737 Answer
Next Question
Submit Answer
Submit

Math 
Calculator  Flag 
0:03

Choose the option that best answers the question.


25%
All the cars in the lot of a particular
dealership have either a manual
37.5%
transmission or an automatic transmission,
and all have either two doors or four doors. 50%
 In this lot, 60% of the cars have a manual
transmission and 70% of the cars have four 62.5%
doors. If 90% of the cars in the lot have
either a manual transmission or four doors 75%
or both, then what percentage of the
automatic transmission cars have four
doors?

EndPractice
End Practice and
and See
See Results
Results Question 652 of 737 Answer
Next Question
Submit Answer
Submit

Math 
Calculator  Flag 
0:06

Choose the option that best answers the question.

15
There are 10 people in a room. If each
person shakes hands with exactly 3 other
30
people, what is the total number of
handshakes? 45

60

120

EndPractice
End Practice and
and See
See Results
Results Question 653 of 737 Answer
Next Question
Submit Answer
Submit


Calculator  Flag 
0:02
Math

Choose the option that best answers the question.

25
Paracelsus University has two kinds of
professors, academic professors and
37.5
professional professors.  At Paracelsus
University, 60% of the professors are 50
academic professors, and 70% of the
professors are tenured.  If 90% of the 62.5
professors at Paracelsus University are
academic professors or tenured or both, 75
then what percent of the professional
professors there are tenured?

EndPractice
End Practice and
and See
See Results
Results Question 654 of 737 Answer
Next Question
Submit Answer
Submit

Math 
Calculator  Flag 
0:02

Choose the option that best answers the question.

190
How many integers between 1 and 1021
are such that the sum of their digits is 2?
210

211

230

231

EndPractice
End Practice and
and See
See Results
Results Question 655 of 737 Answer
Next Question
Submit Answer
Submit
Math 
Calculator  Flag 
0:03

Choose the option that best answers the question.

$239.47
Sarah invested $38,700 in an account that
paid 6.2% annual interest, compounding
$714.73
monthly. She left the money in this
account, collecting interest for three full $2793.80
years. Approximately how much interest
did she earn in the last month of this $7,888.83
period?
$15,529.61

EndPractice
End Practice and
and See
See Results
Results Question 656 of 737 Answer
Next Question
Submit Answer
Submit

Math 
Calculator  Flag 
0:03

Choose the option that best answers the question.


​ √​2
x2,
Triangle ABC has sides x,  √​x​ , and
where x is an integer. What is the area of
2
ABC?  

​​√​3​

​4

2√​2

Cannot be determined from the


information provided.
EndPractice
End Practice and
and See
See Results
Results Question 657 of 737 Answer
Next Question
Submit Answer
Submit

Math 
Calculator  Flag 
0:02

Choose the option that best answers the question.

I only
If x is an odd negative integer and y is an
even integer, which of the following
II only
statements must be true?

I. (3x − 2y) is odd I and II

II. xy2 is an even negative integer I and III

III. (y2 − x) is an odd negative integer


II and III

EndPractice
End Practice and
and See
See Results
Results Question 658 of 737 Answer
Next Question
Submit Answer
Submit

Math 
Calculator  Flag 
0:02

Choose the option that best answers the question.

1
In the game of Dubblefud, red chips, blue
chips and green chips are each worth 2, 4
2
and 5 points respectively. In a certain
selection of chips, the product of the point 3
values of the chips is 16,000. If the number
of blue chips in this selection equals the 4
number of green chips, how many red
chips are in the selection? 5

EndPractice
End Practice and
and See
See Results
Results Question 659 of 737 Answer
Next Question
Submit Answer
Submit

Math 
Calculator  Flag 
0:02

Choose the option that best answers the question.

I only
Two sides of a triangle have length 6 and
8. Which of the following are possible areas
I and II only
of the triangle?

I. 2
II and III only
II. 12

III. 24 I and III only

I, II, and III

EndPractice
End Practice and
and See
See Results
Results Question 660 of 737 Answer
Next Question
Submit Answer
Submit
Magoosh

Math 
Calculator  Flag 
0:02

Choose the option that best answers the question.

10
How many integers from 1 to 900 inclusive
have exactly 3 positive divisors?
14

15

29

30

EndPractice
End Practice and
and See
See Results
Results Question 661 of 737 Answer
Next Question
Submit Answer
Submit

Math 
Calculator  Flag 
0:02

Choose the option that best answers the question.

125
If ABCD is a square with area 625, and
CEFD is a rhombus with area 500, then the
175
area of the shaded region is

Note: Figure not drawn to scale 200

250

275

EndPractice
End Practice and
and See
See Results
Results Question 662 of 737 Answer
Next Question
Submit Answer
Submit

Math 
Calculator  Flag 
0:02

Choose the option that best answers the question.

21
Given that the length of each side of a
quadrilateral is a distinct integer and that
24
the longest side is not greater than 7, how
many different possible combinations of 32
side lengths are there?
34

35

EndPractice
End Practice and
and See
See Results
Results Question 663 of 737 Answer
Next Question
Submit Answer
Submit

Math 
Calculator  Flag 
0:02

Choose the option that best answers the question.


48
Twelve points are spaced evenly around a
circle, lettered from A to L.  Let N be the
52
total number of isosceles triangles,
including equilateral triangles, that can be 60
constructed from three of these points. A
different orientation of the same lengths 72
counts as a different triangle, because a
different combination of points form the 120
vertices.   What is the value of N?

EndPractice
End Practice and
and See
See Results
Results Question 664 of 737 Answer
Next Question
Submit Answer
Submit

Math 
Calculator  Flag 
0:02

Choose the option that best answers the question.

24
How many positive integers less than
10,000 are such that the product of their
30
digits is 210?
48

54

72

EndPractice
End Practice and
and See
See Results
Results Question 665 of 737 Answer
Next Question
Submit Answer
Submit

Math 
Calculator  Flag 
0:01
Choose the option that best answers the question.

60 mph
Car X and Y are traveling from A to B on
the same route at constant speeds.  Car X
70 mph
is initially behind Car Y, but Car X's speed
is 1.25 times Car Y's speed.  Car X passes 80 mph
Car Y at 1:30 pm.  At 3:15 pm, Car X
reaches B, and at that moment, Car Y is 90 mph
still 35 miles away from B.  What is the
speed of Car X?   100 mph

EndPractice
End Practice and
and See
See Results
Results Question 666 of 737 Answer
Next Question
Submit Answer
Submit

Math 
Calculator  Flag 
0:02

Choose the option that best answers the question.

25,000
25,0022 – 24,9982 =

50,000

100,000

200,000

400,000

EndPractice
End Practice and
and See
See Results
Results Question 667 of 737 Answer
Next Question
Submit Answer
Submit

Math 
Calculator  Flag 
0:02
Choose the option that best answers the question.

63,000,000
If 3x = 7y = 100, then 63x2y =

1,000,000

333,333

142,857

47,609

EndPractice
End Practice and
and See
See Results
Results Question 668 of 737 Answer
Next Question
Submit Answer
Submit

Math 
Calculator  Flag 
0:02

Choose the option that best answers the question.

4
When a large municipal water tank is
empty, it takes a Type JQ pump, working
6
alone, 72 hours to fill the tank, whereas as
Type JT pump, working alone, would take 9
only 18 hours to fill the tank completely.  If
the tank starts at half full, how long would it 12
take two Type JQ pumps and a Type JT
pump, all three pumps working together, to 24
fill the tank?

EndPractice
End Practice and
and See
See Results
Results Question 669 of 737 Answer
Next Question
Submit Answer
Submit
Math 
Calculator  Flag 
0:02

Choose the option that best answers the question.

1
A grocery store sells boxes of two kinds of ​
12
cereals, wheat-based and rice-based.  In
this store, the average weight of a box of 1

6
rice-based cereal is 60 ounces, the
average weight of a box of wheat-based 1

cereal is 48 ounces, and the average 5
weight of all the boxes of cereal is 50 1

ounces.  What is the ratio in this store of 4
boxes of rice-based cereals to wheat- 1
based cereals? ​
3

EndPractice
End Practice and
and See
See Results
Results Question 670 of 737 Answer
Next Question
Submit Answer
Submit

Math 
Calculator  Flag 
0:02

Choose the option that best answers the question.

5
A large aquarium contains only two kinds of ​
12
3
fish, guppies and swordtails.  If ​ of the
4 8
2 ​
number of guppies is equal to ​ of the 17
3
number of swordtails, then what fraction of 1

fish in this aquarium are guppies? 2
7

12
8

9

EndPractice
End Practice and
and See
See Results
Results Question 671 of 737 Answer
Next Question
Submit Answer
Submit

Math 
Calculator  Flag 
0:02

Choose the option that best answers the question.

$18
Andrea makes silver pendants in the shape
shown below with dimensions given in
$20
centimeters. Silver tubing used to create
the 4 sides of the rectangle costs $4 per $110
centimeter of length, and silver tubing used
to construct the two congruent sides of the $142
isosceles triangle costs $3 per centimeter
of length. What is the total cost of the silver $164
tubing needed to construct one pendant?

EndPractice
End Practice and
and See
See Results
Results Question 672 of 737 Answer
Next Question
Submit Answer
Submit

Math 
Calculator  Flag 
0:02
Choose the option that best answers the question.

1 1 1/8
If  ​ = 0.4 , then  ​ =
x x+2
1/5

2/9

1/4

2/7

EndPractice
End Practice and
and See
See Results
Results Question 673 of 737 Answer
Next Question
Submit Answer
Submit

Math 
Calculator  Flag 
0:02

Choose the option that best answers the question.

(k3​ × k × k4​ )2​ -1


If k ≠ 0, k ≠ ±1, and  ​ = k14
​ ,
kn​ × k
then n = 1

49

129

EndPractice
End Practice and
and See
See Results
Results Question 674 of 737 Answer
Next Question
Submit Answer
Submit
Math 
Calculator  Flag 
0:03

Choose the option that best answers the question.

​ ​ ​ ​ 1
(2√​3 + √​5 )(2√​3 − √​5 ) =
7

4√​15

12 − 4√​15

17

EndPractice
End Practice and
and See
See Results
Results Question 675 of 737 Answer
Next Question
Submit Answer
Submit

Math 
Calculator  Flag 
0:02

Choose the option that best answers the question.

1011
If 10a x 10b x 10c = 1,000,000, and a, b,
and c are different positive integers, then
1100
10a + 10b + 10c =
1101

1110

1111

EndPractice
End Practice and
and See
See Results
Results Question 676 of 737 Answer
Next Question
Submit Answer
Submit
Math 
Calculator  Flag 
0:06

Choose the option that best answers the question.

1 11 7
(1 ​ ) × (1 ​ ) 1​
6 21 9
11
1​
14
29
1​
42
11
1​
126
29
2​
42

EndPractice
End Practice and
and See
See Results
Results Question 677 of 737 Answer
Next Question
Submit Answer
Submit

Math 
Calculator  Flag 
0:03

Choose the option that best answers the question.

4
In order to qualify for the year-end tennis
tournament, Sam must win at least 60
5
percent of his matches this year. Presently
Sam has won 14 of his 18 matches. Of 6
Sam’s 13 matches remaining in the year,
what is the least number that he must win 7
in order to qualify for the year-end
tournament? 8
EndPractice
End Practice and
and See
See Results
Results Question 678 of 737 Answer
Next Question
Submit Answer
Submit

Math 
Calculator  Flag 
0:02

Choose the option that best answers the question.

52
A, B, and C are consecutive odd integers
such that A < B < C.
54
If A + B + C = 81, then A + C =
56

58

60

EndPractice
End Practice and
and See
See Results
Results Question 679 of 737 Answer
Next Question
Submit Answer
Submit

Math 
Calculator  Flag 
0:02

Choose the option that best answers the question.

11
If the average (arithmetic mean) of x, y,
and 20 is 11, then the average of 2x + 3, 2y
12
– 4, and 8 is
13

14

15
EndPractice
End Practice and
and See
See Results
Results Question 680 of 737 Answer
Next Question
Submit Answer
Submit
Magoosh

Math 
Calculator  Flag 
0:02

Choose the option that best answers the question.

(x−5​ )2​ (x4​ )3​ x2​



x3​ x−5​
x3​
For all values of x ≠ 0 , the above
expression simplifies to which of the x4​
following? 
x5​

x6​

EndPractice
End Practice and
and See
See Results
Results Question 681 of 737 Answer
Next Question
Submit Answer
Submit

Math 
Calculator  Flag 
0:03

Choose the option that best answers the question.

4
In a group of 50 students, 31 are taking
French, 17 are taking Spanish, and 10 are
8
taking neither French nor Spanish. How
many students are taking both French and 12
Spanish?
14

16
EndPractice
End Practice and
and See
See Results
Results Question 682 of 737 Answer
Next Question
Submit Answer
Submit

Math 
Calculator  Flag 
0:01

Choose the option that best answers the question.

–1
If  x3 = –1, what
does x2 – 4x + 4 equal?

EndPractice
End Practice and
and See
See Results
Results Question 683 of 737 Answer
Next Question
Submit Answer
Submit

Math 
Calculator  Flag 
0:06

Choose the option that best answers the question.

Alice's trip took less time than


Alice traveled on a straight line path at a
Bert's.
constant speed V1 from City P to City Q.
 Bert traveled on a straight line path from
Alice's trip took more time than
City P to City R, and then on straight line
Bert's.
path from City R to City Q; throughout his
whole trip, Bert traveled at a constant The two trips took the same time.
speed V2 > V1. Given that Cities P, Q, and
R do not lie all on the same line, which of Alice's trip covered less distance
the following statements must be true? than Bert's.

Alice's trip covered more distance


than Bert's.

EndPractice
End Practice and
and See
See Results
Results Question 684 of 737 Answer
Next Question
Submit Answer
Submit

Math 
Calculator  Flag 
0:02

Choose the option that best answers the question.

A
Which of the following best represents the
quotient P/Q?
B

EndPractice
End Practice and
and See
See Results
Results Question 685 of 737 Answer
Next Question
Submit Answer
Submit

Math 
Calculator  Flag 
0:01

Choose the option that best answers the question.

k2 − 4
If k is an odd integer, which of the following
must be an even integer?
3k + 2
2k + 1

12k
/8

6k/
3

EndPractice
End Practice and
and See
See Results
Results Question 686 of 737 Answer
Next Question
Submit Answer
Submit

Math 
Calculator  Flag 
0:02

Choose the option that best answers the question.

25
What is the value of y?

Note: Figure not drawn to scale 27

29

31

33

EndPractice
End Practice and
and See
See Results
Results Question 687 of 737 Answer
Next Question
Submit Answer
Submit

Math 
Calculator  Flag 
0:02

Choose the option that best answers the question.

-3
If 4x – 3y = 13 and 5x + 2y =–1, then x =
-1

EndPractice
End Practice and
and See
See Results
Results Question 688 of 737 Answer
Next Question
Submit Answer
Submit

Math 
Calculator  Flag 
0:02

Choose the option that best answers the question.

51
A shipment of watermelons weighs 899
pounds.  If each watermelon weighs at
52
least 15 pounds, what is the greatest
number of watermelons that could be in the 59
shipment?
60

61

EndPractice
End Practice and
and See
See Results
Results Question 689 of 737 Answer
Next Question
Submit Answer
Submit

Math 
Calculator  Flag 
0:06

Choose the option that best answers the question.


250
If 3x = 2y = 5, then 24xy2 =

500

750

1000

1250

EndPractice
End Practice and
and See
See Results
Results Question 690 of 737 Answer
Next Question
Submit Answer
Submit

Math 
Calculator  Flag 
0:02

Choose the option that best answers the question.

228 4
​ = ​
494 7
11

21
6

13
14

27
9

17

EndPractice
End Practice and
and See
See Results
Results Question 691 of 737 Answer
Next Question
Submit Answer
Submit

Math 
Calculator  Flag 
0:02
Choose the option that best answers the question.

x > 10
Which of the following inequalities is
equivalent to 12 – 3x < –18
x < 10

x > -10

x < -10

x>2

EndPractice
End Practice and
and See
See Results
Results Question 692 of 737 Answer
Next Question
Submit Answer
Submit

Math 
Calculator  Flag 
0:02

Choose the option that best answers the question.

Cannot be determined
If the average (arithmetic mean) of b and c
is 5, and the average of c and d is 10, then
−5
b-d=
−10

−15

−20

EndPractice
End Practice and
and See
See Results
Results Question 693 of 737 Answer
Next Question
Submit Answer
Submit

Math 
Calculator  Flag 
0:02
Choose the option that best answers the question.

7 15 10 1
If x =  ​ − ​ + ​ , then (1 – x)2 = 

9 18 12 9
4

81
25

144
9

16
25

36

EndPractice
End Practice and
and See
See Results
Results Question 694 of 737 Answer
Next Question
Submit Answer
Submit

Math 
Calculator  Flag 
0:03

Choose the option that best answers the question.

The microcurrent through the electrode in a 3.6 × 10−8000



delicate circuit is usually held constant at
3.6 × 10−24

3.6 x 10–8 amps. Because of a defect in
another part of the circuit, the current was 3.6 × 10−11

1000 times smaller. What was the current,
in amps, caused by this defect? 3.6 × 10−5

3.6 × 10−8/3

EndPractice
End Practice and
and See
See Results
Results Question 695 of 737 Answer
Next Question
Submit Answer
Submit
Math 
Calculator  Flag 
0:02

Choose the option that best answers the question.


Given that x > 0, if x4 = y16, then y = √​4​ x​

√​​x

x2​

x4​

x12

EndPractice
End Practice and
and See
See Results
Results Question 696 of 737 Answer
Next Question
Submit Answer
Submit

Math 
Calculator  Flag 
0:03

Choose the option that best answers the question.

1
If 6k2 + k = 2 and k > 0, then k must equal ​
2
which of the following?  
1

3

2

End Practice and See Results Question 697 of 737 Submit Answer
Next Question
End Practice and See Results Submit Answer

Math 
Calculator  Flag 
0:02

Choose the option that best answers the question.

compounding annually
If $5,000,000 is the initial amount placed in
an account that collects 7% annual interest,
compounding quarterly
which of the following compounding rates
would produce the largest total amount compounding monthly
after two years?
compounding daily

All four of these would produce the


same total

EndPractice
End Practice and
and See
See Results
Results Question 698 of 737 Answer
Next Question
Submit Answer
Submit

Math 
Calculator  Flag 
0:02

Choose the option that best answers the question.

(–10, 0)
In the xy-coordinate system, line k has
1
slope ​  and passes through point (0, 5). (8, 9)
2
Which of the following points cannot lie on
line k? (3, 6.5)

(–2, 2)
(–8, 1)

EndPractice
End Practice and
and See
See Results
Results Question 699 of 737 Answer
Next Question
Submit Answer
Submit

Math 
Calculator  Flag 
0:02

Choose the option that best answers the question.

1:2
In a jar, there are 24 marbles, each of
which is either red or blue.  Which of the
3:5
following CANNOT be the ratio of red
marbles to blue marbles? 1:1

4:3

7:5

EndPractice
End Practice and
and See
See Results
Results Question 700 of 737 Answer
Next Question
Submit Answer
Submit
Magoosh

Math 
Calculator  Flag 
0:02

Choose the option that best answers the question.

125
If m and n are integers, and m =
(48)(35)(51)
​ , which of the following could 126
68n
be a value of n?
127

128

129

EndPractice
End Practice and
and See
See Results
Results Question 701 of 737 Answer
Next Question
Submit Answer
Submit

Math 
Calculator  Flag 
0:02

Choose the option that best answers the question.

27
Joan is allowed to invite 3 of her friends to
join her on a family camping trip. If Joan
120
has 10 friends, in how many ways can she
invite 3 of them? 240

360

720
EndPractice
End Practice and
and See
See Results
Results Question 702 of 737 Answer
Next Question
Submit Answer
Submit

Math 
Calculator  Flag 
0:02

Choose the option that best answers the question.

For which of the following functions is  f(x) = 3x2​


1
f ( − ​ ) > f(2)? f(x) = 3x
2

f(x) = 3 + x2​

1
f(x) = 3 + ​
x
3
f(x) = ​ 2
x​

EndPractice
End Practice and
and See
See Results
Results Question 703 of 737 Answer
Next Question
Submit Answer
Submit

Math 
Calculator  Flag 
0:03

Choose the option that best answers the question.

55
A helicopter company charges $85 for the
first kilometer of a trip and $5 for every
56
kilometer after that. If the total cost of a trip
was $365, how many kilometers were 57
flown?
58
59

EndPractice
End Practice and
and See
See Results
Results Question 704 of 737 Answer
Next Question
Submit Answer
Submit

Math 
Calculator  Flag 
0:06

Choose the option that best answers the question.

1:8
If the areas of the 4 squares are 50, 32, 18
and 12, what is the ratio of the area of
1:6
the small shaded portion to the area of the
large shaded portion? 1:4
Note: Figure not drawn to scale
1:3

1:1

EndPractice
End Practice and
and See
See Results
Results Question 705 of 737 Answer
Next Question
Submit Answer
Submit

Math 
Calculator  Flag 
0:02

Choose the option that best answers the question.


x−2
Which of the following is equivalent to  ​
x−4
2x2​ + 8x − 24
​ 2 for all values of x for which
2x ​ + 20x − 48 x−2

both expressions are defined? x+4
x+2

x+4
x+6

x − 12
x+6

x + 12

EndPractice
End Practice and
and See
See Results
Results Question 706 of 737 Answer
Next Question
Submit Answer
Submit

Math 
Calculator  Flag 
0:02

Choose the option that best answers the question.

2:7
If AD = 3x, AE = 5x, EB = y and DC = 5y,
what is the ratio of the area of triangle DEC
1:3
to the area of rectangle ABCD ?

Note: Figure not drawn to scale 2:5

1:2

3:5

EndPractice
End Practice and
and See
See Results
Results Question 707 of 737 Answer
Next Question
Submit Answer
Submit
Math 
Calculator  Flag 
0:02

Choose the option that best answers the question.

(4,7)
In the xy-coordinate system, the distance

between the point (0,0) and point P is  √​40 .
(4,10)
Which of the following could be the
coordinates of point P? (5,6)

(6,2)

(20,20)

EndPractice
End Practice and
and See
See Results
Results Question 708 of 737 Answer
Next Question
Submit Answer
Submit

Math 
Calculator  Flag 
0:02

Choose the option that best answers the question.

7/
A bag contains x blue chips and y red 11

chips. If the probability of selecting a red 3/


4
chip at random is 3/7, then x/y =
7/
4

4/
3

11/
7

EndPractice
End Practice and
and See
See Results
Results Question 709 of 737 Answer
Next Question
Submit Answer
Submit
Math 
Calculator  Flag 
0:08

Choose the option that best answers the question.

–​22
If f(x) = x3 – 5 and f(k) = 3 then k =

22

EndPractice
End Practice and
and See
See Results
Results Question 710 of 737 Answer
Next Question
Submit Answer
Submit

Math 
Calculator  Flag 
0:02

Choose the option that best answers the question.

-6
(3−1
​ − 2−1
​ )−1
​ =

-5

-1/6

1/
6

End Practice and See Results Question 711 of 737 Submit Answer
Next Question
End Practice and See Results Submit Answer

Math 
Calculator  Flag 
0:02

Choose the option that best answers the question.

26
A retailer purchases shirts from a
wholesaler and then sells the shirts in her
37.5
store at a retail price that is 80 percent
greater than the wholesale price. If the 42
retailer decreases the retail price by 30
percent this will have the same effect as 44
increasing the wholesale price by what
percent? 50

EndPractice
End Practice and
and See
See Results
Results Question 712 of 737 Answer
Next Question
Submit Answer
Submit

Math 
Calculator  Flag 
0:02

Choose the option that best answers the question.

x 2
If y = ​ , then what is the ratio of 2x to 3y? ​
5 15
3

10
3

2
10

3
15

2

EndPractice
End Practice and
and See
See Results
Results Question 713 of 737 Answer
Next Question
Submit Answer
Submit

Math 
Calculator  Flag 
0:02

Choose the option that best answers the question.

1.5
If x2 – y2 = 12 and x – y = 4, then x = 

2.5

3.5

4.5

5.5

EndPractice
End Practice and
and See
See Results
Results Question 714 of 737 Answer
Next Question
Submit Answer
Submit

Math 
Calculator  Flag 
0:02

Choose the option that best answers the question.

​ 2
If pq = 4√​6 and p2 = 12, then which of the
following is the value of q2?  
4

4√​3

8

8√​3

EndPractice
End Practice and
and See
See Results
Results Question 715 of 737 Answer
Next Question
Submit Answer
Submit

Math 
Calculator  Flag 
0:02

Choose the option that best answers the question.

Cannot be determined
The numbers p and q are both positive. If p
percent of 160 equals q percent of 40, then
1/4
p/q =
2/5

5/2

EndPractice
End Practice and
and See
See Results
Results Question 716 of 737 Answer
Next Question
Submit Answer
Submit

Math 
Calculator  Flag 
0:02

Choose the option that best answers the question.

mean = 81 and median = 81


In a certain class, 41 students took a test
given on Monday and, for those 41 scores,
mean < 81 and median = 81
the test had mean = median = 81.  Two
additional students, absent on Monday, mean = 81 and median < 81
took the test on Tuesday: they received
grades of 83 and 47.  What can we say mean < 81 and median < 81
about the new mean & median? 
cannot be determined from the
information given

EndPractice
End Practice and
and See
See Results
Results Question 717 of 737 Answer
Next Question
Submit Answer
Submit

Math 
Calculator  Flag 
0:02

Choose the option that best answers the question.

90
For a long time, the price of a certain
console remained the same. Because of
180
new tariffs, the price of this console
increased by 50% last week, and stayed at 240
this new level. This week, Amanda
purchased the console with a 50%-off 320
coupon. Amanda paid $240. What was the
original price, before the last week's price 360
increase?

EndPractice
End Practice and
and See
See Results
Results Question 718 of 737 Answer
Next Question
Submit Answer
Submit

Math 
Calculator  Flag 
0:02

Choose the option that best answers the question.


1/4
0.25% =

1/40

1/400

1/4000

1/40000

EndPractice
End Practice and
and See
See Results
Results Question 719 of 737 Answer
Next Question
Submit Answer
Submit

Math 
Calculator  Flag 
0:02

Choose the option that best answers the question.

0.05
The probability that event A occurs is 0.4,
and the probability that events A and B
0.15
both occur is 0.25. If the probability that
either event A or event B occurs is 0.6, 0.45
what is the probability that event B will
occur? 0.50

0.55

EndPractice
End Practice and
and See
See Results
Results Question 720 of 737 Answer
Next Question
Submit Answer
Submit
Magoosh

Math 
Calculator  Flag 
0:02

Choose the option that best answers the question.

15
Ben is three times as old as Ron. Ed is 8
years younger than Ben. Ron is 7 years
19
older than Ken. If the sum of the ages of all
four people is 161, how many years old is 22
Ron?
24

27

EndPractice
End Practice and
and See
See Results
Results Question 721 of 737 Answer
Next Question
Submit Answer
Submit

Math 
Calculator  Flag 
0:02

Choose the option that best answers the question.

x−3 x+3 1
If x ≠ -2, x ≠ 7 and  ​ = ​ , then x =
x+2 x−7
2

5
EndPractice
End Practice and
and See
See Results
Results Question 722 of 737 Answer
Next Question
Submit Answer
Submit

Math 
Calculator  Flag 
0:02

Choose the option that best answers the question.

3.4
If an object travels 100 feet in 2 seconds,
what is the object's approximate speed in
3.8
miles per hour? (Note: 1 mile = 5280 feet)
34

38

340

EndPractice
End Practice and
and See
See Results
Results Question 723 of 737 Answer
Next Question
Submit Answer
Submit

Math 
Calculator  Flag 
0:06

Choose the option that best answers the question.

1
In a school outing with only adults and ​
12
young children, the average weight of the
adults is 60 kg and the average weight of 1

6
the children is 24 kg.  If the average weight
of everyone on the school outing is 30 kg, 1

what is the ratio of adults to children on this 5
outing? 1

4
1

3

EndPractice
End Practice and
and See
See Results
Results Question 724 of 737 Answer
Next Question
Submit Answer
Submit

Math 
Calculator  Flag 
0:02

Choose the option that best answers the question.

30
If a triangle in the xy-coordinate system has
vertices at
36
(-2 , -3), (4, -3) and (28, 7), what is the area
of the triangle? 48

60

65

EndPractice
End Practice and
and See
See Results
Results Question 725 of 737 Answer
Next Question
Submit Answer
Submit

Math 
Calculator  Flag 
0:02

Choose the option that best answers the question.

​ 1
8a​ b( ​ + c)
If 8c​ × √​8 = ​ b  then a = 2
8​
bc

2
b+c

2

2b + c

1
​ +b+c
2

EndPractice
End Practice and
and See
See Results
Results Question 726 of 737 Answer
Next Question
Submit Answer
Submit

Math 
Calculator  Flag 
0:04

Choose the option that best answers the question.

1 2
1 4
If ( ​ + x)​ = 16, then  ​ + x2​ =
x x2​
8

14

16

18

EndPractice
End Practice and
and See
See Results
Results Question 727 of 737 Answer
Next Question
Submit Answer
Submit

You might also like